Вы находитесь на странице: 1из 73

company used it to entertain guests, to accommodate officials, and to

COLLECTOR V. HENDERSON entertain customers. According to taxpayers, only P 4,800 per year is
the reasonable amount that the spouses would be spending on rental
Rental and Travel Allowance are not Part of Taxable Income if they were not required to live in those apartments. Thus, it is the
amount they deem is subject to tax. The excess is to be treated as
expense of the company.

Rental allowances and travel allowances by a company are not part of


taxable income.
2) The entrance fee should not be considered income since it is an
expense of his employer, and membership therein is merely incidental
to his duties of increasing and sustaining the business of his employer.
FACTS:

3) His wife merely accompanied him to New York on a business trip as


• Sps. Arthur Henderson and Marie Henderson filed their annual his secretary, and at the employer-corporation’s request, for the wife to
income tax with the BIR. Arthur is president of American International look at details of the plans of a building that his employer intended to
Underwriters for the Philippines, Inc., which is a domestic corporation construct. Such must not be considered taxable income.
engaged in the business of general non-life insurance, and represents
a group of American insurance companies engaged in the business of
general non-life insurance.

• The Collector of Internal Revenue merely allowed the entrance fee as


• The BIR demanded payment for alleged deficiency taxes. In their nontaxable. The rent expense and travel expenses were still held to be
computation, the BIR included as part of taxable income: 1) Arthur’s taxable. The Court of Tax Appeals ruled in favor of the taxpayers, that
allowances for rental, residential expenses, subsistence, water, such expenses must not be considered part of taxable income. Letters
electricity and telephone expenses 2) entrance fee to the Marikina Gun of the wife while in New York concerning the proposed building were
and Country Club which was paid by his employer for his account and presented as evidence.
3) travelling allowance of his wife

• The taxpayers justifications are as follows:


ISSUE: Whether or not the rental allowances and travel allowances
furnished and given by the employer-corporation are part of taxable
income?
1) as to allowances for rental and utilities, Arthur did not receive money
for the allowances. Instead, the apartment is furnished and paid for by
his employer-corporation (the mother company of American
International), for the employer corporation’s purposes. The spouses
had no choice but to live in the expensive apartment, since the
HELD: NO. Such claims are substantially supported by evidence.

These claims are therefore NOT part of taxable income. No part of the
allowances in question redounded to their personal benefit, nor were
such amounts retained by them. These bills were paid directly by the
employer-corporation to the creditors. The rental expenses and
subsistence allowances are to be considered not subject to income tax.
Arthur’s high executive position and social standing, demanded and
compelled the couple to live in a more spacious and expensive
quarters. Such ‘subsistence allowance’ was a SEPARATE account
from the account for salaries and wages of employees. The company
did not charge rentals as deductible from the salaries of the employees.
These expenses are COMPANY EXPENSES, not income by
employees which are subject to tax.
46,290 and 46,287 shares were respectively received by the Don
Andres estate and Doña Carmen from ANSCOR. Hence, increasing
GENERAL RULE: A stock dividend representing the transfer of surplus their accumulated shareholdings to 138,867 and 138,864 common
to capital account shall not be subject to tax. shares each.

EXCEPTION: The redemption or cancellation of stock dividends,


depending on the "time" and "manner" it was made, is essentially • On June 30, 1968, pursuant to a Board Resolution, ANSCOR
equivalent to a distribution of taxable dividends," making the proceeds redeemed 28,000 common shares from the Don Andres' estate. By
thereof "taxable income" "to the extent it represents profits". November 1968, the Board further increased ANSCOR's capital stock
to P75M. About a year later, ANSCOR again redeemed 80,000
common shares from the Don Andres' estate. As stated in the Board
FACTS: -- reversal of the decision of the CA Resolutions, ANSCOR's business purpose for both redemptions of
stocks is to partially retire said stocks as treasury shares in order to
reduce the company's foreign exchange remittances in case cash
• Don Andres Soriano, a citizen and resident of the United States, dividends are declared.
formed the corporation "A. Soriano Y Cia", predecessor of ANSCOR,
with a P1,000,000.00 capitalization divided into 10,000 common shares
at a par value of P100/share. ANSCOR is wholly owned and controlled
by the family of Don Andres, who are all nonresident aliens. • In 1973, after examining ANSCOR's books of account and records,
Revenue examiners issued a report proposing that ANSCOR be
assessed for deficiency withholding tax-at-source, pursuant to Sections
53 and 54 of the 1939 Revenue Code for the year 1968 and the second
• In 1937, Don Andres subscribed to 4,963 shares of the 5,000 shares quarter of 1969 based on the transactions of exchange and redemption
originally issued. In 1945, ANSCOR's authorized capital stock was of stocks.
increased to P2,500,000.00 divided into 25,000 common shares with
the same par value. Don Andres' increased his subscription to 14,963
common shares. A month later, Don Andres transferred 1,250 shares
each to his two sons, Jose and Andres, Jr., as their initial investments ISSUE:
in ANSCOR. Both sons are foreigners.

• Whether or not ANSCOR's redemption of stocks from its stockholder


• From 1947-1963, ANSCOR declared stock dividends. On December as well as the exchange of common with preferred shares can be
30, 1964 Don Andres died. As of that date, the records revealed that considered as "essentially equivalent to the distribution of taxable
he has a total shareholdings of 185,154 shares. Correspondingly, one- dividend" making the proceeds thereof taxable.
half of that shareholdings or 92,577 shares were transferred to his wife,
Doña Carmen Soriano, as her conjugal share. The other half formed
part of his estate.
HELD:

• A day after Don Andres died, ANSCOR increased its capital stock to • YES. The bone of contention is the interpretation and application of
P20M and in 1966 further increased it to P30M. Stock dividends worth Section 83(b) of the 1939 Revenue Act 38 which provides:
and can exercise the freedom of choice. Having realized gain from that
• Sec. 83. Distribution of dividends or assets by corporations. — (b) redemption, the income earner cannot escape income tax. For the
Stock dividends — A stock dividend representing the transfer of surplus exempting clause of Section, 83(b) to apply, it is indispensable that: (a)
to capital account shall not be subject to tax. However, if a corporation there is redemption or cancellation; (b) the transaction involves stock
cancels or redeems stock issued as a dividend at such time and in such dividends and (c) the "time and manner" of the transaction makes it
manner as to make the distribution and cancellation or redemption, in "essentially equivalent to a distribution of taxable dividends."
whole or in part, essentially equivalent to the distribution of a taxable
dividend, the amount so distributed in redemption or cancellation of the
stock shall be considered as taxable income to the extent it represents • Redemption is repurchase, a reacquisition of stock by a corporation
a distribution of earnings or profits accumulated after March first, which issued the stock 89 in exchange for property, whether or not the
nineteen hundred and thirteen. acquired stock is cancelled, retired or held in the treasury. 90
Essentially, the corporation gets back some of its stock, distributes
cash or property to the shareholder in payment for the stock, and
• Sec. 83(b) of the 1939 NIRC was taken from the Section 115(g)(1) of continues in business as before. In the case, ANSCOR redeemed
the U.S. Revenue Code of 1928. It laid down the general rule known as shares twice. But where did the shares redeemed come from? If its
the proportionate test wherein stock dividends once issued form part of source is the original capital subscriptions upon establishment of the
the capital and, thus, subject to income tax. Specifically, the general corporation or from initial capital investment in an existing enterprise,
rule states that: A stock dividend representing the transfer of surplus to its redemption to the concurrent value of acquisition may not invite the
capital account shall not be subject to tax. application of Sec. 83(b) under the 1939 Tax Code, as it is not income
but a mere return of capital. On the contrary, if the redeemed shares
are from stock dividend declarations other than as initial capital
• Stock dividends, strictly speaking, represent capital and do not investment, the proceeds of the redemption is additional wealth, for it
constitute income to its recipient. So that the mere issuance thereof is is not merely a return of capital but a gain thereon.
not yet subject to income tax as they are nothing but an "enrichment
through increase in value of capital investment."
• It is not the stock dividends but the proceeds of its redemption that
may be deemed as taxable dividends. At the time of the last
redemption, the original common shares owned by the estate were only
• The exception provides that the redemption or cancellation of stock 25,247.5 91 This means that from the total of 108,000 shares redeemed
dividends, depending on the "time" and "manner" it was made, is from the estate, the balance of 82,752.5 (108,000 less 25,247.5) must
essentially equivalent to a distribution of taxable dividends," making the have come from stock dividends. In the absence of evidence to the
proceeds thereof "taxable income" "to the extent it represents profits". contrary, the Tax Code presumes that every distribution of corporate
The exception was designed to prevent the issuance and cancellation property, in whole or in part, is made out of corporate profits such as
or redemption of stock dividends, which is fundamentally not taxable, stock dividends. The capital cannot be distributed in the form of
from being made use of as a device for the actual distribution of cash redemption of stock dividends without violating the trust fund doctrine.
dividends, which is taxable.

• Simply put, depending on the circumstances, the proceeds of • With respect to the third requisite, ANSCOR redeemed stock
redemption of stock dividends are essentially distribution of cash dividends issued just 2 to 3 years earlier. The time alone that lapsed
dividends, which when paid becomes the absolute property of the from the issuance to the redemption is not a sufficient indicator to
stockholder. Thereafter, the latter becomes the exclusive owner thereof determine taxability. It is a must to consider the factual circumstances
as to the manner of both the issuance and the redemption. The
issuance of stock dividends and its subsequent redemption must be • Even if the said purposes support the redemption and justify the
separate, distinct, and not related, for the redemption to be considered issuance of stock dividends, the same has no bearing whatsoever on
a legitimate tax scheme. Redemption cannot be used as a cloak to the imposition of the tax herein assessed because the proceeds of the
distribute corporate earnings. redemption are deemed taxable dividends since it was shown that
income was generated therefrom.

• ANSCOR invoked two reasons to justify the redemptions — (1) the • The proceeds thereof are essentially considered equivalent to a
alleged "filipinization" program and (2) the reduction of foreign distribution of taxable dividends. As "taxable dividend" under Section
exchange remittances in case cash dividends are declared. The Court 83(b), it is part of the "entire income" subject to tax under Section 22 in
is not concerned with the wisdom of these purposes but on their relation to Section 21 120 of the 1939 Code. Moreover, under Section
relevance to the whole transaction which can be inferred from the 29(a) of said Code, dividends are included in "gross income". As
outcome thereof. It is the "net effect rather than the motives and plans income, it is subject to income tax which is required to be withheld at
of the taxpayer or his corporation". The test of taxability under the source.
exempting clause, when it provides "such time and manner" as would
make the redemption "essentially equivalent to the distribution of a
taxable dividend", is whether the redemption resulted into a flow of
wealth. If no wealth is realized from the redemption, there may not be
a dividend equivalence treatment.

• The test of taxability under the exempting clause of Section 83(b) is,
whether income was realized through the redemption of stock
dividends. The redemption converts into money the stock dividends
which become a realized profit or gain and consequently, the
stockholder's separate property. Profits derived from the capital
invested cannot escape income tax. As realized income, the proceeds
of the redeemed stock dividends can be reached by income taxation
regardless of the existence of any business purpose for the redemption.
Otherwise, to rule that the said proceeds are exempt from income tax
when the redemption is supported by legitimate business reasons
would defeat the very purpose of imposing tax on income.

• The issuance and the redemption of stocks are two different


transactions. Although the existence of legitimate corporate purposes
may justify a corporation's acquisition of its own shares under Section
41 of the Corporation Code, such purposes cannot excuse the
stockholder from the effects of taxation arising from the redemption.
G.R. No. 108576 January 20, 1999 50,495 of which are original issues and the balance of 134.659 shares
as stock dividend declarations. 13 Correspondingly, one-half of that
COMMISSIONER OF INTERNAL REVENUE, petitioner, shareholdings or 92,577 14 shares were transferred to his wife, Doña
vs. Carmen Soriano, as her conjugal share. The other half formed part of
THE COURT OF APPEALS, COURT OF TAX APPEALS and A. his estate. 15
SORIANO CORP., respondents.
A day after Don Andres died, ANSCOR increased its capital stock to
P20M 16 and in 1966 further increased it to P30M. 17 In the same year
(December 1966), stock dividends worth 46,290 and 46,287 shares
MARTINEZ, J.: were respectively received by the Don Andres estate 18 and Doña
Carmen from ANSCOR. Hence, increasing their accumulated
Petitioner Commissioner of Internal Revenue (CIR) seeks the reversal shareholdings to 138,867 and 138,864 19 common shares each. 20
of the decision of the Court of Appeals (CA) 1 which affirmed the ruling
of the Court of Tax Appeals (CTA) 2 that private respondent A. Soriano On December 28, 1967, Doña Carmen requested a ruling from the
Corporation's (hereinafter ANSCOR) redemption and exchange of the United States Internal Revenue Service (IRS), inquiring if an exchange
stocks of its foreign stockholders cannot be considered as "essentially of common with preferred shares may be considered as a tax
equivalent to a distribution of taxable dividends" under, Section 83(b) avoidance scheme 21 under Section 367 of the 1954 U.S. Revenue
of the 1939 Internal Revenue Act. 3 Act. 22 By January 2, 1968, ANSCOR reclassified its existing 300,000
common shares into 150,000 common and 150,000 preferred
The undisputed facts are as follows: shares. 23

Sometime in the 1930s, Don Andres Soriano, a citizen and resident of In a letter-reply dated February 1968, the IRS opined that the exchange
the United States, formed the corporation "A. Soriano Y Cia", is only a recapitalization scheme and not tax
predecessor of ANSCOR, with a P1,000,000.00 capitalization divided avoidance. 24 Consequently, 25 on March 31, 1968 Doña Carmen
into 10,000 common shares at a par value of P100/share. ANSCOR is exchanged her whole 138,864 common shares for 138,860 of the newly
wholly owned and controlled by the family of Don Andres, who are all reclassified preferred shares. The estate of Don Andres in turn,
non-resident aliens. 4 In 1937, Don Andres subscribed to 4,963 shares exchanged 11,140 of its common shares, for the remaining 11,140
of the 5,000 shares originally issued. 5 preferred shares, thus reducing its (the estate) common shares to
127,727. 26
On September 12, 1945, ANSCOR's authorized capital stock was
increased to P2,500,000.00 divided into 25,000 common shares with On June 30, 1968, pursuant to a Board Resolution, ANSCOR
the same par value of the additional 15,000 shares, only 10,000 was redeemed 28,000 common shares from the Don Andres' estate. By
issued which were all subscribed by Don Andres, after the other November 1968, the Board further increased ANSCOR's capital stock
stockholders waived in favor of the former their pre-emptive rights to to P75M divided into 150,000 preferred shares and 600,000 common
subscribe to the new issues. 6 This increased his subscription to 14,963 shares. 27 About a year later, ANSCOR again redeemed 80,000
common shares. 7 A month later, 8 Don Andres transferred 1,250 common shares from the Don Andres' estate, 28 further reducing the
shares each to his two sons, Jose and Andres, Jr., as their initial latter's common shareholdings to 19,727. As stated in the Board
investments in ANSCOR. 9 Both sons are foreigners. 10 Resolutions, ANSCOR's business purpose for both redemptions of
stocks is to partially retire said stocks as treasury shares in order to
By 1947, ANSCOR declared stock dividends. Other stock dividend reduce the company's foreign exchange remittances in case cash
declarations were made between 1949 and December 20, 1963. 11 On dividends are declared. 29
December 30, 1964 Don Andres died. As of that date, the records
revealed that he has a total shareholdings of 185,154 shares 12 —
In 1973, after examining ANSCOR's books of account and records, Petitioner contends that the exchange transaction a tantamount to
Revenue examiners issued a report proposing that ANSCOR be "cancellation" under Section 83(b) making the proceeds thereof
assessed for deficiency withholding tax-at-source, pursuant to Sections taxable. It also argues that the Section applies to stock dividends which
53 and 54 of the 1939 Revenue Code, 30 for the year 1968 and the is the bulk of stocks that ANSCOR redeemed. Further, petitioner claims
second quarter of 1969 based on the transactions of exchange 31 and that under the "net effect test," the estate of Don Andres gained from
redemption of stocks. 31 The Bureau of Internal Revenue (BIR) made the redemption. Accordingly, it was the duty of ANSCOR to withhold
the corresponding assessments despite the claim of ANSCOR that it the tax-at-source arising from the two transactions, pursuant to Section
availed of the tax amnesty under Presidential Decree 53 and 54 of the 1939 Revenue Act. 39
(P.D.) 23 32 which were amended by P.D.'s 67 and 157. 33 However,
petitioner ruled that the invoked decrees do not cover Sections 53 and ANSCOR, however, avers that it has no duty to withhold any tax either
54 in relation to Article 83(b) of the 1939 Revenue Act under which from the Don Andres estate or from Doña Carmen based on the two
ANSCOR was assessed. 34 ANSCOR's subsequent protest on the transactions, because the same were done for legitimate business
assessments was denied in 1983 by petitioner. 35 purposes which are (a) to reduce its foreign exchange remittances in
the event the company would declare cash dividends, 40 and to (b)
Subsequently, ANSCOR filed a petition for review with the CTA subsequently "filipinized" ownership of ANSCOR, as allegedly,
assailing the tax assessments on the redemptions and exchange of envisioned by Don Andres. 41 It likewise invoked the amnesty
stocks. In its decision, the Tax Court reversed petitioner's ruling, after provisions of P.D. 67.
finding sufficient evidence to overcome the prima facie correctness of
the questioned assessments. 36 In a petition for review the CA as We must emphasize that the application of Sec. 83(b) depends on the
mentioned, affirmed the ruling of the CTA. 37 Hence, this petition. special factual circumstances of each case. 42 The findings of facts of
a special court (CTA) exercising particular expertise on the subject of
The bone of contention is the interpretation and application of Section tax, generally binds this Court, 43 considering that it is substantially
83(b) of the 1939 Revenue Act 38 which provides: similar to the findings of the CA which is the final arbiter of questions of
facts. 44 The issue in this case does not only deal with facts but whether
Sec. 83. Distribution of dividends or assets by corporations. — the law applies to a particular set of facts. Moreover, this Court is not
necessarily bound by the lower courts' conclusions of law drawn from
(b) Stock dividends — A stock dividend representing the transfer of such facts. 45
surplus to capital account shall not be subject to tax. However, if a
corporation cancels or redeems stock issued as a dividend at such time AMNESTY:
and in such manner as to make the distribution and cancellation or
redemption, in whole or in part, essentially equivalent to the distribution We will deal first with the issue of tax amnesty. Section 1 of P.D.
of a taxable dividend, the amount so distributed in redemption or 67 46 provides:
cancellation of the stock shall be considered as taxable income to the
extent it represents a distribution of earnings or profits accumulated 1. In all cases of voluntary disclosures of previously untaxed income
after March first, nineteen hundred and thirteen. (Emphasis supplied) and/or wealth such as earnings, receipts, gifts, bequests or any other
acquisitions from any source whatsoever which are taxable under the
Specifically, the issue is whether ANSCOR's redemption of stocks from National Internal Revenue Code, as amended, realized here or abroad
its stockholder as well as the exchange of common with preferred by any taxpayer, natural or judicial; the collection of all internal revenue
shares can be considered as "essentially equivalent to the distribution taxes including the increments or penalties or account of non-payment
of taxable dividend" making the proceeds thereof taxable under the as well as all civil, criminal or administrative liabilities arising from or
provisions of the above-quoted law. incident to such disclosures under the National Internal Revenue Code,
the Revised Penal Code, the Anti-Graft and Corrupt Practices Act, the
Revised Administrative Code, the Civil Service laws and regulations, Codal provisions on withholding tax are mandatory and must be
laws and regulations on Immigration and Deportation, or any other complied with by the withholding agent. 55 The taxpayer should not
applicable law or proclamation, are hereby condoned and, in lieu answer for the non-performance by the withholding agent of its legal
thereof, a tax of ten (10%) per centum on such previously untaxed duty to withhold unless there is collusion or bad faith. The former could
income or wealth, is hereby imposed, subject to the following not be deemed to have evaded the tax had the withholding agent
conditions: (conditions omitted) [Emphasis supplied]. performed its duty. This could be the situation for which the amnesty
decree was intended. Thus, to curtail tax evasion and give tax evaders
The decree condones "the collection of all internal revenue taxes a chance to reform, 56 it was deemed administratively feasible to grant
including the increments or penalties or account of non-payment as well tax amnesty in certain instances. In addition, a "tax amnesty, much like
as all civil, criminal or administrative liable arising from or incident to" a tax exemption, is never favored nor presumed in law and if granted
(voluntary) disclosures under the NIRC of previously untaxed income by a statute, the term of the amnesty like that of a tax exemption must
and/or wealth "realized here or abroad by any taxpayer, natural or be construed strictly against the taxpayer and liberally in favor of the
juridical." taxing authority.57 The rule on strictissimi juris equally applies. 58 So
that, any doubt in the application of an amnesty law/decree should be
May the withholding agent, in such capacity, be deemed a taxpayer for resolved in favor of the taxing authority.
it to avail of the amnesty? An income taxpayer covers all persons who
derive taxable income. 47 ANSCOR was assessed by petitioner for Furthermore, ANSCOR's claim of amnesty cannot prosper. The
deficiency withholding tax under Section 53 and 54 of the 1939 Code. implementing rules of P.D. 370 which expanded amnesty on previously
As such, it is being held liable in its capacity as a withholding agent and untaxed income under P.D. 23 is very explicit, to wit:
not its personality as a taxpayer.
Sec. 4. Cases not covered by amnesty. — The following cases are not
In the operation of the withholding tax system, the withholding agent is covered by the amnesty subject of these regulations:
the payor, a separate entity acting no more than an agent of the
government for the collection of the tax 48 in order to ensure its xxx xxx xxx
payments; 49 the payer is the taxpayer — he is the person subject to
tax impose by law; 50 and the payee is the taxing authority. 51 In other (2) Tax liabilities with or without assessments, on withholding tax at
words, the withholding agent is merely a tax collector, not a taxpayer. source provided under Section 53 and 54 of the National Internal
Under the withholding system, however, the agent-payor becomes a Revenue Code, as amended; 59
payee by fiction of law. His (agent) liability is direct and independent
from the taxpayer, 52 because the income tax is still impose on and due ANSCOR was assessed under Sections 53 and 54 of the 1939 Tax
from the latter. The agent is not liable for the tax as no wealth flowed Code. Thus, by specific provision of law, it is not covered by the
into him — he earned no income. The Tax Code only makes the agent amnesty.
personally liable for the tax 53 arising from the breach of its legal duty to
withhold as distinguish from its duty to pay tax since: TAX ON STOCK DIVIDENDS

the government's cause of action against the withholding is not for the General Rule
collection of income tax, but for the enforcement of the withholding
provision of Section 53 of the Tax Code, compliance with which is Sec. 83(b) of the 1939 NIRC was taken from the Section 115(g)(1) of
imposed on the withholding agent and not upon the taxpayer. 54 the U.S. Revenue Code of 1928. 60 It laid down the general rule known
as the proportionate test 61 wherein stock dividends once issued form
Not being a taxpayer, a withholding agent, like ANSCOR in this part of the capital and, thus, subject to income tax.62 Specifically, the
transaction is not protected by the amnesty under the decree. general rule states that:
A stock dividend representing the transfer of surplus to capital account income), the exempting clause above quoted was added because
shall not be subject to tax. provision corporation found a loophole in the original provision. They
resorted to devious means to circumvent the law and evade the tax.
Having been derived from a foreign law, resort to the jurisprudence of Corporate earnings would be distributed under the guise of its initial
its origin may shed light. Under the US Revenue Code, this provision capitalization by declaring the stock dividends previously issued and
originally referred to "stock dividends" only, without any exception. later redeem said dividends by paying cash to the stockholder. This
Stock dividends, strictly speaking, represent capital and do not process of issuance-redemption amounts to a distribution of taxable
constitute income to its cash dividends which was lust delayed so as to escape the tax. It
recipient. 63 So that the mere issuance thereof is not yet subject to becomes a convenient technical strategy to avoid the effects of
income tax 64 as they are nothing but an "enrichment through increase taxation.
in value of capital
investment." 65 As capital, the stock dividends postpone the realization Thus, to plug the loophole — the exempting clause was added. It
of profits because the "fund represented by the new stock has been provides that the redemption or cancellation of stock dividends,
transferred from surplus to capital and no longer available for actual depending on the "time" and "manner" it was made, is essentially
distribution." 66 Income in tax law is "an amount of money coming to a equivalent to a distribution of taxable dividends," making the proceeds
person within a specified time, whether as payment for services, thereof "taxable income" "to the extent it represents profits". The
interest, or profit from investment." 67 It means cash or its exception was designed to prevent the issuance and cancellation or
equivalent. 68 It is gain derived and severed from capital, 69 from labor redemption of stock dividends, which is fundamentally not taxable, from
or from both combined 70 — so that to tax a stock dividend would be to being made use of as a device for the actual distribution of cash
tax a capital increase rather than the income. 71 In a loose sense, stock dividends, which is taxable. 76 Thus,
dividends issued by the corporation, are considered unrealized gain,
and cannot be subjected to income tax until that gain has been realized. the provision had the obvious purpose of preventing a corporation from
Before the realization, stock dividends are nothing but a representation avoiding dividend tax treatment by distributing earnings to its
of an interest in the corporate properties. 72 As capital, it is not yet shareholders in two transactions — a pro rata stock dividend followed
subject to income tax. It should be noted that capital and income are by a pro rata redemption — that would have the same economic
different. Capital is wealth or fund; whereas income is profit or gain or consequences as a simple dividend. 77
the flow of wealth. 73 The determining factor for the imposition of income
tax is whether any gain or profit was derived from a transaction. 74 Although redemption and cancellation are generally considered capital
transactions, as such. they are not subject to tax. However, it does not
The Exception necessarily mean that a shareholder may not realize a taxable gain
from such transactions. 78 Simply put, depending on the circumstances,
However, if a corporation cancels or redeems stock issued as the proceeds of redemption of stock dividends are essentially
a dividend at such time and in such manner as to make the distribution distribution of cash dividends, which when paid becomes the absolute
and cancellation or redemption, in whole or in part, essentially property of the stockholder. Thereafter, the latter becomes the
equivalent to the distribution of a taxable dividend, the amount so exclusive owner thereof and can exercise the freedom of
distributed in redemption or cancellation of the stock shall be choice. 79 Having realized gain from that redemption, the income earner
considered as taxable income to the extent it represents a distribution cannot escape income tax. 80
of earnings or profits accumulated after March first, nineteen hundred
and thirteen. (Emphasis supplied). As qualified by the phrase "such time and in such manner," the
exception was not intended to characterize as taxable dividend every
In a response to the ruling of the American Supreme Court in the case distribution of earnings arising from the redemption of stock
of Eisner v. Macomber 75 (that pro rata stock dividends are not taxable dividend. 81 So that, whether the amount distributed in the redemption
should be treated as the equivalent of a "taxable dividend" is a question redeemed come from? If its source is the original capital subscriptions
of fact, 82 which is determinable on "the basis of the particular facts of upon establishment of the corporation or from initial capital investment
the transaction in question. 83 No decisive test can be used to in an existing enterprise, its redemption to the concurrent value of
determine the application of the exemption under Section 83(b). The acquisition may not invite the application of Sec. 83(b) under the 1939
use of the words "such manner" and "essentially equivalent" negative Tax Code, as it is not income but a mere return of capital. On the
any idea that a weighted formula can resolve a crucial issue — Should contrary, if the redeemed shares are from stock dividend declarations
the distribution be treated as taxable dividend. 84 On this aspect, other than as initial capital investment, the proceeds of the redemption
American courts developed certain recognized criteria, which includes is additional wealth, for it is not merely a return of capital but a gain
the following: 85 thereon.

1) the presence or absence of real business purpose, It is not the stock dividends but the proceeds of its redemption that may
be deemed as taxable dividends. Here, it is undisputed that at the time
2) the amount of earnings and profits available for the declaration of a of the last redemption, the original common shares owned by the estate
regular dividends and the corporation's past record with respect to the were only 25,247.5 91 This means that from the total of 108,000 shares
declaration of dividends, redeemed from the estate, the balance of 82,752.5 (108,000 less
25,247.5) must have come from stock dividends. Besides, in the
3) the effect of the distribution, as compared with the declaration of absence of evidence to the contrary, the Tax Code presumes that every
regular dividend, distribution of corporate property, in whole or in part, is made out of
corporate profits 92 such as stock dividends. The capital cannot be
4) the lapse of time between issuance and redemption, 86 distributed in the form of redemption of stock dividends without violating
the trust fund doctrine — wherein the capital stock, property and other
5) the presence of a substantial surplus 87 and a generous supply of assets of the corporation are regarded as equity in trust for the payment
cash which invites suspicion as does a meager policy in relation both of the corporate creditors. 93 Once capital, it is always capital. 94 That
to current earnings and accumulated surplus, 88 doctrine was intended for the protection of corporate creditors. 95

REDEMPTION AND CANCELLATION With respect to the third requisite, ANSCOR redeemed stock dividends
issued just 2 to 3 years earlier. The time alone that lapsed from the
For the exempting clause of Section, 83(b) to apply, it is indispensable issuance to the redemption is not a sufficient indicator to determine
that: (a) there is redemption or cancellation; (b) the transaction involves taxability. It is a must to consider the factual circumstances as to the
stock dividends and (c) the "time and manner" of the transaction makes manner of both the issuance and the redemption. The "time" element
it "essentially equivalent to a distribution of taxable dividends." Of these, is a factor to show a device to evade tax and the scheme of cancelling
the most important is the third. or redeeming the same shares is a method usually adopted to
accomplish the end sought. 96 Was this transaction used as a
Redemption is repurchase, a reacquisition of stock by a corporation "continuing plan," "device" or "artifice" to evade payment of tax? It is
which issued the stock 89 in exchange for property, whether or not the necessary to determine the "net effect" of the transaction between the
acquired stock is cancelled, retired or held in the shareholder-income taxpayer and the acquiring (redeeming)
treasury. 90 Essentially, the corporation gets back some of its stock, corporation. 97 The "net effect" test is not evidence or testimony to be
distributes cash or property to the shareholder in payment for the stock, considered; it is rather an inference to be drawn or a conclusion to be
and continues in business as before. The redemption of stock dividends reached. 98 It is also important to know whether the issuance of stock
previously issued is used as a veil for the constructive distribution of dividends was dictated by legitimate business reasons, the presence of
cash dividends. In the instant case, there is no dispute that ANSCOR which might negate a tax evasion plan. 99
redeemed shares of stocks from a stockholder (Don Andres) twice
(28,000 and 80,000 common shares). But where did the shares
The issuance of stock dividends and its subsequent redemption must The three elements in the imposition of income tax are: (1) there must
be separate, distinct, and not related, for the redemption to be be gain or and profit, (2) that the gain or profit is realized or received,
considered a legitimate tax scheme. 100 Redemption cannot be used as actually or constructively, 108 and (3) it is not exempted by law or treaty
a cloak to distribute corporate earnings. 101 Otherwise, the apparent from income tax. Any business purpose as to why or how the income
intention to avoid tax becomes doubtful as the intention to evade was earned by the taxpayer is not a requirement. Income tax is
becomes manifest. It has been ruled that: assessed on income received from any property, activity or service that
produces the income because the Tax Code stands as an indifferent
[A]n operation with no business or corporate purpose — is a mere neutral party on the matter of where income comes
devise which put on the form of a corporate reorganization as a disguise from. 109
for concealing its real character, and the sole object and
accomplishment of which was the consummation of a preconceived As stated above, the test of taxability under the exempting clause of
plan, not to reorganize a business or any part of a business, but to Section 83(b) is, whether income was realized through the redemption
transfer a parcel of corporate shares to a stockholder. 102 of stock dividends. The redemption converts into money the stock
dividends which become a realized profit or gain and consequently, the
Depending on each case, the exempting provision of Sec. 83(b) of the stockholder's separate property. 110 Profits derived from the capital
1939 Code may not be applicable if the redeemed shares were issued invested cannot escape income tax. As realized income, the proceeds
with bona fide business purpose, 103 which is judged after each and of the redeemed stock dividends can be reached by income taxation
every step of the transaction have been considered and the whole regardless of the existence of any business purpose for the redemption.
transaction does not amount to a tax evasion scheme. Otherwise, to rule that the said proceeds are exempt from income tax
when the redemption is supported by legitimate business reasons
ANSCOR invoked two reasons to justify the redemptions — (1) the would defeat the very purpose of imposing tax on income. Such
alleged "filipinization" program and (2) the reduction of foreign argument would open the door for income earners not to pay tax so
exchange remittances in case cash dividends are declared. The Court long as the person from whom the income was derived has legitimate
is not concerned with the wisdom of these purposes but on their business reasons. In other words, the payment of tax under the
relevance to the whole transaction which can be inferred from the exempting clause of Section 83(b) would be made to depend not on the
outcome thereof. Again, it is the "net effect rather than the motives and income of the taxpayer, but on the business purposes of a third party
plans of the taxpayer or his corporation" 104 that is the fundamental (the corporation herein) from whom the income was earned. This is
guide in administering Sec. 83(b). This tax provision is aimed at the absurd, illogical and impractical considering that the Bureau of Internal
result. 105 It also applies even if at the time of the issuance of the stock Revenue (BIR) would be pestered with instances in determining the
dividend, there was no intention to redeem it as a means of distributing legitimacy of business reasons that every income earner may
profit or avoiding tax on dividends. 106 The existence of legitimate interposed. It is not administratively feasible and cannot therefore be
business purposes in support of the redemption of stock dividends is allowed.
immaterial in income taxation. It has no relevance in determining
"dividend equivalence". 107 Such purposes may be material only upon The ruling in the American cases cited and relied upon by ANSCOR
the issuance of the stock dividends. The test of taxability under the that "the redeemed shares are the equivalent of dividend only if the
exempting clause, when it provides "such time and manner" as would shares were not issued for genuine business purposes", 111 or the
make the redemption "essentially equivalent to the distribution of a "redeemed shares have been issued by a corporation bona
taxable dividend", is whether the redemption resulted into a flow of fide" 112 bears no relevance in determining the non-taxability of the
wealth. If no wealth is realized from the redemption, there may not be proceeds of redemption ANSCOR, relying heavily and applying said
a dividend equivalence treatment. In the metaphor of Eisner v. cases, argued that so long as the redemption is supported by valid
Macomber, income is not deemed "realize" until the fruit has fallen or corporate purposes the proceeds are not subject to tax. 113 The
been plucked from the tree. adoption by the courts below 114 of such argument is misleading if not
misplaced. A review of the cited American cases shows that the Furthermore, even if the said purposes support the redemption and
presence or absence of "genuine business purposes" may be material justify the issuance of stock dividends, the same has no bearing
with respect to the issuance or declaration of stock dividends but not whatsoever on the imposition of the tax herein assessed because the
on its subsequent redemption. The issuance and the redemption of proceeds of the redemption are deemed taxable dividends since it was
stocks are two different transactions. Although the existence of shown that income was generated therefrom.
legitimate corporate purposes may justify a corporation's acquisition of
its own shares under Section 41 of the Corporation Code, 115 such Thirdly, ANSCOR argued that to treat as "taxable dividend" the
purposes cannot excuse the stockholder from the effects of taxation proceeds of the redeemed stock dividends would be to impose on such
arising from the redemption. If the issuance of stock dividends is part stock an undisclosed lien and would be extremely unfair to intervening
of a tax evasion plan and thus, without legitimate business reasons, the purchase, i.e. those who buys the stock dividends after their
redemption becomes suspicious which exempting clause. The issuance. 118 Such argument, however, bears no relevance in this case
substance of the whole transaction, not its form, usually controls the tax as no intervening buyer is involved. And even if there is an intervening
consequences. 116 buyer, it is necessary to look into the factual milieu of the case if income
was realized from the transaction. Again, we reiterate that the dividend
The two purposes invoked by ANSCOR, under the facts of this case equivalence test depends on such "time and manner" of the transaction
are no excuse for its tax liability. First, the alleged "filipinization" plan and its net effect. The undisclosed lien 119 may be unfair to a
cannot be considered legitimate as it was not implemented until the BIR subsequent stock buyer who has no capital interest in the company.
started making assessments on the proceeds of the redemption. Such But the unfairness may not be true to an original subscriber like Don
corporate plan was not stated in nor supported by any Board Resolution Andres, who holds stock dividends as gains from his investments. The
but a mere afterthought interposed by the counsel of ANSCOR. Being subsequent buyer who buys stock dividends is investing capital. It just
a separate entity, the corporation can act only through its Board of so happen that what he bought is stock dividends. The effect of its
Directors. 117 The Board Resolutions authorizing the redemptions state (stock dividends) redemption from that subsequent buyer is merely to
only one purpose — reduction of foreign exchange remittances in case return his capital subscription, which is income if redeemed from the
cash dividends are declared. Not even this purpose can be given original subscriber.
credence. Records show that despite the existence of enormous
corporate profits no cash dividend was ever declared by ANSCOR from After considering the manner and the circumstances by which the
1945 until the BIR started making assessments in the early 1970's. issuance and redemption of stock dividends were made, there is no
Although a corporation under certain exceptions, has the prerogative other conclusion but that the proceeds thereof are essentially
when to issue dividends, yet when no cash dividends was issued for considered equivalent to a distribution of taxable dividends. As "taxable
about three decades, this circumstance negates the legitimacy of dividend" under Section 83(b), it is part of the "entire income" subject
ANSCOR's alleged purposes. Moreover, to issue stock dividends is to to tax under Section 22 in relation to Section 21 120 of the 1939 Code.
increase the shareholdings of ANSCOR's foreign stockholders contrary Moreover, under Section 29(a) of said Code, dividends are included in
to its "filipinization" plan. This would also increase rather than reduce "gross income". As income, it is subject to income tax which is required
their need for foreign exchange remittances in case of cash dividend to be withheld at source. The 1997 Tax Code may have altered the
declaration, considering that ANSCOR is a family corporation where situation but it does not change this disposition.
the majority shares at the time of redemptions were held by Don
Andres' foreign heirs. EXCHANGE OF COMMON WITH PREFERRED SHARES 121

Secondly, assuming arguendo, that those business purposes are Exchange is an act of taking or giving one thing for another
legitimate, the same cannot be a valid excuse for the imposition of tax. involving 122 reciprocal transfer 123 and is generally considered as a
Otherwise, the taxpayer's liability to pay income tax would be made to taxable transaction. The exchange of common stocks with preferred
depend upon a third person who did not earn the income being taxed. stocks, or preferred for common or a combination of either for both, may
not produce a recognized gain or loss, so long as the provisions of In this case, the exchange of shares, without more, produces no
Section 83(b) is not applicable. This is true in a trade between two (2) realized income to the subscriber. There is only a modification of the
persons as well as a trade between a stockholder and a corporation. In subscriber's rights and privileges — which is not a flow of wealth for tax
general, this trade must be parts of merger, transfer to controlled purposes. The issue of taxable dividend may arise only once a
corporation, corporate acquisitions or corporate reorganizations. No subscriber disposes of his entire interest and not when there is still
taxable gain or loss may be recognized on exchange of property, stock maintenance of proprietary interest. 130
or securities related to reorganizations. 124
WHEREFORE, premises considered, the decision of the Court of
Both the Tax Court and the Court of Appeals found that ANSCOR Appeals is MODIFIED in that ANSCOR's redemption of 82,752.5 stock
reclassified its shares into common and preferred, and that parts of the dividends is herein considered as essentially equivalent to a distribution
common shares of the Don Andres estate and all of Doña Carmen's of taxable dividends for which it is LIABLE for the withholding tax-at-
shares were exchanged for the whole 150.000 preferred shares. source. The decision is AFFIRMED in all other respects.
Thereafter, both the Don Andres estate and Doña Carmen remained
as corporate subscribers except that their subscriptions now include SO ORDERED.
preferred shares. There was no change in their proportional interest
after the exchange. There was no cash flow. Both stocks had the same
par value. Under the facts herein, any difference in their market value
would be immaterial at the time of exchange because no income is yet
realized — it was a mere corporate paper transaction. It would have
been different, if the exchange transaction resulted into a flow of wealth,
in which case income tax may be imposed. 125

Reclassification of shares does not always bring any substantial


alteration in the subscriber's proportional interest. But the exchange is
different — there would be a shifting of the balance of stock features,
like priority in dividend declarations or absence of voting rights. Yet
neither the reclassification nor exchange per se, yields realize income
for tax purposes. A common stock represents the residual ownership
interest in the corporation. It is a basic class of stock ordinarily and
usually issued without extraordinary rights or privileges and entitles the
shareholder to a pro rata division of profits. 126 Preferred stocks are
those which entitle the shareholder to some priority on dividends and
asset distribution. 127

Both shares are part of the corporation's capital stock. Both


stockholders are no different from ordinary investors who take on the
same investment risks. Preferred and common shareholders
participate in the same venture, willing to share in the profits and losses
of the enterprise. 128 Moreover, under the doctrine of equality of shares
— all stocks issued by the corporation are presumed equal with the
same privileges and liabilities, provided that the Articles of
Incorporation is silent on such differences. 129
The Deed of

Bañas Jr. v. Court of Appeals Sale provided that upon the signing of the contract AYALA shall pay

[G.R. No. 102967. February 10, 2000] P461,754.00

FACTS The balance of

Petitioner entered into a deed of sale purportedly on installment. He (P1,847,016.00... was to be paid in four equal... consecutive annual
discounted the promissory note covering the future installments for installments, with
purposes of taxation.
12%... interest per annum on the outstanding balance. AYALA issued
one promissory note covering four equal annual installments. Each
periodic payment of P461,754.00 pesos shall be payable starting on
ISSUE February 20, 1977, and... every year thereafter, or until February 20,
1980
Whether or not the promissory note should be declared cash
transaction for purposes of taxation. The same day, petitioner discounted the promissory note with AYALA,
for its face value of P1,847,016.00, evidenced by a Deed of Assignment
signed by the petitioner and AYALA. AYALA issued nine (9) checks to
petitioner, all dated February 20, 1976, drawn against Bank of the
RULING
Philippine Islands with the uniform amount of
YES. A negotiable instrument is deemed a substitute for money and for
value. According to Sec. 25 of NIL: “value is any consideration sufficient P205,224.00
to support a simple contract. An antecedent or pre-existing debt
constitutes value; and is deemed such whether the instrument is In his 1976 Income Tax Return, petitioner reported the P461,754 initial
payable on demand or at a future time”. Although the proceed of a payment as income from disposition of capital asset.
discounted promissory note is not considered part of the initial
payment, it is still taxable income for the year it was converted into In the succeeding years, until 1979, petitioner reported a uniform
cash. income of

P230,877.00... as gain from sale of capital asset. In his 1980 income


tax amnesty return, petitioner also reported... the same amount of
BIBIANO V. BAÑAS v. CA, GR No. 102967, 2000-02-10 P230,877.00 as the realized gain on disposition of capital asset for the
year.
Facts:
On April 11, 1978, then Revenue Director Mauro Calaguio authorized
On February 20, 1976, petitioner, Bibiano V. Bañas Jr. sold to AYALA, tax examiners, Rodolfo Tuazon and Procopio Talon to examine the
128,265 square meters of land located at Bayanan, Muntinlupa, for books and records of petitioner for the year 1976. They discovered that
petitioner had no outstanding receivable from the 1976 land sale to
P2,308,770.00 AYALA... and concluded that the sale was cash and the entire profit
should have been taxable in 1976 since the income was wholly derived On November 2, 1981, petitioner again filed an Amnesty Tax Return
in 1976. under P.D. 1840 and paid an... additional amount of P1,525.62

Tuazon and Talon filed their audit report and declared a discrepancy of In both, petitioner did not recognize that his sale of land to AYALA was
on cash basis.
P2,095,915.00... in petitioner's 1976 net income. They recommended
deficiency tax assessment for P2,473,673.00 Reacting to the complaint for tax evasion and the news reports,
petitioner filed with the RTC... an action[6] for damages against
Meantime, Aquilino Larin succeeded Calaguio as Regional Director of respondents Larin, Tuazon and Talon for extortion and malicious
Manila Region IV-A. After reviewing the examiners' report, Larin publication of the BIR's tax audit report. He claimed... that the filing of
directed the revision of the audit report, with instruction to consider the criminal complaints against him for violation of tax laws were improper
land as capital asset. The tax due was only because he had already availed of two tax amnesty decrees,
Presidential Decree Nos. 1740 and 1840.
50%... of the... total gain from sale of the property held by the taxpayer
beyond twelve months pursuant to Section 34[5] of the 1977 The trial court decided in favor of the respondents and awarded Larin
damages, as already stated. Petitioner seasonably appealed to the
NIRC Court of Appeals. In its decision of November 29, 1991, the respondent
court affirmed the trial court's decision
The deficiency tax assessment was reduced to
Issues:
P936,598.50, inclusive of surcharges and penalties for the year 1976.
Whether respondent court erred in finding that petitioner's income from
On June 27, 1980, respondent Larin sent a letter to petitioner informing the sale of land in 1976 should be declared as a cash transaction in his
him of the income tax deficiency that must be settled immediately. tax return for the same year

On September 26, 1980, petitioner acknowledged receipt of the letter Ruling:


but insisted that the sale of his land to AYALA was on installment.
As a general rule, the whole profit accruing from a sale of property is
On June 8, 1981, the matter was endorsed to the Acting Chief of the taxable as income in the year the sale is made. But, if not all of the sale
Legal Branch of the National Office of the BIR. The Chief of the Tax price is received during such year, and a statute provides that income
Fraud Unit recommended the prosecution of a criminal case for shall be taxable in the year in which it is "received," the... profit from an
conspiring to file false and fraudulent returns, in violation of Section 51 installment sale is to be apportioned between or among the years in
of... the Tax Code against petitioner and his accountants, Andres P. which such installments are paid and received.
Alejandre and Conrado Bañas.
Section 43 and Sec. 175 says that among the entities who may use the
On June 17, 1981, Larin filed a criminal complaint for tax evasion above-mentioned installment method is a seller of real property who
against the petitioner. disposes his property on installment, provided that the initial payment
does not exceed 25% of the selling price. They also state what may
On July 2, 1981, petitioner filed an Amnesty Tax Return under P. D. be... regarded as installment payment and what constitutes initial
1740 and paid the amount of payment. Initial payment means the payment received in cash or
property excluding evidences of indebtedness due and payable in
P41,729.81 subsequent years, like promissory notes or mortgages, given of the
purchaser during the... taxable year of sale. Initial payment does not
include amounts received by the vendor in the year of sale from the
disposition to a third person of notes given by the vendee as part of the
purchase price which are due and payable in subsequent years.[14]
Such disposition or discounting of receivable is material only as to the
computation of the initial payment. If the initial payment is within 25%
of total contract price, exclusive of the proceeds of discounted notes,
the sale qualifies as an installment sale,... otherwise it is a deferred
sale.

Although the proceed of a discounted promissory note is not


considered part of the initial payment, it is still taxable income for the
year it was converted into cash. The subsequent payments or
liquidation of certificates of indebtedness is reported using the
installment method... in computing the proportionate income[16] to be
returned, during the respective year it was realized. Non-dealer sales
of real or personal property may be reported as income under the
installment method provided that the obligation is still outstanding at...
the close of that year. If the seller disposes the entire installment
obligation by discounting the bill or the promissory note, he necessarily
must report the balance of the income from the discounting not only
income from the initial installment payment.

Where an installment obligation is discounted at a bank or finance


company, a taxable disposition results, even if the seller guarantees its
payment, continues to collect on the installment obligation, or handles
repossession of merchandise in case of default.

Although the proceeds of a discounted promissory note is not


considered... initial payment, still it must be included as taxable income
on the year it was converted to cash. When petitioner had the
promissory notes covering the succeeding installment payments of the
land issued by AYALA, discounted by AYALA itself, on the same day
of the sale, he lost... entitlement to report the sale as a sale on
installment since, a taxable disposition resulted and petitioner was
required by law to report in his returns the income derived from the
discounting. What petitioner did is tantamount to an attempt to
circumvent the rule on payment of... income taxes gained from the sale
of the land to AYALA for the year 1976.
G.R. No. 102967 February 10, 2000 Philippine Islands with the uniform amount of two hundred five
thousand, two hundred twenty-four (P205,224.00) pesos.
BIBIANO V. BAÑAS, JR., petitioner,
vs. In his 1976 Income Tax Return, petitioner reported the P461,754 initial
COURT OF APPEALS, AQUILINO T. LARIN, RODOLFO TUAZON payment as income from disposition of capital asset.2
AND PROCOPIO TALON, respondents.
Selling Price of Land P2,308,770.00
QUISUMBING, J.:
3
For review is the Decision of the Court of Appeals in CA-C.R. CV No. Less Initial Payment 461,754.00
17251 promulgated on November 29, 1991. It affirmed in toto the
judgment of the Regional Trial Court (RTC), Branch 39, Manila, in Civil
Case No. 82-12107. Said judgment disposed as follows:
Unrealized Gain P1,847,016.00
FOR ALL THE FOREGOING CONSIDERATIONS, this Court hereby
renders judgment DISMISSING the complaint against all the
defendants and ordering plaintiff [herein petitioner] to pay defendant
Larin the amount of P200,000.00 (Two Hundred Thousand Pesos) as
actual and compensatory damages; P200,000.00 as moral damages; 1976 Declaration of Income on Disposition of Capital Asset
and P50,000.00 as exemplary damages and attorneys fees of subject to Tax:
P100,000.00.1
Initial Payment P461,754.00
The facts, which we find supported by the records, have been
summarized by the Court of Appeals as follows:
Less: Cost of land and other incidental
On February 20, 1976, petitioner, Bibiano V. Bañas Jr. sold to Ayala ( 76,547.90)
Expenses
Investment Corporation (AYALA), 128,265 square meters of land
located at Bayanan, Muntinlupa, for two million, three hundred eight
thousand, seven hundred seventy (P2,308,770.00) pesos. The Deed of
Sale provided that upon the signing of the contract AYALA shall pay
four hundred sixty-one thousand, seven hundred fifty-four Income P385,206.10
(P461,754.00) pesos. The balance of one million, eight hundred forty-
seven thousand and sixteen (P1,847,016.00) pesos was to be paid in
four equal consecutive annual installments, with twelve (12%) percent
interest per annum on the outstanding balance. AYALA issued one Income subject to tax (P385,206. 10 x
P192,603.65
promissory note covering four equal annual installments. Each periodic 50%)
payment of P461,754.00 pesos shall be payable starting on February
20, 1977, and every year thereafter, or until February 20, 1980.
In the succeeding years, until 1979, petitioner reported a uniform
The same day, petitioner discounted the promissory note with AYALA, income of two hundred thirty thousand, eight hundred seventy-seven
for its face value of P1,847,016.00, evidenced by a Deed of Assignment (P230,877.00) pesos4 as gain from sale of capital asset. In his 1980
signed by the petitioner and AYALA. AYALA issued nine (9) checks to income tax amnesty return, petitioner also reported the same amount
petitioner, all dated February 20, 1976, drawn against Bank of the
of P230,877.00 as the realized gain on disposition of capital asset for On July 1, 1981, news items appeared in the now defunct Evening
the year. Express with the headline: "BIR Charges Realtor" and another in the
defunct Evening Post with a news item: "BIR raps Realtor, 2
On April 11, 1978, then Revenue Director Mauro Calaguio authorized accountants." Another news item also appeared in the July 2, 1981,
tax examiners, Rodolfo Tuazon and Procopio Talon to examine the issue of the Bulletin Today entitled: "3-face P1-M tax evasion raps." All
books and records of petitioner for the year 1976. They discovered that news items mentioned petitioner's false income tax return concerning
petitioner had no outstanding receivable from the 1976 land sale to the sale of land to AYALA.
AYALA and concluded that the sale was cash and the entire profit
should have been taxable in 1976 since the income was wholly derived On July 2, 1981, petitioner filed an Amnesty Tax Return under P.D.
in 1976. 1740 and paid the amount of forty-one thousand, seven hundred
twenty-nine pesos and eighty-one centavos (P41,729.81). On
Tuazon and Talon filed their audit report and declared a discrepancy of November 2, 1981, petitioner again filed an Amnesty Tax Return under
two million, ninety-five thousand, nine hundred fifteen (P2,095,915.00) P.D. 1840 and paid an additional amount of one thousand, five hundred
pesos in petitioner's 1976 net income. They recommended deficiency twenty-five pesos and sixty-two centavos (P1,525.62). In both,
tax assessment for two million, four hundred seventy-three thousand, petitioner did not recognize that his sale of land to AYALA was on cash
six hundred seventy-three (P2,473,673.00) pesos. basis.

Meantime, Aquilino Larin succeeded Calaguio as Regional Director of Reacting to the complaint for tax evasion and the news reports,
Manila Region IV-A. After reviewing the examiners' report, Larin petitioner filed with the RTC of Manila an action6 for damages against
directed the revision of the audit report, with instruction to consider the respondents Larin, Tuazon and Talon for extortion and malicious
land as capital asset. The tax due was only fifty (50%) percent of the publication of the BIR's tax audit report. He claimed that the filing of
total gain from sale of the property held by the taxpayer beyond twelve criminal complaints against him for violation of tax laws were improper
months pursuant to Section 345 of the 1977 National Internal Revenue because he had already availed of two tax amnesty decrees,
Code (NIRC). The deficiency tax assessment was reduced to nine Presidential Decree Nos. 1740 and 1840.
hundred thirty six thousand, five hundred ninety-eight pesos and fifty
centavos (P936,598.50), inclusive of surcharges and penalties for the The trial court decided in favor of the respondents and awarded Larin
year 1976. damages, as already stated. Petitioner seasonably appealed to the
Court of Appeals. In its decision of November 29, 1991, the respondent
On June 27, 1980, respondent Larin sent a letter to petitioner informing court affirmed the trial court's decision, thus:
of the income tax deficiency that must be settled him immediately.
The finding of the court a quo that plaintiff-appellant's actions against
On September 26, 1980, petitioner acknowledged receipt of the letter defendant-appellee Larin were unwarranted and baseless and as a
but insisted that the sale of his land to AYALA was on installment. result thereof, defendant-appellee Larin was subjected to unnecessary
anxiety and humiliation is therefore supported by the evidence on
On June 8, 1981, the matter was endorsed to the Acting Chief of the record.1âwphi1.nêt
Legal Branch of the National Office of the BIR. The Chief of the Tax
Fraud Unit recommended the prosecution of a criminal case for Defendant-appellee Larin acted only in pursuance of the authority
conspiring to file false and fraudulent returns, in violation of Section 51 granted to him. In fact, the criminal charges filed against him in the
of the Tax Code against petitioner and his accountants, Andres P. Tanodbayan and in the City Fiscal's Office were all dismissed.
Alejandre and Conrado Bañas.
WHEREFORE, the appealed judgment is hereby AFFIRMED in toto.7
On June 17, 1981, Larin filed a criminal complaint for tax evasion
against the petitioner.
Hence this petition, wherein petitioner raises before us the following The first issue, on whether the Court of Appeals erred in finding that
queries: there was no extortion, involves a determination of fact. The Court of
Appeals observed,
I. WHETHER THE COURT OF APPEALS ERRED IN ITS
INTERPRETATION OF PERTINENT TAX LAWS, THUS IT FAILED TO The only evidence to establish the alleged extortion attempt by
APPRECIATE THE CORRECTNESS AND ACCURACY OF defendants-appellees is the plaintiff-appellant's self serving
PETITIONER'S RETURN OF THE INCOME DERIVED FROM THE declarations.
SALE OF THE LAND TO AYALA.
As found by the court a quo, "said attempt was known to plaintiff-
II. WHETHER THE RESPONDENT COURT ERRED IN NOT FINDING appellant's son-in-law and counsel on record, yet, said counsel did not
THAT THERE WAS AN ALLEGED ATTEMPT TO EXTORT [MONEY take the witness stand to corroborate the testimony of plaintiff."8
FROM] PETITIONER BY PRIVATE RESPONDENTS.
As repeatedly held, findings of fact by the Court of Appeals especially
III. WHETHER THE RESPONDENT COURT ERRED IN ITS if they affirm factual findings of the trial court will not be disturbed by
INTERPRETATION OF PRESIDENTIAL DECREE NOS. 1740 AND this Court, unless these findings are not supported by
1840, AMONG OTHERS, PETITIONER'S IMMUNITY FROM evidence.9 Similarly, neither should we disturb a finding of the trial court
CRIMINAL PROSECUTION. and appellate court that an allegation is not supported by evidence on
record. Thus, we agree with the conclusion of respondent court that
IV. WHETHER THE RESPONDENT COURT ERRED IN ITS herein private respondents, on the basis of evidence, could not be held
INTERPRETATION OF WELL-ESTABLISHED DOCTRINES OF THIS liable for extortion.
HONORABLE COURT AS REGARDS THE AWARD OF ACTUAL,
MORAL AND EXEMPLARY DAMAGES IN FAVOR OF RESPONDENT On the second issue of whether P.D. Nos. 1740 and 1840 which
LARIN. granted tax amnesties also granted immunity from criminal prosecution
against tax offenses, the pertinent sections of these laws state:
In essence, petitioner asks the Court to resolve seriatim the following
issues: P.D. No. 1740. CONDONING PENALTIES FOR CERTAIN
VIOLATIONS OF THE INCOME TAX LAW UPON VOLUNTARY
1. Whether respondent court erred in ruling that there was no extortion DISCLOSURE OF UNDECLARED INCOME FOR INCOME TAX
attempt by BIR officials; PURPOSES AND REQUIRING PERIODIC SUBMISSION OF NET
WORTH STATEMENT.
2. Whether respondent court erred in holding that P.D. 1740 and 1840
granting tax amnesties did not grant immunity from tax suits; xxx xxx xxx

3. Whether respondent court erred in finding that petitioner's income Sec. 1. Voluntary Disclosure of Correct Taxable Income. — Any
from the sale of land in 1976 should be declared as a cash transaction individual who, for any or all of the taxable years 1974 to 1979, had
in his tax return for the same year (because the buyer discounted the failed to file a return is hereby, allowed to file a return for each of the
promissory note issued to the seller on future installment payments of aforesaid taxable years and accurately declare therein the true and
the sale, on the same day of the sale); correct income, deductions and exemptions and pay the income tax
due per return. Likewise, any individual who filed a false or fraudulent
4. Whether respondent court erred and committed grave abuse of return for any taxable year in the period mentioned above may amend
discretion in awarding damages to respondent Larin. his return and pay the correct amount of tax due after deducting the
taxes already paid, if any, in the original declaration. (emphasis ours)
xxx xxx xxx d) The taxpayer must file a statement of assets, liabilities and net worth
as of December 31, 1980, as required under Section 6 hereof.
Sec. 5. Immunity from Penalties. — Any individual who voluntarily files (emphasis ours)
a return under this Decree and pays the income tax due thereon shall
be immune from the penalties, civil or criminal, under the National It will be recalled that petitioner entered into a deed of sale purportedly
Internal Revenue Code arising from failure to pay the correct income on installment. On the same day, he discounted the promissory note
tax with respect to the taxable years from which an amended return covering the future installments. The discounting seems questionable
was filed or for which an original return was filed in cases where no because ordinarily, when a bill is discounted, the lender (e.g. banks,
return has been filed for any of the taxable years 1974 to financial institution) charges or deducts a certain percentage from the
1979: Provided, however, That these immunities shall not apply in principal value as its compensation. Here, the discounting was done by
cases where the amount of net taxable income declared under this the buyer. On July 2, 1981, two weeks after the filing of the tax evasion
Decree is understated to the extent of 25% or more of the correct net complaint against him by respondent Larin on June 17, 1981, petitioner
taxable income. (emphasis ours) availed of the tax amnesty under P.D. No. 1740. His amended tax
return for the years 1974 - 1979 was filed with the BIR office of
P.D. NO. 1840 — GRANTING A TAX AMNESTY ON UNTAXED Valenzuela, Bulacan, instead of Manila where the petitioner's principal
INCOME AND/OR WEALTH EARNED OR ACQUIRED DURING THE office was located. He again availed of the tax amnesty under P.D. No.
TAXABLE YEARS 1974 TO 1980 AND REQUIRING THE FILING OF 1840. His disclosure, however, did not include the income from his sale
THE STATEMENT OF ASSETS, LIABILITIES, AND NET WORTH. of land to AYALA on cash basis. Instead he insisted that such sale was
on installment. He did not amend his income tax return. He did not pay
Sec. 1. Coverage. — In case of voluntary disclosure of previously the tax which was considerably increased by the income derived from
untaxed income and/or wealth such as earnings, receipts, gifts, the discounting. He did not meet the twin requirements of P.D. 1740
bequests or any other acquisition from any source whatsoever, realized and 1840, declaration of his untaxed income and full payment of tax
here or abroad, by any individual taxpayer, which are taxable under the due thereon. Clearly, the petitioner is not entitled to the benefits of P.D.
National Internal Revenue Code, as amended, the assessment and Nos. 1740 and 1840. The mere filing of tax amnesty return under P.D.
collection of all internal revenue taxes, including the increments or 1740 and 1840 does not ipso facto shield him from immunity against
penalties on account of non-payment, as well as all civil, criminal or prosecution. Tax amnesty is a general pardon to taxpayers who want
administrative liabilities arising from or incident thereto under the to start a clean tax slate. It also gives the government a chance to
National Internal Revenue Code, are hereby condoned provided that collect uncollected tax from tax evaders without having to go through
the individual taxpayer shall pay. (emphasis ours) . . . the tedious process of a tax case. To avail of a tax amnesty granted by
the government, and to be immune from suit on its delinquencies, the
Sec. 2. Conditions for Immunity. — The immunity granted under tax payer must have voluntarily disclosed his previously untaxed
Section one of this Decree shall apply only under the following income and must have paid the corresponding tax on such previously
conditions: untaxed income.10

a) Such previously untaxed income and/or wealth must have been It also bears noting that a tax amnesty, much like a tax exemption, is
earned or realized in any of the years 1974 to 1980; never favored nor presumed in law and if granted by statute, the terms
of the amnesty like that of a tax exemption must be construed strictly
b) The taxpayer must file an amnesty return on or before November 30, against the taxpayer and liberally in favor of the taxing
1981, and fully pay the tax due thereon; authority.11 Hence, on this matter, it is our view that petitioner's claim of
immunity from prosecution under the shield of availing tax amnesty is
c) The amnesty tax paid by the taxpayer under this Decree shall not be untenable.
less than P1,000.00 per taxable year; and
On the third issue, petitioner asserts that his sale of the land to AYALA transfer of title, the vendor being protected by a mortgage or other lien
was not on cash basis but on installment as clearly specified in the as to deferred payments. Such sales either under (a) or (b), fall into two
Deed of Sale which states: classes when considered with respect to the terms of sale, as follows:

That for and in consideration of the sum of TWO MILLION THREE (1) Sales of property on the installment plan, that is, sales in which the
HUNDRED EIGHT THOUSAND SEVEN HUNDRED SEVENTY payments received in cash or property other than evidences of
(P2,308,770.00) PESOS Philippine Currency, to be paid as follows: indebtedness of the purchaser during the taxable year in which the sale
is made do not exceed 25 per cent of the selling price;
1. P461,754.00, upon the signing of the Deed of Sale; and,
(2) Deferred-payment sales not on the installment plan, that is sales in
2. The balance of P1,847,016.00, to be paid in four (4) equal, which the payments received in cash or property other than evidences
consecutive, annual installments with interest thereon at the rate of of indebtedness of the purchaser during the taxable year in which the
twelve percent (12%) per annum, beginning on February 20, 1976, said sale is made exceed 25 per cent of the selling price;
installments to be evidenced by four (4) negotiable promissory notes.12
In the sale of mortgaged property the amount of the mortgage, whether
Petitioner resorts to Section 43 of the NIRC and Sec. 175 of Revenue the property is merely taken subject to the mortgage or whether the
Regulation No. 2 to support his claim. mortgage is assumed by the purchaser, shall be included as a part of
the "selling price" but the amount of the mortgage, to the extent it does
Sec. 43 of the 1977 NIRC states, not exceed the basis to the vendor of the property sold, shall not be
considered as a part of the "initial payments" or of the "total contract
Installment basis. — (a) Dealers in personal property. — . . . price," as those terms are used in section 43 of the Code, in sections
174 and 176 of these regulations, and in this section. The term "initial
(b) Sales of realty and casual sales of personalty — In the case (1) of payments" does not include amounts received by the vendor in the year
a casual sale or other casual disposition of personal property (other of sale from the disposition to a third person of notes given by the
than property of a kind which would properly be included in the vendee as part of the purchase price which are due and payable in
inventory of the taxpayer if on hand at the close of the taxable year), for subsequent years. Commissions and other selling expenses paid or
a price exceeding one thousand pesos, or (2) of a sale or other incurred by the vendor are not to be deducted or taken into account in
disposition of real property if in either case the initial payments do not determining the amount of the "initial payments," the "total contract
exceed twenty-five percentum of the selling price, the income may, price," or the "selling price." The term "initial payments" contemplates
under regulations prescribed by the Minister of Finance, be returned on at least one other payment in addition to the initial payment. If the entire
the basis and in the manner above prescribed in this section. As used purchase price is to be paid in a lump sum in a later year, there being
in this section the term "initial payment" means the payments received no payment during the year, the income may not be returned on the
in cash or property other than evidences of indebtedness of the installment basis. Income may not be returned on the installment basis
purchaser during the taxable period in which the sale or other where no payment in cash or property, other than evidences of
disposition is made. . . . (emphasis ours) indebtedness of the purchaser, is received during the first year, the
purchaser having promised to make two or more payments, in later
Revenue Regulation No. 2, Section 175 provides, years.
Sale of real property involving deferred payments. — Under section 43 Petitioner asserts that Sec. 43 allows him to return as income in the
deferred-payment sales of real property include (1) agreements of taxable years involved, the respective installments as provided by the
purchase and sale which contemplate that a conveyance is not to be deed of sale between him and AYALA. Consequently, he religiously
made at the outset, but only after all or a substantial portion of the
selling price has been paid, and (b) sales in which there is an immediate
reported his yearly income from sale of capital asset, subject to tax, as Sec. 43 and Sec. 175 says that among the entities who may use the
follows: above-mentioned installment method is a seller of real property who
disposes his property on installment, provided that the initial payment
Year 1977 (50% of P461,754) P230,877.00 does not exceed 25% of the selling price. They also state what may be
regarded as installment payment and what constitutes initial payment.
Initial payment means the payment received in cash or property
1978 230,877.00 excluding evidences of indebtedness due and payable in subsequent
years, like promissory notes or mortgages, given of the purchaser
1979 230,877.00 during the taxable year of sale. Initial payment does not include
amounts received by the vendor in the year of sale from the disposition
to a third person of notes given by the vendee as part of the purchase
1980 230,877.00 price which are due and payable in subsequent years.14 Such
disposition or discounting of receivable is material only as to the
Petitioner says that his tax declarations are acceptable modes of computation of the initial payment. If the initial payment is within 25%
payment under Section 175 of the Revenue Regulations (RR) No. 2. of total contract price, exclusive of the proceeds of discounted notes,
The term "initial payment", he argues, does not include amounts the sale qualifies as an installment sale, otherwise it is a deferred sale.15
received by the vendor which are part of the complete purchase price,
still due and payable in subsequent years. Thus, the proceeds of the Although the proceed of a discounted promissory note is not
promissory notes, not yet due which he discounted to AYALA should considered part of the initial payment, it is still taxable income for the
not be included as income realized in 1976. Petitioner states that the year it was converted into cash. The subsequent payments or
original agreement in the Deed of Sale should not be affected by the liquidation of certificates of indebtedness is reported using the
subsequent discounting of the bill. installment method in computing the proportionate income16 to be
returned, during the respective year it was realized. Non-dealer sales
On the other hand, respondents assert that taxation is a matter of of real or personal property may be reported as income under the
substance and not of form. Returns are scrutinized to determine if installment method provided that the obligation is still outstanding at the
transactions are what they are and not declared to evade taxes. close of that year. If the seller disposes the entire installment obligation
Considering the progressive nature of our income taxation, when by discounting the bill or the promissory note, he necessarily must
income is spread over several installment payments through the years, report the balance of the income from the discounting not only income
the taxable income goes down and the tax due correspondingly from the initial installment payment.
decreases. When payment is in lump sum the tax for the year
proportionately increases. Ultimately, a declaration that a sale is on Where an installment obligation is discounted at a bank or finance
installment diminishes government taxes for the year of initial company, a taxable disposition results, even if the seller guarantees its
installment as against a declaration of cash sale where taxes to the payment, continues to collect on the installment obligation, or handles
government is larger. repossession of merchandise in case of default.17 This rule prevails in
the United States.18 Since our income tax laws are of American
As a general rule, the whole profit accruing from a sale of property is origin,19 interpretations by American courts an our parallel tax laws
taxable as income in the year the sale is made. But, if not all of the sale have persuasive effect on the interpretation of these laws.20 Thus, by
price is received during such year, and a statute provides that income analogy, all the more would a taxable disposition result when the
shall be taxable in the year in which it is "received," the profit from an discounting of the promissory note is done by the seller himself. Clearly,
installment sale is to be apportioned between or among the years in the indebtedness of the buyer is discharged, while the seller acquires
which such installments are paid and received.13 money for the settlement of his receivables. Logically then, the income
should be reported at the time of the actual gain. For income tax
purposes, income is an actual gain or an actual increase of Moral damages may be recovered in cases involving acts referred to in
wealth.21 Although the proceeds of a discounted promissory note is not Article 2127 of the Civil Code.28 As a rule, a public official may not
considered initial payment, still it must be included as taxable income recover damages for charges of falsehood related to his official conduct
on the year it was converted to cash. When petitioner had the unless he proves that the statement was made with actual malice.
promissory notes covering the succeeding installment payments of the In Babst, et. al. vs. National Intelligence Board, et. al., 132 SCRA 316,
land issued by AYALA, discounted by AYALA itself, on the same day 330 (1984), we reiterated the test for actual malice as set forth in the
of the sale, he lost entitlement to report the sale as a sale on installment landmark American case of New York Times vs. Sullivan,29 which we
since, a taxable disposition resulted and petitioner was required by law have long adopted, in defamation and libel cases, viz.:
to report in his returns the income derived from the discounting. What
petitioner did is tantamount to an attempt to circumvent the rule on . . . with knowledge that it was false or with reckless disregard of
payment of income taxes gained from the sale of the land to AYALA for whether it was false or not.
the year 1976.
We appreciate petitioner's claim that he filed his 1976 return in good
Lastly, petitioner questions the damages awarded to respondent Larin. faith and that he had honestly believed that the law allowed him to
declare the sale of the land, in installment. We can further grant that
Any person who seeks to be awarded actual or compensatory damages the pertinent tax laws needed construction, as we have earlier done.
due to acts of another has the burden of proving said damages as well That petitioner was offended by the headlines alluding to him as tax
as the amount thereof.22 Larin says the extortion cases filed against him evader is also fully understandable. All these, however, do not justify
hampered his immediate promotion, caused him strong anxiety and what amounted to a baseless prosecution of respondent Larin.
social humiliation. The trial court awarded him two hundred thousand Petitioner presented no evidence to prove Larin extorted money from
(P200,000,00) pesos as actual damages. However, the appellate court him. He even admitted that he never met nor talked to respondent Larin.
stated that, despite pendency of this case, Larin was given a promotion When the tax investigation against the petitioner started, Larin was not
at the BIR. Said respondent court: yet the Regional Director of BIR Region IV-A, Manila. On respondent
Larin's instruction, petitioner's tax assessment was considered one
We find nothing on record, aside from defendant-appellee Larin's involving a sale of capital asset, the income from which was subjected
statements (TSN, pp. 6-7, 11 December 1985), to show that he suffered to only fifty percent (50%) assessment, thus reducing the original tax
loss of seniority that allegedly barred his promotion. In fact, he was assessment by half. These circumstances may be taken to show that
promoted to his present position despite the pendency of the instant Larin's involvement in extortion was not indubitable. Yet, petitioner went
case (TSN, pp. 35-39, 04 November 1985).23 on to file the extortion cases against Larin in different fora. This is where
actual malice could attach on petitioner's part. Significantly, the trial
Moreover, the records of the case contain no statement whatsoever of court did not err in dismissing petitioner's complaints, a ruling affirmed
the amount of the actual damages sustained by the respondents. by the Court of Appeals.
Actual damages cannot be allowed unless supported by evidence on
the record.24 The court cannot rely on speculation, conjectures or Keeping all these in mind, we are constrained to agree that there is
guesswork as to the fact and amount of damages.25 To justify a grant sufficient basis for the award of moral and exemplary damages in favor
of actual or compensatory damages, it is necessary to prove with a of respondent Larin. The appellate court believed respondent Larin
reasonable degree of certainty, the actual amount of loss.26 Since we when he said he suffered anxiety and humiliation because of the
have no basis with which to assess, with certainty, the actual or unfounded charges against him. Petitioner's actions against Larin were
compensatory damages counter-claimed by respondent Larin, the found "unwarranted and baseless," and the criminal charges filed
award of such damages should be deleted. against him in the Tanodbayan and City Fiscal's Office were all
dismissed.30 Hence, there is adequate support for respondent court's
conclusion that moral damages have been proved.
Now, however, what would be a fair amount to be paid as compensation government officials. Thus, conformably with our declaration that moral
for moral damages also requires determination. Each case must be damages are not intended to enrich anyone,33 we hereby reduce the
governed by its own peculiar circumstances.31 On this score, Del moral damages award in this case from two hundred thousand
Rosario vs. Court of Appeals,32 cites several cases where no actual (P200,000.00) pesos to seventy five thousand (P75,000.00) pesos,
damages were adjudicated, and where moral and exemplary damages while the exemplary damage is set at P25,000.00 only.
were reduced for being "too excessive," thus:
The law allows the award of attorney's fees when exemplary damages
In the case of PNB v. C.A., [256 SCRA 309 (1996)], this Court quoted are awarded, and when the party to a suit was compelled to incur
with approval the following observation from RCPI v. Rodriguez, viz: expenses to protect his interest.34 Though government officers are
usually represented by the Solicitor General in cases connected with
** **. Nevertheless, we find the award of P100,000.00 as moral the performance of official functions, considering the nature of the
damages in favor of respondent Rodriguez excessive and charges, herein respondent Larin was compelled to hire a private
unconscionable. In the case of Prudenciado v. Alliance Transport lawyer for the conduct of his defense as well as the successful pursuit
System, Inc. (148 SCRA 440 [1987]) we said: . . . [I]t is undisputed that of his counterclaims. In our view, given the circumstances of this case,
the trial courts are given discretion to determine the amount of moral there is ample ground to award in his favor P50,000,00 as reasonable
damages (Alcantara v. Surro, 93 Phil. 472) and that the Court of attorney's fees.
Appeals can only modify or change the amount awarded when they are
palpably and scandalously excessive "so as to indicate that it was the WHEREFORE, the assailed decision of the Court of Appeals dated
result of passion, prejudice or corruption on the part of the trial court" November 29, 1991, is hereby AFFIRMED with MODIFICATION so that
(Gellada v. Warner Barnes & Co., Inc., 57 O.G. [4] 7347, 7358; Sadie the award of actual damages are deleted; and that petitioner is hereby
v. Bacharach Motors Co., Inc., 57 O.G. [4] 636 and Adone v. Bacharach ORDERED to pay to respondent Larin moral damages in the amount
Motor Co., Inc., 57 O.G. 656). But in more recent cases where the of P75,000.00, exemplary damages in the amount of P25,000.00, and
awards of moral and exemplary damages are far too excessive attorney's fees in the amount of P50,000.00 only.1âwphi1.nêt
compared to the actual loses sustained by the aggrieved party, this
Court ruled that they should be reduced to more reasonable amounts. No pronouncement as to costs.
. . . . (Emphasis ours.)
SO ORDERED.
In other words, the moral damages awarded must be commensurate
with the loss or injury suffered.

In the same case (PNB v. CA), this Court found the amount of
exemplary damages required to be paid (P1,000,000,00) "too
excessive" and reduced it to an "equitable level" (P25,000.00).

It will be noted that in above cases, the parties who were awarded moral
damages were not public officials. Considering that here, the award is
in favor of a government official in connection with his official function,
it is with caution that we affirm granting moral damages, for it might
open the floodgates for government officials counter-claiming damages
in suits filed against them in connection with their functions. Moreover,
we must be careful lest the amounts awarded make citizens hesitate to
expose corruption in the government, for fear of lawsuits from vindictive
SECOND DIVISION its corporate income tax return. As a consequence, respondent did not
pay income tax for 1995.
G.R. No. 148512 June 26, 2006
Subsequently, on December 27, 1996, claiming that according to Sec.
COMMISSIONER OF INTERNAL REVENUE, Petitioner, 4(a) of R.A. No. 7432, the amount of P219,778 should be applied as a
vs. tax credit, respondent filed a claim for refund in the amount
CENTRAL LUZON DRUG CORPORATION, Respondent. of P150,193, thus:

DECISION
Net Sales P 37,014,807.00
AZCUNA, J.:
Add: Cost of 20% Discount to Senior 219,778.00
This is a petition for review under Rule 45 of the Rules of Court seeking Citizens
the nullification of the Decision, dated May 31, 2001, of the Court of
Appeals (CA) in CA-G.R. SP No. 60057, entitled "Central Luzon Drug
Corporation v. Commissioner of Internal Revenue," granting herein
respondent Central Luzon Drug Corporation’s claim for tax credit equal Gross Sales P 37,234,585.00
to the amount of the 20% discount that it extended to senior citizens on
the latter’s purchase of medicines pursuant to Section 4(a) of Republic
Less: Cost of Sales
Act (R.A.) No. 7432, entitled "An Act to Maximize the Contribution of
Senior Citizens to Nation Building, Grant Benefits and Special
Privileges and for other Purposes" otherwise known as the Senior Merchandise
Citizens Act. P 1,232,740.00
Inventory, beg

The antecedents are as follows:


Purchases 41,145,138.00
Central Luzon Drug Corporation has been a retailer of medicines and
other pharmaceutical products since December 19, 1994. In 1995, it
opened three (3) drugstores as a franchisee under the business name 8,521,557.00 33,856,621.00
Merchandise
and style of "Mercury Drug." Inventory, end

For the period January 1995 to December 1995, in conformity to the


mandate of Sec. 4(a) of R.A. No. 7432, petitioner granted a 20% Gross Profit P 3,377,964.00
discount on the sale of medicines to qualified senior citizens amounting
to P219,778.
Miscellaneous Income 39,014.00
Pursuant to Revenue Regulations No. 2-941 implementing R.A. No.
7432, which states that the discount given to senior citizens shall be
deducted by the establishment from its gross sales for value-added tax
and other percentage tax purposes, respondent deducted the total
Total Income 3,416,978.00
amount of P219,778 from its gross income for the taxable year 1995.
For said taxable period, respondent reported a net loss of P20,963 in
Thus, respondent filed with the CA a Petition for Review on August 3,
Operating Expenses 3,199,230.00 2000.

On May 31, 2001, the CA rendered a Decision stating that Section 229
of the Tax Code does not apply in this case. It concluded that the 20%
discount given to senior citizens which is treated as a tax credit
Net Income Before Tax P 217,748.00 pursuant to Sec. 4(a) of R.A. No. 7432 is considered just compensation
and, as such, may be carried over to the next taxable period if there is
no current tax liability. In view of this, the CA held:
Income Tax (35%) 69,585.00
WHEREFORE, the instant petition is hereby GRANTED and the
Less: Tax Credit decision of the CTA dated 24 April 2000 and its resolution dated 06 July
2000 are SET ASIDE. A new one is entered granting petitioner’s claim
for tax credit in the amount of Php: 150,193.00. No costs.
(Cost of 20% Discount to Senior 219,778.00
Citizens) SO ORDERED.4

Hence, this petition raising the sole issue of whether the 20% sales
discount granted by respondent to qualified senior citizens pursuant to
Income Tax Payable (P 150,193.00) Sec. 4(a) of R.A. No. 7432 may be claimed as a tax credit or as a
deduction from gross sales in accordance with Sec. 2(1) of Revenue
Income Tax Actually Paid -0- Regulations No. 2-94.

Sec. 4(a) of R.A. No. 7432 provides:


Tax Refundable/Overpaid Income Tax (P 150,193.00)
Sec. 4. Privileges for the Senior citizens. – The senior citizens shall be
entitled to the following:
As shown above, the amount of P150,193 claimed as a refund
represents the tax credit allegedly due to respondent under R.A. No.
(a) the grant of twenty percent (20%) discount from all establishments
7432. Since the Commissioner of Internal Revenue "was not able to
relative to utilization of transportations services, hotels and similar
decide the claim for refund on time,"2 respondent filed a Petition for
lodging establishments, restaurants and recreation centers and
Review with the Court of Tax Appeals (CTA) on March 18, 1998.
purchase of medicines anywhere in the country: Provided, That private
establishments may claim the cost as tax credit.
On April 24, 2000, the CTA dismissed the petition, declaring that even
if the law treats the 20% sales discounts granted to senior citizens as a
The CA and the CTA correctly ruled that based on the plain wording of
tax credit, the same cannot apply when there is no tax liability or the
the law discounts given under R.A. No. 7432 should be treated as tax
amount of the tax credit is greater than the tax due. In the latter case,
credits, not deductions from income.
the tax credit will only be to the extent of the tax liability.3 Also, no refund
can be granted as no tax was erroneously, illegally and actually It is a fundamental rule in statutory construction that the legislative
collected based on the provisions of Section 230, now Section 229, of intent must be determined from the language of the statute itself
the Tax Code. Furthermore, the law does not state that a refund can be especially when the words and phrases therein are clear and
claimed by the private establishment concerned as an alternative to the unequivocal. The statute in such a case must be taken to mean exactly
tax credit.
what it says.5 Its literal meaning should be followed;6 to depart from the It must also be stressed that unlike in Sec. 229 of the Tax Code wherein
meaning expressed by the words is to alter the statute.7 the remedy of refund is available to the taxpayer, Sec. 4 of the law
speaks only of a tax credit, not a refund.
The above provision explicitly employed the word "tax credit." Nothing
in the provision suggests for it to mean a "deduction" from gross sales. As earlier mentioned, the tax credit benefit granted to the
To construe it otherwise would be a departure from the clear mandate establishments can be deemed as their just compensation for private
of the law. property taken by the State for public use. The privilege enjoyed by the
senior citizens does not come directly from the State, but rather from
Thus, the 20% discount required by the Act to be given to senior the private establishments concerned.12
citizens is a tax credit, not a deduction from the gross sales of the
establishment concerned. As a corollary to this, the definition of ‘tax WHEREFORE, the petition is DENIED. The Decision of the Court of
credit’ found in Section 2(1) of Revenue Regulations No. 2-94 is Appeals in CA-G.R. SP No. 60057, dated May 31, 2001, is AFFIRMED.
erroneous as it refers to tax credit as the amount representing the 20%
discount that "shall be deducted by the said establishment from their
gross sales for value added tax and other percentage tax purposes."
This definition is contrary to what our lawmakers had envisioned with
regard to the treatment of the discount granted to senior citizens.

Accordingly, when the law says that the cost of the discount may be
claimed as a tax credit, it means that the amount -- when claimed –
shall be treated as a reduction from any tax liability.8 The law cannot be
amended by a mere regulation. The administrative agencies issuing
these regulations may not enlarge, alter or restrict the provisions of the
law they administer.9 In fact, a regulation that "operates to create a rule
out of harmony with the statute is a mere nullity."10

Finally, for purposes of clarity, Sec. 22911 of the Tax Code does not
apply to cases that fall under Sec. 4 of R.A. No. 7432 because the
former provision governs exclusively all kinds of refund or credit of
internal revenue taxes that were erroneously or illegally imposed and
collected pursuant to the Tax Code while the latter extends the tax
credit benefit to the private establishments concerned even before tax
payments have been made. The tax credit that is contemplated under
the Act is a form of just compensation, not a remedy for taxes that were
erroneously or illegally assessed and collected. In the same vein, prior
payment of any tax liability is not a precondition before a taxable entity
can benefit from the tax credit. The credit may be availed of upon
payment of the tax due, if any. Where there is no tax liability or where
a private establishment reports a net loss for the period, the tax credit
can be availed of and carried over to the next taxable year.
THIRD DIVISION (2) The alleged understatement of ICC’s interest income on the three
promissory notes due from Realty Investment, Inc.
G.R. No. 172231 February 12, 2007
The deficiency expanded withholding tax of P4,897.79 (inclusive of
COMMISSIONER OF INTERNAL REVENUE, Petitioner, interest and surcharge) was allegedly due to the failure of ICC to
vs. withhold 1% expanded withholding tax on its claimed P244,890.00
ISABELA CULTURAL CORPORATION, Respondent. deduction for security services.7

DECISION On March 23, 1990, ICC sought a reconsideration of the subject


assessments. On February 9, 1995, however, it received a final notice
YNARES-SANTIAGO, J.: before seizure demanding payment of the amounts stated in the said
notices. Hence, it brought the case to the CTA which held that the
Petitioner Commissioner of Internal Revenue (CIR) assails the petition is premature because the final notice of assessment cannot be
September 30, 2005 Decision1 of the Court of Appeals in CA-G.R. SP considered as a final decision appealable to the tax court. This was
No. 78426 affirming the February 26, 2003 Decision2 of the Court of reversed by the Court of Appeals holding that a demand letter of the
Tax Appeals (CTA) in CTA Case No. 5211, which cancelled and set BIR reiterating the payment of deficiency tax, amounts to a final
aside the Assessment Notices for deficiency income tax and expanded decision on the protested assessment and may therefore be
withholding tax issued by the Bureau of Internal Revenue (BIR) against questioned before the CTA. This conclusion was sustained by this
respondent Isabela Cultural Corporation (ICC). Court on July 1, 2001, in G.R. No. 135210.8 The case was thus
remanded to the CTA for further proceedings.
The facts show that on February 23, 1990, ICC, a domestic corporation,
received from the BIR Assessment Notice No. FAS-1-86-90-000680 for On February 26, 2003, the CTA rendered a decision canceling and
deficiency income tax in the amount of P333,196.86, and Assessment setting aside the assessment notices issued against ICC. It held that
Notice No. FAS-1-86-90-000681 for deficiency expanded withholding the claimed deductions for professional and security services were
tax in the amount of P4,897.79, inclusive of surcharges and interest, properly claimed by ICC in 1986 because it was only in the said year
both for the taxable year 1986. when the bills demanding payment were sent to ICC. Hence, even if
some of these professional services were rendered to ICC in 1984 or
The deficiency income tax of P333,196.86, arose from: 1985, it could not declare the same as deduction for the said years as
the amount thereof could not be determined at that time.
(1) The BIR’s disallowance of ICC’s claimed expense deductions for
professional and security services billed to and paid by ICC in 1986, to The CTA also held that ICC did not understate its interest income on
wit: the subject promissory notes. It found that it was the BIR which made
an overstatement of said income when it compounded the interest
(a) Expenses for the auditing services of SGV & Co.,3 for the year income receivable by ICC from the promissory notes of Realty
ending December 31, 1985;4 Investment, Inc., despite the absence of a stipulation in the contract
providing for a compounded interest; nor of a circumstance, like delay
(b) Expenses for the legal services [inclusive of retainer fees] of the law in payment or breach of contract, that would justify the application of
firm Bengzon Zarraga Narciso Cudala Pecson Azcuna & Bengson for compounded interest.
the years 1984 and 1985.5
Likewise, the CTA found that ICC in fact withheld 1% expanded
(c) Expense for security services of El Tigre Security & Investigation withholding tax on its claimed deduction for security services as shown
Agency for the months of April and May 1986.6
by the various payment orders and confirmation receipts it presented auditing services, are: (a) the expense must be ordinary and necessary;
as evidence. The dispositive portion of the CTA’s Decision, reads: (b) it must have been paid or incurred during the taxable year; (c) it
must have been paid or incurred in carrying on the trade or business of
WHEREFORE, in view of all the foregoing, Assessment Notice No. the taxpayer; and (d) it must be supported by receipts, records or other
FAS-1-86-90-000680 for deficiency income tax in the amount of pertinent papers.11
P333,196.86, and Assessment Notice No. FAS-1-86-90-000681 for
deficiency expanded withholding tax in the amount of P4,897.79, The requisite that it must have been paid or incurred during the taxable
inclusive of surcharges and interest, both for the taxable year 1986, are year is further qualified by Section 45 of the National Internal Revenue
hereby CANCELLED and SET ASIDE. Code (NIRC) which states that: "[t]he deduction provided for in this Title
shall be taken for the taxable year in which ‘paid or accrued’ or ‘paid or
SO ORDERED.9 incurred’, dependent upon the method of accounting upon the basis of
which the net income is computed x x x".
Petitioner filed a petition for review with the Court of Appeals, which
affirmed the CTA decision,10 holding that although the professional Accounting methods for tax purposes comprise a set of rules for
services (legal and auditing services) were rendered to ICC in 1984 and determining when and how to report income and deductions.12 In the
1985, the cost of the services was not yet determinable at that time, instant case, the accounting method used by ICC is the accrual method.
hence, it could be considered as deductible expenses only in 1986
when ICC received the billing statements for said services. It further Revenue Audit Memorandum Order No. 1-2000, provides that under
ruled that ICC did not understate its interest income from the the accrual method of accounting, expenses not being claimed as
promissory notes of Realty Investment, Inc., and that ICC properly deductions by a taxpayer in the current year when they are incurred
withheld and remitted taxes on the payments for security services for cannot be claimed as deduction from income for the succeeding year.
the taxable year 1986. Thus, a taxpayer who is authorized to deduct certain expenses and
other allowable deductions for the current year but failed to do so
Hence, petitioner, through the Office of the Solicitor General, filed the cannot deduct the same for the next year.13
instant petition contending that since ICC is using the accrual method
of accounting, the expenses for the professional services that accrued The accrual method relies upon the taxpayer’s right to receive amounts
in 1984 and 1985, should have been declared as deductions from or its obligation to pay them, in opposition to actual receipt or payment,
income during the said years and the failure of ICC to do so bars it from which characterizes the cash method of accounting. Amounts of
claiming said expenses as deduction for the taxable year 1986. As to income accrue where the right to receive them become fixed, where
the alleged deficiency interest income and failure to withhold expanded there is created an enforceable liability. Similarly, liabilities are accrued
withholding tax assessment, petitioner invoked the presumption that when fixed and determinable in amount, without regard to
the assessment notices issued by the BIR are valid. indeterminacy merely of time of payment.14

The issue for resolution is whether the Court of Appeals correctly: (1) For a taxpayer using the accrual method, the determinative question is,
sustained the deduction of the expenses for professional and security when do the facts present themselves in such a manner that the
services from ICC’s gross income; and (2) held that ICC did not taxpayer must recognize income or expense? The accrual of income
understate its interest income from the promissory notes of Realty and expense is permitted when the all-events test has been met. This
Investment, Inc; and that ICC withheld the required 1% withholding tax test requires: (1) fixing of a right to income or liability to pay; and (2) the
from the deductions for security services. availability of the reasonable accurate determination of such income or
liability.
The requisites for the deductibility of ordinary and necessary trade,
business, or professional expenses, like expenses paid for legal and
The all-events test requires the right to income or liability be fixed, and deductions cannot thus be attributed solely to the delayed billing of
the amount of such income or liability be determined with reasonable these liabilities by the firm. For one, ICC, in the exercise of due
accuracy. However, the test does not demand that the amount of diligence could have inquired into the amount of their obligation to the
income or liability be known absolutely, only that a taxpayer has at his firm, especially so that it is using the accrual method of accounting. For
disposal the information necessary to compute the amount with another, it could have reasonably determined the amount of legal and
reasonable accuracy. The all-events test is satisfied where computation retainer fees owing to its familiarity with the rates charged by their long
remains uncertain, if its basis is unchangeable; the test is satisfied time legal consultant.
where a computation may be unknown, but is not as much as
unknowable, within the taxable year. The amount of liability does not As previously stated, the accrual method presents largely a question of
have to be determined exactly; it must be determined with fact and that the taxpayer bears the burden of establishing the accrual
"reasonable accuracy." Accordingly, the term "reasonable of an expense or income. However, ICC failed to discharge this burden.
accuracy" implies something less than an exact or completely As to when the firm’s performance of its services in connection with the
accurate amount.[15] 1984 tax problems were completed, or whether ICC exercised
reasonable diligence to inquire about the amount of its liability, or
The propriety of an accrual must be judged by the facts that a whether it does or does not possess the information necessary to
taxpayer knew, or could reasonably be expected to have known, compute the amount of said liability with reasonable accuracy, are
at the closing of its books for the taxable year.[16] Accrual method questions of fact which ICC never established. It simply relied on the
of accounting presents largely a question of fact; such that the taxpayer defense of delayed billing by the firm and the company, which under
bears the burden of proof of establishing the accrual of an item of the circumstances, is not sufficient to exempt it from being charged with
income or deduction.17 knowledge of the reasonable amount of the expenses for legal and
auditing services.
Corollarily, it is a governing principle in taxation that tax exemptions
must be construed in strictissimi juris against the taxpayer and liberally In the same vein, the professional fees of SGV & Co. for auditing the
in favor of the taxing authority; and one who claims an exemption must financial statements of ICC for the year 1985 cannot be validly claimed
be able to justify the same by the clearest grant of organic or statute as expense deductions in 1986. This is so because ICC failed to
law. An exemption from the common burden cannot be permitted to present evidence showing that even with only "reasonable accuracy,"
exist upon vague implications. And since a deduction for income tax as the standard to ascertain its liability to SGV & Co. in the year 1985,
purposes partakes of the nature of a tax exemption, then it must also it cannot determine the professional fees which said company would
be strictly construed.18 charge for its services.

In the instant case, the expenses for professional fees consist of ICC thus failed to discharge the burden of proving that the claimed
expenses for legal and auditing services. The expenses for legal expense deductions for the professional services were allowable
services pertain to the 1984 and 1985 legal and retainer fees of the law deductions for the taxable year 1986. Hence, per Revenue Audit
firm Bengzon Zarraga Narciso Cudala Pecson Azcuna & Bengson, and Memorandum Order No. 1-2000, they cannot be validly deducted from
for reimbursement of the expenses of said firm in connection with ICC’s its gross income for the said year and were therefore properly
tax problems for the year 1984. As testified by the Treasurer of ICC, disallowed by the BIR.
the firm has been its counsel since the 1960’s.19 From the nature of the
claimed deductions and the span of time during which the firm was As to the expenses for security services, the records show that these
retained, ICC can be expected to have reasonably known the retainer expenses were incurred by ICC in 198620 and could therefore be
fees charged by the firm as well as the compensation for its legal properly claimed as deductions for the said year.
services. The failure to determine the exact amount of the expense
during the taxable year when they could have been claimed as
Anent the purported understatement of interest income from the
promissory notes of Realty Investment, Inc., we sustain the findings of
the CTA and the Court of Appeals that no such understatement exists
and that only simple interest computation and not a compounded one
should have been applied by the BIR. There is indeed no stipulation
between the latter and ICC on the application of compounded
interest.21 Under Article 1959 of the Civil Code, unless there is a
stipulation to the contrary, interest due should not further earn interest.

Likewise, the findings of the CTA and the Court of Appeals that ICC
truly withheld the required withholding tax from its claimed deductions
for security services and remitted the same to the BIR is supported by
payment order and confirmation receipts.22 Hence, the Assessment
Notice for deficiency expanded withholding tax was properly cancelled
and set aside.

In sum, Assessment Notice No. FAS-1-86-90-000680 in the amount of


P333,196.86 for deficiency income tax should be cancelled and set
aside but only insofar as the claimed deductions of ICC for security
services. Said Assessment is valid as to the BIR’s disallowance of
ICC’s expenses for professional services. The Court of Appeal’s
cancellation of Assessment Notice No. FAS-1-86-90-000681 in the
amount of P4,897.79 for deficiency expanded withholding tax, is
sustained.

WHEREFORE, the petition is PARTIALLY GRANTED. The September


30, 2005 Decision of the Court of Appeals in CA-G.R. SP No. 78426, is
AFFIRMED with the MODIFICATION that Assessment Notice No. FAS-
1-86-90-000680, which disallowed the expense deduction of Isabela
Cultural Corporation for professional and security services, is declared
valid only insofar as the expenses for the professional fees of SGV &
Co. and of the law firm, Bengzon Zarraga Narciso Cudala Pecson
Azcuna & Bengson, are concerned. The decision is affirmed in all other
respects.

The case is remanded to the BIR for the computation of Isabela Cultural
Corporation’s liability under Assessment Notice No. FAS-1-86-90-
000680.
THIRD DIVISION expenditure was incurred "to create or maintain some form of good will
for the taxpayer’s trade or business or for the industry or profession of
G.R. No. 143672 April 24, 2003 which the taxpayer is a member." The term "good will" can hardly be
said to have any precise signification; it is generally used to denote the
COMMISSIONER OF INTERNAL REVENUE, petitioner, benefit arising from connection and reputation (Words and Phrases,
vs. Vol. 18, p. 556 citing Douhart vs. Loagan, 86 III. App. 294). As held in
GENERAL FOODS (PHILS.), INC., respondent. the case of Welch vs. Helvering, efforts to establish reputation are akin
to acquisition of capital assets and, therefore, expenses related thereto
CORONA, J.: are not business expenses but capital expenditures. (Atlas Mining and
Development Corp. vs. Commissioner of Internal Revenue, supra). For
Petitioner Commissioner of Internal Revenue (Commissioner) assails sure such expenditure was meant not only to generate present sales
the resolution1 of the Court of Appeals reversing the decision2 of the but more for future and prospective benefits. Hence, "abnormally large
Court of Tax Appeals which in turn denied the protest filed by expenditures for advertising are usually to be spread over the period of
respondent General Foods (Phils.), Inc., regarding the assessment years during which the benefits of the expenditures are received"
made against the latter for deficiency taxes. (Mertens, supra, citing Colonial Ice Cream Co., 7 BTA 154).
The records reveal that, on June 14, 1985, respondent corporation, WHEREFORE, in all the foregoing, and finding no error in the case
which is engaged in the manufacture of beverages such as "Tang," appealed from, we hereby RESOLVE to DISMISS the instant petition
"Calumet" and "Kool-Aid," filed its income tax return for the fiscal year for lack of merit and ORDER the Petitioner to pay the respondent
ending February 28, 1985. In said tax return, respondent corporation Commissioner the assessed amount of P2,635,141.42 representing its
claimed as deduction, among other business expenses, the amount deficiency income tax liability for the fiscal year ended February 28,
of P9,461,246 for media advertising for "Tang." 1985."3

On May 31, 1988, the Commissioner disallowed 50% or P4,730,623 of Aggrieved, respondent corporation filed a petition for review at the
the deduction claimed by respondent corporation. Consequently, Court of Appeals which rendered a decision reversing and setting aside
respondent corporation was assessed deficiency income taxes in the the decision of the Court of Tax Appeals:
amount of P2,635, 141.42. The latter filed a motion for reconsideration
but the same was denied. Since it has not been sufficiently established that the item it claimed as
a deduction is excessive, the same should be allowed.
On September 29, 1989, respondent corporation appealed to the Court
of Tax Appeals but the appeal was dismissed: WHEREFORE, the petition of petitioner General Foods (Philippines),
Inc. is hereby GRANTED. Accordingly, the Decision, dated 8 February
With such a gargantuan expense for the advertisement of a singular 1994 of respondent Court of Tax Appeals is REVERSED and SET
product, which even excludes "other advertising and promotions" ASIDE and the letter, dated 31 May 1988 of respondent Commissioner
expenses, we are not prepared to accept that such amount is of Internal Revenue is CANCELLED.
reasonable "to stimulate the current sale of merchandise" regardless of
Petitioner’s explanation that such expense "does not connote SO ORDERED.4
unreasonableness considering the grave economic situation taking
place after the Aquino assassination characterized by capital fight, Thus, the instant petition, wherein the Commissioner presents for the
strong deterioration of the purchasing power of the Philippine peso and Court’s consideration a lone issue: whether or not the subject media
the slacking demand for consumer products" (Petitioner’s advertising expense for "Tang" incurred by respondent corporation was
Memorandum, CTA Records, p. 273). We are not convinced with such
an explanation. The staggering expense led us to believe that such
an ordinary and necessary expense fully deductible under the National in carrying on a trade or business. Hence, it was necessary. However,
Internal Revenue Code (NIRC). their views conflict as to whether or not it was ordinary. To be
deductible, an advertising expense should not only be necessary but
It is a governing principle in taxation that tax exemptions must be also ordinary. These two requirements must be met.
construed in strictissimi juris against the taxpayer and liberally in favor
of the taxing authority;5 and he who claims an exemption must be able The Commissioner maintains that the subject advertising expense was
to justify his claim by the clearest grant of organic or statute law. An not ordinary on the ground that it failed the two conditions set by U.S.
exemption from the common burden cannot be permitted to exist upon jurisprudence: first, "reasonableness" of the amount incurred and
vague implications.6 second, the amount incurred must not be a capital outlay to create
"goodwill" for the product and/or private respondent’s business.
Deductions for income tax purposes partake of the nature of tax Otherwise, the expense must be considered a capital expenditure to be
exemptions; hence, if tax exemptions are strictly construed, then spread out over a reasonable time.
deductions must also be strictly construed.
We agree.
We then proceed to resolve the singular issue in the case at bar. Was
the media advertising expense for "Tang" paid or incurred by There is yet to be a clear-cut criteria or fixed test for determining the
respondent corporation for the fiscal year ending February 28, 1985 reasonableness of an advertising expense. There being no hard and
"necessary and ordinary," hence, fully deductible under the NIRC? Or fast rule on the matter, the right to a deduction depends on a number
was it a capital expenditure, paid in order to create "goodwill and of factors such as but not limited to: the type and size of business in
reputation" for respondent corporation and/or its products, which which the taxpayer is engaged; the volume and amount of its net
should have been amortized over a reasonable period? earnings; the nature of the expenditure itself; the intention of the
taxpayer and the general economic conditions. It is the interplay of
Section 34 (A) (1), formerly Section 29 (a) (1) (A), of the NIRC provides: these, among other factors and properly weighed, that will yield a
proper evaluation.
(A) Expenses.-
In the case at bar, the P9,461,246 claimed as media advertising
(1) Ordinary and necessary trade, business or professional expenses.- expense for "Tang" alone was almost one-half of its total claim for
"marketing expenses." Aside from that, respondent-corporation also
(a) In general.- There shall be allowed as deduction from gross income claimed P2,678,328 as "other advertising and promotions expense"
all ordinary and necessary expenses paid or incurred during the taxable and another P1,548,614, for consumer promotion.
year in carrying on, or which are directly attributable to, the
development, management, operation and/or conduct of the trade, Furthermore, the subject P9,461,246 media advertising expense for
business or exercise of a profession. "Tang" was almost double the amount of respondent corporation’s
P4,640,636 general and administrative expenses.
Simply put, to be deductible from gross income, the subject advertising
expense must comply with the following requisites: (a) the expense We find the subject expense for the advertisement of a single product
must be ordinary and necessary; (b) it must have been paid or incurred to be inordinately large. Therefore, even if it is necessary, it cannot be
during the taxable year; (c) it must have been paid or incurred in considered an ordinary expense deductible under then Section 29 (a)
carrying on the trade or business of the taxpayer; and (d) it must be (1) (A) of the NIRC.
supported by receipts, records or other pertinent papers.7
Advertising is generally of two kinds: (1) advertising to stimulate
The parties are in agreement that the subject advertising expense was the current sale of merchandise or use of services and (2) advertising
paid or incurred within the corresponding taxable year and was incurred designed to stimulate the future sale of merchandise or use of services.
The second type involves expenditures incurred, in whole or in part, to for the promotion of a single product, almost one-half of petitioner
create or maintain some form of goodwill for the taxpayer’s trade or corporation’s entire claim for marketing expenses for that year under
business or for the industry or profession of which the taxpayer is a review, inclusive of other advertising and promotion expenses of
member. If the expenditures are for the advertising of the first kind, P2,678,328 and P1,548,614 for consumer promotion, is doubtlessly
then, except as to the question of the reasonableness of amount, there unreasonable.
is no doubt such expenditures are deductible as business expenses. If,
however, the expenditures are for advertising of the second kind, then It has been a long standing policy and practice of the Court to respect
normally they should be spread out over a reasonable period of time. the conclusions of quasi-judicial agencies such as the Court of Tax
Appeals, a highly specialized body specifically created for the purpose
We agree with the Court of Tax Appeals that the subject advertising of reviewing tax cases. The CTA, by the nature of its functions, is
expense was of the second kind. Not only was the amount staggering; dedicated exclusively to the study and consideration of tax problems. It
the respondent corporation itself also admitted, in its letter protest8 to has necessarily developed an expertise on the subject. We extend due
the Commissioner of Internal Revenue’s assessment, that the subject consideration to its opinion unless there is an abuse or improvident
media expense was incurred in order to protect respondent exercise of authority.13 Since there is none in the case at bar, the Court
corporation’s brand franchise, a critical point during the period under adheres to the findings of the CTA.
review.
Accordingly, we find that the Court of Appeals committed reversible
The protection of brand franchise is analogous to the maintenance of error when it declared the subject media advertising expense to be
goodwill or title to one’s property. This is a capital expenditure which deductible as an ordinary and necessary expense on the ground that
should be spread out over a reasonable period of time.9 "it has not been established that the item being claimed as deduction is
excessive." It is not incumbent upon the taxing authority to prove that
Respondent corporation’s venture to protect its brand franchise was the amount of items being claimed is unreasonable. The burden of
tantamount to efforts to establish a reputation. This was akin to the proof to establish the validity of claimed deductions is on the
acquisition of capital assets and therefore expenses related thereto taxpayer.14 In the present case, that burden was not discharged
were not to be considered as business expenses but as capital satisfactorily.
expenditures.10
WHEREFORE, premises considered, the instant petition is GRANTED.
True, it is the taxpayer’s prerogative to determine the amount of The assailed decision of the Court of Appeals is hereby REVERSED
advertising expenses it will incur and where to apply them.11 Said and SET ASIDE. Pursuant to Sections 248 and 249 of the Tax Code,
prerogative, however, is subject to certain considerations. The first respondent General Foods (Phils.), Inc. is hereby ordered to pay its
relates to the extent to which the expenditures are actually capital deficiency income tax in the amount of P2,635,141.42, plus 25%
outlays; this necessitates an inquiry into the nature or purpose of such surcharge for late payment and 20% annual interest computed from
expenditures.12 The second, which must be applied in harmony with the August 25, 1989, the date of the denial of its protest, until the same is
first, relates to whether the expenditures are ordinary and necessary. fully paid.
Concomitantly, for an expense to be considered ordinary, it must be
reasonable in amount. The Court of Tax Appeals ruled that respondent
corporation failed to meet the two foregoing limitations.

We find said ruling to be well founded. Respondent corporation incurred


the subject advertising expense in order to protect its brand franchise.
We consider this as a capital outlay since it created goodwill for its
business and/or product. The P9,461,246 media advertising expense
G.R. No. L-24059 November 28, 1969 within thirty (30) days from the date this decision becomes final,
petitioner is also ordered to pay surcharge and interest as provided for
C. M. HOSKINS & CO., INC., petitioner, in Section 51 (e) of the Tax Code, without costs.
vs.
COMMISSIONER OF INTERNAL REVENUE, respondent. Petitioner questions in this appeal the Tax Court's findings that the
disallowed payment to Hoskins was an inordinately large one, which
Ross, Salcedo, Del Rosario, Bito and Misa for petitioner. bore a close relationship to the recipient's dominant stockholdings and
Office of the Solicitor General Arturo A. Alafriz, Assistant Solicitor therefore amounted in law to a distribution of its earnings and profits.
General Felicisimo R. Rosete and Special Attorney Michaelina R.
Balasbas for respondent. We find no merit in petitioner's appeal.

TEEHANKEE, J.: As found by the Tax Court, "petitioner was founded by Mr. C. M.
Hoskins in 1937, with a capital stock of 1,000 shares at a par value of
We uphold in this taxpayer's appeal the Tax Court's ruling that payment P1.00 each share; that of these 1,000 shares, Mr. C. M. Hoskins owns
by the taxpayer to its controlling stockholder of 50% of its supervision 996 shares (the other 4 shares being held by the other four officers of
fees or the amount of P99,977.91 is not a deductible ordinary and the corporation), which constitute exactly 99.6% of the total authorized
necessary expense and should be treated as a distribution of earnings capital stock (p. 92, t.s.n.); that during the first four years of its
and profits of the taxpayer. existence, Mr. C. M. Hoskins was the President, but during the taxable
period in question, that is, from October 1, 1956 to September 30, 1957,
Petitioner, a domestic corporation engaged in the real estate business he was the chairman of the Board of Directors and salesman-broker for
as brokers, managing agents and administrators, filed its income tax the company (p. 93, t.s.n.); that as chairman of the Board of Directors,
return for its fiscal year ending September 30, 1957 showing a net he received a salary of P3,750.00 a month, plus a salary bonus of about
income of P92,540.25 and a tax liability due thereon of P18,508.00, P40,000.00 a year (p. 94, t.s.n.); that he was also a stockholder and
which it paid in due course. Upon verification of its return, respondent officer of the Paradise Farms, Inc. and Realty Investments, Inc., from
Commissioner of Internal Revenue, disallowed four items of deduction which petitioner derived a large portion of its income in the form of
in petitioner's tax returns and assessed against it an income tax supervision fees and commissions earned on sales of lots (pp. 97-99,
deficiency in the amount of P28,054.00 plus interests. The Court of Tax t.s.n.; Financial Statements, attached to Exhibit '1', p. 11, BIR rec.); that
Appeals upon reviewing the assessment at the taxpayer's petition, as chairman of the Board of Directors of petitioner, his duties were: "To
upheld respondent's disallowance of the principal item of petitioner's act as a salesman; as a director, preside over meetings and to get all
having paid to Mr. C. M. Hoskins, its founder and controlling stockholder of the real estate business I could for the company by negotiating sales,
the amount of P99,977.91 representing 50% of supervision fees earned purchases, making appraisals, raising funds to finance real estate
by it and set aside respondent's disallowance of three other minor operations where that was necessary' (p. 96, t.s.n.); that he was familiar
items. The Tax Court therefore determined petitioner's tax deficiency to with the contract entered into by the petitioner with the Paradise Farms,
be in the amount of P27,145.00 and on November 8, 1964 rendered Inc. and the Realty Investments, Inc. by the terms of which petitioner
judgment against it, as follows: was 'to program the development, arrange financing, plan the proposed
subdivision as outlined in the prospectus of Paradise Farms, Inc.,
WHEREFORE, premises considered, the decision of the respondent is arrange contract for road constructions, with the provision of water
hereby modified. Petitioner is ordered to pay to the latter or his supply to all of the lots and in general to serve as managing agents for
representative the sum of P27,145.00, representing deficiency income the Paradise Farms, Inc. and subsequently for the Realty Investment,
tax for the year 1957, plus interest at 1/2% per month from June 20, Inc." (pp. 96-97. t.s.n.)
1959 to be computed in accordance with the provisions of Section 51(d)
of the National Internal Revenue Code. If the deficiency tax is not paid
Considering that in addition to being Chairman of the board of directors corporation, such as directing the policy of the corporation, delegating
of petitioner corporation, which bears his name, Hoskins, who owned powers to the president and advising the corporation in determining
99.6% of its total authorized capital stock while the four other officers- executive salaries, bonus plans and pensions, dividend policies, etc.1
stockholders of the firm owned a total of four-tenths of 1%, or one-tenth
of 1% each, with their respective nominal shareholdings of one share Petitioner's invoking of its policy since its incorporation of sharing
each was also salesman-broker for his company, receiving a 50% equally sales commissions with its salesmen, in accordance with its
share of the sales commissions earned by petitioner, besides his board resolution of June 18, 1946, is equally untenable. Petitioner's
monthly salary of P3,750.00 amounting to an annual compensation of Sales Regulations provide:
P45,000.00 and an annual salary bonus of P40,000.00, plus free use
of the company car and receipt of other similar allowances and benefits, Compensation of Salesmen
the Tax Court correctly ruled that the payment by petitioner to Hoskins
of the additional sum of P99,977.91 as his equal or 50% share of the 8. Schedule I — In the case of sales to prospects discovered and
8% supervision fees received by petitioner as managing agents of the worked by a salesman, even though the closing is done by or with the
real estate, subdivision projects of Paradise Farms, Inc. and Realty help of the Sales Manager or other members of the staff, the salesmen
Investments, Inc. was inordinately large and could not be accorded the get one-half (1/2) of the total commission received by the Company, but
treatment of ordinary and necessary expenses allowed as deductible not exceeding five percent (5%). In the case of subdivisions, when the
items within the purview of Section 30 (a) (i) of the Tax Code. office commission covers general supervision, the 1/2-rule does not
apply, the salesman's share being stipulated in the case of each
If such payment of P99,977.91 were to be allowed as a deductible item, subdivision. In most cases the salesman's share is 4%. (Exh. "N-1").2
then Hoskins would receive on these three items alone (salary, bonus
and supervision fee) a total of P184,977.91, which would be double the It will be readily seen therefrom that when the petitioner's commission
petitioner's reported net income for the year of P92,540.25. As correctly covers general supervision, it is provided that the 1/2 rule of equal
observed by respondent. If independently, a one-time P100,000.00-fee sharing of the sales commissions does not apply and that the
to plan and lay down the rules for supervision of a subdivision project salesman's share is stipulated in the case of each subdivision.
were to be paid to an experienced realtor such as Hoskins, its fairness Furthermore, what is involved here is not Hoskins' salesman's share in
and deductibility by the taxpayer could be conceded; but here 50% of the petitioner's 12% sales commission, which he presumably collected
the supervision fee of petitioner was being paid by it to Hoskins every also from petitioner without respondent's questioning it, but a 50%
year since 1955 up to 1963 and for as long as its contract with the share besides in petitioner's planning and supervision fee of 8% of the
subdivision owner subsisted, regardless of whether services were gross sales, as mentioned above. This is evident from petitioner's
actually rendered by Hoskins, since his services to petitioner included board's resolution of July 14, 1953 (Exhibit 7), wherein it is recited that
such planning and supervision and were already handsomely paid for in addition to petitioner's sales commission of 12% of gross sales, the
by petitioner. subdivision owners were paying to petitioner 8% of gross sales as
supervision fee, and a collection fee of 5% of gross collections, or total
The fact that such payment was authorized by a standing resolution of fees of 25% of gross sales.
petitioner's board of directors, since "Hoskins had personally conceived
and planned the project" cannot change the picture. There could be no The case before us is similar to previous cases of disallowances as
question that as Chairman of the board and practically an absolutely deductible items of officers' extra fees, bonuses and commissions,
controlling stockholder of petitioner, holding 99.6% of its stock, Hoskins upheld by this Court as not being within the purview of ordinary and
wielded tremendous power and influence in the formulation and making necessary expenses and not passing the test of reasonable
of the company's policies and decisions. Even just as board chairman, compensation.3 In Kuenzle & Streiff, Inc. vs. Commissioner of Internal
going by petitioner's own enumeration of the powers of the office, Revenue decided by this Court on May 29, 1969,4 we reaffirmed the
Hoskins, could exercise great power and influence within the test of reasonableness, enunciated in the earlier 1967 case involving
the same parties, that: "It is a general rule that 'Bonuses to employees bonuses as deductible expenses unless they are shown to be
made in good faith and as additional compensation for the services reasonable. To hold otherwise would open the gate of rampant tax
actually rendered by the employees are deductible, provided such evasion.
payments, when added to the stipulated salaries, do not exceed a
reasonable compensation for the services rendered' (4 Mertens Law of "Lastly, We must not lose sight of the fact that the question of allowing
Federal Income Taxation, Sec. 25.50, p. 410). The conditions or disallowing as deductible expenses the amounts paid to corporate
precedent to the deduction of bonuses to employees are: (1) the officers by way of bonus is determined by respondent exclusively for
payment of the bonuses is in fact compensation; (2) it must be for income tax purposes. Concededly, he has no authority to fix the
personal services actually rendered; and (3) the bonuses, when added amounts to be paid to corporate officers by way of basic salary, bonus
to the salaries, are 'reasonable . . . when measured by the amount and or additional remuneration — a matter that lies more or less exclusively
quality of the services performed with relation to the business of the within the sound discretion of the corporation itself. But this right of the
particular taxpayer' (Idem., Sec. 25, 44, p. 395). corporation is, of course, not absolute. It cannot exercise it for the
purpose of evading payment of taxes legitimately due to the State."
"There is no fixed test for determining the reasonableness of a given
bonus as compensation. This depends upon many factors, one of them Finally, it should be noted that we have here a case practically of a sole
being 'the amount and quality of the services performed with relation to proprietorship of C. M. Hoskins, who however chose to incorporate his
the business.' Other tests suggested are: payment must be 'made in business with himself holding virtually absolute control thereof with
good faith'; 'the character of the taxpayer's business, the volume and 99.6% of its stock with four other nominal shareholders holding one
amount of its net earnings, its locality, the type and extent of the share each. Having chosen to use the corporate form with its legal
services rendered, the salary policy of the corporation'; 'the size of the advantages of a separate corporate personality as distinguished from
particular business'; 'the employees' qualifications and contributions to his individual personality, the corporation so created, i.e., petitioner, is
the business venture'; and 'general economic conditions' (4 Mertens, bound to comport itself in accordance with corporate norms and comply
Law of Federal Income Taxation, Secs. 25.44, 25.49, 25.50, 25.51, pp. with its corporate obligations. Specifically, it is bound to pay the income
407-412). However, 'in determining whether the particular salary or tax imposed by law on corporations and may not legally be permitted,
compensation payment is reasonable, the situation must be considered by way of corporate resolutions authorizing payment of inordinately
as whole. Ordinarily, no single factor is decisive. . . . it is important to large commissions and fees to its controlling stockholder, to dilute and
keep in mind that it seldom happens that the application of one test can diminish its corresponding corporate tax liability.
give satisfactory answer, and that ordinarily it is the interplay of several
factors, properly weighted for the particular case, which must furnish ACCORDINGLY, the decision appealed from is hereby affirmed, with
the final answer." costs in both instances against petitioner.

Petitioner's case fails to pass the test. On the right of the employer as
against respondent Commissioner to fix the compensation of its officers
and employees, we there held further that while the employer's right
may be conceded, the question of the allowance or disallowance
thereof as deductible expenses for income tax purposes is subject to
determination by respondent Commissioner of Internal Revenue. Thus:
"As far as petitioner's contention that as employer it has the right to fix
the compensation of its officers and employees and that it was in the
exercise of such right that it deemed proper to pay the bonuses in
question, all that We need say is this: that right may be conceded, but
for income tax purposes the employer cannot legally claim such
G.R. No. L-13325 April 20, 1961 (a) Issue a writ of preliminary injunction, enjoining the respondents from
enforcing the collection of the alleged tax liability due from the petitioner
SANTIAGO GANCAYCO, petitioner, through summary proceeding pending determination of the present
vs. case;
THE COLLECTOR OF INTERNAL REVENUE, respondent.
(b) After a review of the present case adjudge that the right of the
Benjamin J. Molina for petitioner. government to enforce collection of any liability due on this account had
Office of the Solicitor General and Special Attorney Antonio A. Garces already prescribed;
for respondent.
(c) That even assuming that prescription had not set in the objections
CONCEPCION, J.: of petitioner to the disallowance of the entertainment, representation
and farming expenses be allowed;
Petitioner Santiago Gancayco seeks the review of a decision of the
Court of Tax Appeals, requiring him to pay P16,860.31, plus surcharge xxx xxx xxx
and interest, by way of deficiency income tax for the year 1949.
In his answer respondent admitted some allegations the amended
On May 10, 1950, Gancayco filed his income tax return for the year petition, denied other allegations thereof an set up some special
1949. Two (2) days later, respondent Collector of Internal Revenue defenses. Thereafter Gancayco received from the municipal treasurer
issued the corresponding notice advising him that his income tax of Catanauan, Quezon, another notice of auction sale of his properties,
liability for that year amounted P9,793.62, which he paid on May 15, to take place on August 29, 1956. On motion of Gancayco, the Court of
1950. A year later, on May 14, 1951, respondent wrote the Tax Appeals, by resolution dated August 27, 1956, "cancelled" the
communication Exhibit C, notifying Gancayco, inter alia, that, upon aforementioned sale and enjoined respondent and the municipal
investigation, there was still due from him, a efficiency income tax for treasurer of Catanauan, Quezon, from proceeding with the same. After
the year 1949, the sum of P29,554.05. Gancayco sought a appropriate proceedings, the Court of Tax Appeals rendered, on
reconsideration, which was part granted by respondent, who in a letter November 14, 1957, the decision adverted to above.
dated April 8, 1953 (Exhibit D), informed petitioner that his income tax
defendant efficiency for 1949 amounted to P16,860.31. Gancayco Gancayco maintains that the right to collect the deficiency income tax
urged another reconsideration (Exhibit O), but no action taken on this in question is barred by the statute of limitations. In this connection, it
request, although he had sent several communications calling should be noted, however, that there are two (2) civil remedies for the
respondent's attention thereto. collection of internal revenue taxes, namely: (a) by distraint of personal
property and levy upon real property; and (b) by "judicial action"
On April 15, 1956, respondent issued a warrant of distraint and levy (Commonwealth Act 456, section 316). The first may not be availed of
against the properties of Gancayco for the satisfaction of his deficiency except within three (3) years after the "return is due or has been made
income tax liability, and accordingly, the municipal treasurer of ..." (Tax Code, section 51 [d] ). After the expiration of said Period,
Catanauan, Quezon issued on May 29, 1956, a notice of sale of said income taxes may not be legally and validly collected by distraint and/or
property at public auction on June 19, 1956. Upon petition of Gancayco levy (Collector of Internal Revenue v. Avelino, L-9202, November 19,
filed on June 16, 1956, the Court of Tax Appeal issued a resolution 1956; Collector of Internal Revenue v. Reyes, L-8685, January 31,
ordering the cancellation of the sale and directing that the same be 1957; Collector of Internal Revenue v. Zulueta, L-8840, February 8,
readvertised at a future date, in accordance with the procedure 1957; Sambrano v. Court of Tax Appeals, L-8652, March 30, 1957).
established by the National Internal Revenue Code. Subsequently, or Gancayco's income tax return for 1949 was filed on May 10, 1950; so
on June 22, 1956, Gancayco filed an amended petition praying that that the warrant of distraint and levy issued on May 15, 1956, long after
said Court:
the expiration of said three-year period, was illegal and void, and so section 316 of the Tax Code, which petitioner denies, upon the ground
was the attempt to sell his properties in pursuance of said warrant. that

The "judicial action" mentioned in the Tax Code may be resorted to a. "The Court of Tax Appeals does not have original jurisdiction to
within five (5) years from the date the return has been filed, if there has entertain an action for the collection of the tax due;
been no assessment, or within five (5) years from the date of the
assessment made within the statutory period, or within the period b. "The proper party to commence the judicial action to collect the tax
agreed upon, in writing, by the Collector of Internal Revenue and the due is the government, and
taxpayer. before the expiration of said five-year period, or within such
extension of said stipulated period as may have been agreed upon, in c. "The remedies provided by law for the collection of the tax are
writing, made before the expiration of the period previously situated, exclusive."
except that in the case of a false or fraudulent return with intent to evade
tax or of a failure to file a return, the judicial action may be begun at any Said Section 316 provides:
time within ten (10) years after the discovery of the falsity, fraud or
omission (Sections 331 and 332 of the Tax Code). In the case at bar, The civil remedies for the collection of internal revenue taxes, fees, or
respondent made three (3) assessments: (a) the original assessment charges, and any increment thereto resulting from delinquency shall be
of P9,793.62, made on May 12, 1950; (b) the first deficiency income tax (a) by distraint of goods, chattels, or effects, and other personal
assessment of May 14, 1951, for P29,554.05; and (c) the amended property of whatever character, including stocks and other securities,
deficiency income tax assessment of April 8, 1953, for P16,860.31. debts, credits, bank accounts, and interest in and rights to personal
property, and by levy upon real property; and (b) by judicial action.
Gancayco argues that the five-year period for the judicial action should Either of these remedies or both simultaneously may be pursued in the
be counted from May 12, 1950, the date of the original assessment, discretion of the authorities charged with the collection of such taxes.
because the income tax for 1949, he says, could have been collected
from him since then. Said assessment was, however, not for the No exemption shall be allowed against the internal revenue taxes in
deficiency income tax involved in this proceedings, but for P9,793.62, any case.
which he paid forthwith. Hence, there never had been any cause for a
judicial action against him, and, per force, no statute of limitations to Petitioner contends that the judicial action referred to in this provision
speak of, in connection with said sum of P9,793.62. is commenced by filing, with a court of first instance, of a complaint for
the collection of taxes. This was true at the time of the approval of
Neither could said statute have begun to run from May 14, 1951, the Commonwealth Act No. 456, on June 15, 1939. However, Republic Act
date of the first deficiency income tax assessment or P29,554.05, No. 1125 has vested the Court of Tax Appeals, not only with exclusive
because the same was, upon Gancayco's request, reconsidered or appellate jurisdiction to review decisions of the Collector (now
modified by the assessment made on April 8, 1953, for P16,860.31. Commissioner) of Internal Revenue in cases involving disputed
Indeed, this last assessment is what Gancayco contested in the assessments, like the one at bar, but, also, with authority to decide "all
amended petition filed by him with the Court of Tax Appeals. The cases involving disputed assessments of Internal Revenue taxes or
amount involved in such assessment which Gancayco refused to pay customs duties pending determination before the court of first instance"
and respondent tried to collect by warrant of distraint and/or levy, is the at the time of the approval of said Act, on June 16, 1954 (Section 22,
one in issue between the parties. Hence, the five-year period Republic Act No. 1125). Moreover, this jurisdiction to decide all cases
aforementioned should be counted from April 8, 1953, so that the involving disputed assessments of internal revenue taxes and customs
statute of limitations does not bar the present proceedings, instituted duties necessarily implies the power to authorize and sanction the
on April 12, 1956, if the same is a judicial action, as contemplated in collection of the taxes and duties involved in such assessments as may
be upheld by the Court of Tax Appeals. At any rate, the same now has
the authority formerly vested in courts of first instance to hear and
decide cases involving disputed assessments of internal revenue taxes permanent improvements, or betterments made to increase the
and customs duties. Inasmuch as those cases filed with courts of first value of any property or estate. (Emphasis supplied.)
instance constituted judicial actions, such is, likewise, the nature of the
proceedings before the Court of Tax Appeals, insofar as sections 316 We concur in this view, which is a necessary consequence of section
and 332 of the Tax Code are concerned. 31 of the Tax Code, pursuant to which:

The question whether the sum of P16,860.31 is due from Gancayco as (a) General Rule — In computing net income no deduction shall
deficiency income tax for 1949 hinges on the validity of his claim for in any case be allowed in respect of —
deduction of two (2) items, namely: (a) for farming expenses,
P27,459.00; and (b) for representation expenses, P8,933.45. (1) Personal, living, or family expenses;

Section 30 of the Tax Code partly reads: (2) Any amount paid out for new buildings or for permanent
improvements, or betterments made to increase the value of any
(a) Expenses: property or estate;

(1) In General — All the ordinary and necessary expenses paid or (3) Any amount expended in restoring property or in making good the
incurred during the taxable year in carrying on any trade or business, exhaustion thereof for which an allowance is or has been made; or
including a reasonable allowance for salaries or other compensation for
personal services actually rendered; traveling expenses while away (4) Premiums paid on any life insurance policy covering the life of any
from home in the pursuit of a trade or business; and rentals or other officer or employee, or any person financially interested in any trade or
payments required to be made as a condition to the continued use or business carried on by the taxpayer, individual or corporate, when the
possession, for the purposes of the trade or business, of property to taxpayer is directly or indirectly a beneficiary under such policy.
which the taxpayer has not taken or is not taking title or in which he has (Emphasis supplied.)
no equity. (Emphasis supplied.)
Said view is, likewise, in accord with the consensus of the authorities
Referring to the item of P27,459, for farming expenses allegedly on the subject.
incurred by Gancayco, the decision appealed from has the following to
say: Expenses incident to the acquisition of property follow the same rule as
applied to payments made as direct consideration for the property. For
No evidence has been presented as to the nature of the said "farming example, commission paid in acquiring property are considered as
expenses" other than the bare statement of petitioner that they were representing part of the cost of the property acquired. The same
spent for the "development and cultivation of (his) property". No treatment is to be accorded to amounts expended for maps, abstracts,
specification has been made as to the actual amount spent for legal opinions on titles, recording fees and surveys. Other non-
purchase of tools, equipment or materials, or the amount spent for deductible expenses include amounts paid in connection with
improvement. Respondent claims that the entire amount was spent geological explorations, development and subdividing of real
exclusively for clearing and developing the farm which were necessary estate; clearing and grading; restoration of soil, drilling wells,
to place it in a productive state. It is not, therefore, an ordinary expense architects's fees and similar types of expenditures. (4 Merten's Law of
but a capitol expenditure. Accordingly, it is not deductible but it may be Federal Income Taxation, Sec. 25.20, pp. 348-349; see also sec. 75 of
amortized, in accordance with section 75 of Revenue Regulations No. the income Regulation of the B.I.R.; Emphasis supplied.)
2, cited above. See also, section 31 of the Revenue Code which
provides that in computing net income, no deduction shall in any case The cost of farm machinery, equipment and farm building represents a
be allowed in respect of any amount paid out for new buildings or for capital investment and is not an allowable deduction as an item of
expense. Amounts expended in the development of farms, orchards,
and ranches prior to the time when the productive state is reached may Padilla, Bautista Angelo, Labrador, Reyes, J.B.L., Barrera and Dizon,
be regarded as investments of capital. (Merten's Law of Federal Income JJ., concur.
Taxation, supra, sec. 25.108, p. 525.)

Expenses for clearing off and grading lots acquired is


a capital expenditure, representing part of the cost of the land and was
not deductible as an expense. (Liberty Banking Co. v. Heiner 37 F [2d]
703 [8AFTR 100111] [CCA 3rd]; The B.L. Marble Chair Company v.
U.S., 15 AFTR 746).

An item of expenditure, in order to be deductible under this section of


the statute providing for the deduction
of ordinary and necessary business expenses, must
fall squarely within the language of the statutory provision. This section
is intended primarily, although not always necessarily, to cover
expenditures of a recurring nature where the benefit derived from the
payment is realized and exhausted within the taxable year.
Accordingly, if the result of the expenditure is the acquisition of an
asset which has an economically useful life beyond the taxable year,
no deduction of such payment may be obtained under the provisions of
the statute. In such cases, to the extent that a deduction is allowable, it
must be obtained under the provisions of the statute which permit
deductions for amortization, depreciation, depletion or loss. (W.B.
Harbeson Co. 24 BTA, 542; Clark Thread Co., 28 BTA 1128 aff'd 100
F [2d] 257 [CCA 3rd, 1938]; 4 Merten's Law of Federal Income
Taxation, Sec. 25.17, pp. 337-338.)

Gancayco's claim for representation expenses aggregated P31,753.97,


of which P22,820.52 was allowed, and P8,933.45 disallowed. Such
disallowance is justified by the record, for, apart from the absence of
receipts, invoices or vouchers of the expenditures in question,
petitioner could not specify the items constituting the same, or when or
on whom or on what they were incurred. The case of Cohan v.
Commissioner, 39 F (2d) 540, cited by petitioner is not in point, because
in that case there was evidence on the amounts spent and the persons
entertained and the necessity of entertaining them, although there were
no receipts an vouchers of the expenditures involved therein. Such is
not the case of petitioner herein.

Being in accordance with the facts and law, the decision of the Court of
Tax Appeals is hereby affirmed therefore, with costs against petitioner
Santiago Cancayco. It is so ordered.
G.R. Nos. 106949-50 December 1, 1995 35% Transaction tax due

PAPER INDUSTRIES CORPORATION OF THE PHILIPPINES thereon 16,020,147.00


(PICOP), petitioner,
vs. Add: 25% surcharge 4,005,036.75
COURT OF APPEALS, COMMISSIONER OF INTERNAL REVENUE
and COURT OF TAX APPEALS, respondents. ——————

G.R. Nos. 106984-85 December 1, 1995 T o t a l P 20,025,183.75

COMMISSIONER INTERNAL REVENUE, petitioner, Add:


vs.
PAPER INDUSTRIES CORPORATION OF THE PHILIPPINES, THE 14% int. fr.
COURT OF APPEALS and THE COURT OF TAX
APPEALS, respondents. 1-20-78 to

7-31-80 P 7,093,302.57

FELICIANO, J.: 20% int, fr.

The Paper Industries Corporation of the Philippines ("Picop"), which is 8-1-80 to


petitioner in G.R. Nos. 106949-50 and private respondent in G.R. Nos.
106984-85, is a Philippine corporation registered with the Board of 3-31-83 10,675,523.58
Investments ("BOI") as a preferred pioneer enterprise with respect to
its integrated pulp and paper mill, and as a preferred non- ——————
pioneer enterprise with respect to its integrated plywood and veneer
mills. 17,768,826.15

On 21 April 1983, Picop received from the Commissioner of Internal ——————


Revenue ("CIR") two (2) letters of assessment and demand both dated
31 March 1983: (a) one for deficiency transaction tax and for P 37,794,009.90
documentary and science stamp tax; and (b) the other for deficiency
Documentary and Science Stamps Tax
income tax for 1977, for an aggregate amount of P88,763,255.00.
These assessments were computed as follows:
Total face value of
Transaction Tax
debentures P100,000,000.00
Interest payments on
Documentary Stamps
money market
Tax Due
borrowings P 45,771,849.00
(P0.30 x P100,000.000 )
———————
( P200 ) P 150,000.00 of machinery & other

Science Stamps Tax Due equipment 42,840,131.00

(P0.30 x P100,000,000 ) 3) Unexplained financial

( P200 ) P 150,000.00 guarantee expense 1,237,421.00

—————— 4) Understatement

T o t a l P 300,000.00 of sales 2,391,644.00

Add: Compromise for 5) Overstatement of

non-affixture 300.00 cost of sales 604,018.00

—————— ——————

300,300.00 P91,406,194.00

—————— Net income per investigation P91,664,360.00

TOTAL AMOUNT DUE AND COLLECTIBLE P 38,094,309.90 Income tax due thereon 34,734,559.00

=========== Less: Tax already assessed per return 80,358.00

Deficiency Income Tax for 1977 ——————

Net income per return P 258,166.00 Deficiency P34,654,201.00

Add: Unallowable deductions Add:

1) Disallowed deductions 14% int. fr.

availed of under 4-15-78 to

R.A. No. 5186 P 44,332,980.00 7-31-81 P 11,128,503.56

2) Capitalized interest 20% int. fr.

expenses on funds 8-1-80 to

used for acquisition 4-15-81 4,886,242.34


—————— Appeals. The Court of Appeals consolidated the two (2) cases and
rendered a decision, dated 31 August 1992, which further reduced the
P16,014,745.90 liability of Picop to P6,338,354.70. The dispositive portion of the Court
of Appeals decision reads as follows:
——————
WHEREFORE, the appeal of the Commissioner of Internal Revenue is
TOTAL AMOUNT DUE AND COLLECTIBLE P 50,668,946.90 1 denied for lack of merit. The judgment against PICOP is modified, as
follows:
===========
1. PICOP is declared liable for the 35% transaction tax in the amount
On 26 April 1983, Picop protested the assessment of deficiency of P3,578,543.51;
transaction tax and documentary and science stamp taxes. Picop also
protested on 21 May 1983 the deficiency income tax assessment for 2. PICOP is absolved from the payment of documentary and science
1977. These protests were not formally acted upon by respondent CIR. stamp tax of P300,000.00 and the compromise penalty of P300.00;
On 26 September 1984, the CIR issued a warrant of distraint on
personal property and a warrant of levy on real property against Picop, 3. PICOP shall pay 20% interest per annum on the deficiency income
to enforce collection of the contested assessments; in effect, the CIR tax of P1,481,579.15, for a period of three (3) years from 21 May 1983,
denied Picop's protests. or in the total amount of P888,947.49, and a surcharge of 10% on the
latter amount, or P88,984.75.
Thereupon, Picop went before the Court of Tax Appeals ("CTA")
appealing the assessments. After trial, the CTA rendered a decision No pronouncement as to costs.
dated 15 August 1989, modifying the findings of the CIR and holding
Picop liable for the reduced aggregate amount of P20,133,762.33, SO ORDERED.
which was itemized in the dispositive portion of the decision as follows:
Picop and the CIR once more filed separate Petitions for Review before
35% Transaction Tax P 16,020,113.20 the Supreme Court. These cases were consolidated and, on 23 August
1993, the Court resolved to give due course to both Petitions in G.R.
Documentary & Science Nos. 106949-50 and 106984-85 and required the parties to file their
Memoranda.
Stamp Tax 300,300.00
Picop now maintains that it is not liable at all to pay any of the
Deficiency Income Tax Due 3,813,349.33 assessments or any part thereof. It assails the propriety of the thirty-
five percent (35%) deficiency transaction tax which the Court of
—————— Appeals held due from it in the amount of P3,578,543.51. Picop also
questions the imposition by the Court of Appeals of the deficiency
TOTAL AMOUNT DUE AND PAYABLE P 20,133,762.53 2 income tax of P1,481,579.15, resulting from disallowance of certain
claimed financial guarantee expenses and claimed year-end
=========== adjustments of sales and cost of sales figures by Picop's external
auditors. 3
Picop and the CIR both went to the Supreme Court on separate
Petitions for Review of the above decision of the CTA. In two (2) The CIR, upon the other hand, insists that the Court of Appeals erred
Resolutions dated 7 February 1990 and 19 February 1990, in finding Picop not liable for surcharge and interest on unpaid
respectively, the Court referred the two (2) Petitions to the Court of
transaction tax and for documentary and science stamp taxes and in We will consider these issues in the foregoing sequence.
allowing Picop to claim as deductible expenses:
I.
(a) the net operating losses of another corporation (i.e., Rustan Pulp
and Paper Mills, Inc.); and (1) Whether Picop is liable
for the thirty-five percent
(b) interest payments on loans for the purchase of machinery and (35%) transaction tax.
equipment.
With the authorization of the Securities and Exchange Commission,
The CIR also claims that Picop should be held liable for interest at Picop issued commercial paper consisting of serially numbered
fourteen percent (14%) per annum from 15 April 1978 for three (3) promissory notes with the total face value of P229,864,000.00 and a
years, and interest at twenty percent (20%) per annum for a maximum maturity period of one (1) year, i.e., from 24 December 1977 to 23
of three (3) years; and for a surcharge of ten percent (10%), on Picop's December 1978. These promissory notes were purchased by various
deficiency income tax. Finally, the CIR contends that Picop is liable for commercial banks and financial institutions. On these promissory
the corporate development tax equivalent to five percent (5%) of its notes, Picop paid interest in the aggregate amount of P45,771,849.00.
correct 1977 net income. In respect of these interest payments, the CIR required Picop to pay
the thirty-five percent (35%) transaction tax.
The issues which we must here address may be sorted out and
grouped in the following manner: The CIR based this assessment on Presidential Decree No. 1154 dated
3 June 1977, which reads in part as follows:
I. Whether Picop is liable for:
Sec. 1. The National Internal Revenue Code, as amended, is hereby
(1) the thirty-five percent (35%) transaction tax; further amended by adding a new section thereto to read as follows:

(2) interest and surcharge on unpaid transaction tax; and Sec. 195-C. Tax on certain interest. — There shall be levied, assessed,
collected and paid on every commercial paper issued in the primary
(3) documentary and science stamp taxes; market as principal instrument, a transaction tax equivalent to thirty-five
percent (35%) based on the gross amount of interest thereto as defined
II. Whether Picop is entitled to deductions against income of: hereunder, which shall be paid by the borrower/issuer: Provided,
however, that in the case of a long-term commercial paper whose
(1) interest payments on loans for the purchase of machinery and maturity exceeds more than one year, the borrower shall pay the tax
equipment; based on the amount of interest corresponding to one year, and
thereafter shall pay the tax upon accrual or actual payment (whichever
(2) net operating losses incurred by the Rustan Pulp and Paper Mills, is earlier) of the untaxed portion of the interest which corresponds to a
Inc.; and period not exceeding one year.
(3) certain claimed financial guarantee expenses; and The transaction tax imposed in this section shall be a final tax to be paid
by the borrower and shall be allowed as a deductible item for purposes
III. (1) Whether Picop had understated its sales and overstated its cost of computing the borrower's taxable income.
of sales for 1977; and
For purposes of this tax —
(2) Whether Picop is liable for the corporate development tax of five
percent (5%) of its net income for 1977.
(a) "Commercial paper" shall be defined as an instrument evidencing which in the form it existed in 1977-1978, read in relevant part as
indebtedness of any person or entity, including banks and non-banks follows:
performing quasi-banking functions, which is issued, endorsed, sold,
transferred or in any manner conveyed to another person or entity, Sec. 8. Incentives to a Pioneer Enterprise. In addition to the incentives
either with or without recourse and irrespective of provided in the preceding section, pioneer enterprises shall be granted
maturity. Principally, commercial papers are promissory notes and/or the following incentive benefits:
similar instruments issued in the primary market and shall not include
repurchase agreements, certificates of assignments, certificates of (a) Tax Exemption. Exemption from all taxes under the National
participations, and such other debt instruments issued in the secondary Internal Revenue Code, except income tax, from the date the area of
market. investment is included in the Investment Priorities Plan to the following
extent:
(b) The term "interest" shall mean the difference between what the
principal borrower received and the amount it paid upon maturity of the (1) One hundred per cent (100%) for the first five years;
commercial paper which shall, in no case, be lower than the interest
rate prevailing at the time of the issuance or renewal of the commercial (2) Seventy-five per cent (75%) for the sixth through the eighth years;
paper. Interest shall be deemed synonymous with discount and shall
include all fees, commissions, premiums and other payments which (3) Fifty per cent (50%) for the ninth and tenth years;
form integral parts of the charges imposed as a consequence of the
use of money. (4) Twenty per cent (20%) for the eleventh and twelfth years; and

In all cases, where no interest rate is stated or if the rate stated is lower (5) Ten per cent (10%) for the thirteenth through the fifteenth year.
than the prevailing interest rate at the time of the issuance or renewal
of commercial paper, the Commissioner of Internal Revenue, upon xxx xxx xxx 4
consultation with the Monetary Board of the Central Bank of the
Philippines, shall adjust the interest rate in accordance herewith, and We agree with the CTA and the Court of Appeals that Picop's tax
assess the tax on the basis thereof. exemption under R.A. No. 5186, as amended, does not include
exemption from the thirty-five percent (35%) transaction tax. In the first
The tax herein imposed shall be remitted by the borrower to the place, the thirty-five percent (35%) transaction tax 5 is an income tax,
Commissioner of Internal Revenue or his Collection Agent in the that is, it is a tax on the interest income of the lenders or creditors.
municipality where such borrower has its principal place of business In Western Minolco Corporation v. Commissioner of Internal
within five (5) working days from the issuance of the commercial paper. Revenue, 6 the petitioner corporation borrowed funds from several
In the case of long term commercial paper, the tax upon the untaxed financial institutions from June 1977 to October 1977 and paid the
portion of the interest which corresponds to a period not exceeding one corresponding thirty-five (35%) transaction tax thereon in the amount
year shall be paid upon accrual payment, whichever is earlier. of P1,317,801.03, pursuant to Section 210 (b) of the 1977 Tax Code.
(Emphasis supplied) Western Minolco applied for refund of that amount alleging it was
exempt from the thirty-five (35%) transaction tax by reason of Section
Both the CTA and the Court of Appeals sustained the assessment of 79-A of C.A. No. 137, as amended, which granted new mines and old
transaction tax. mines resuming operation "five (5) years complete tax exemptions,
except income tax, from the time of its actual bonafide orders for
In the instant Petition, Picop reiterates its claim that it is exempt from equipment for commercial production." In denying the claim for refund,
the payment of the transaction tax by virtue of its tax exemption under this Court held:
R.A. No. 5186, as amended, known as the Investment Incentives Act,
The petitioner's contentions deserve scant consideration. The 35% Much the same issue was passed upon in Marinduque Mining Industrial
transaction tax is imposed on interest income from commercial papers Corporation v. Commissioner of Internal Revenue 8 and resolved in the
issued in the primary money market. Being a tax on interest, it is a tax same way:
on income.
It is very obvious that the transaction tax, which is a tax on interest
As correctly ruled by the respondent Court of Tax Appeals: derived from commercial paper issued in the money market, is not a
tax contemplated in the above-quoted legal provisions. The petitioner
Accordingly, we need not and do not think it necessary to discuss admits that it is subject to income tax. Its tax exemption should be
further the nature of the transaction tax more than to say that the strictly construed.
incipient scheme in the issuance of Letter of Instructions No. 340 on
November 24, 1975 (O.G. Dec. 15, 1975), i.e., to achieve operational We hold that petitioner's claim for refund was justifiably denied. The
simplicity and effective administration in capturing the interest-income transaction tax, although nominally categorized as a business tax, is in
"windfall" from money market operations as a new source of revenue, reality a withholding tax as positively stated in LOI No. 340. The
has lost none of its animating principle in parturition of amendatory petitioner could have shifted the tax to the lenders or recipients of the
Presidential Decree No. 1154, now Section 210 (b) of the Tax interest. It did not choose to do so. It cannot be heard now to complain
Code. The tax thus imposed is actually a tax on interest earnings of the about the tax. LOI No. 340 is an extraneous or extrinsic aid to the
lenders or placers who are actually the taxpayers in whose income is construction of section 210 (b).
imposed. Thus "the borrower withholds the tax of 35% from the interest
he would have to pay the lender so that he (borrower) can pay the 35% xxx xxx xxx 9
of the interest to the Government." (Citation omitted) . . . . Suffice it to
state that the broad consensus of fiscal and monetary authorities is that (Emphasis supplied)
"even if nominally, the borrower is made to pay the tax, actually, the tax
is on the interest earning of the immediate and all prior lenders/placers It is thus clear that the transaction tax is an income tax and as such, in
of the money. . . ." (Rollo, pp. 36-37) any event, falls outside the scope of the tax exemption granted to
registered pioneer enterprises by Section 8 of R.A. No. 5186, as
The 35% transaction tax is an income tax on interest earnings to the amended. Picop was the withholding agent, obliged to withhold thirty-
lenders or placers. The latter are actually the taxpayers. Therefore, the five percent (35%) of the interest payable to its lenders and to remit the
tax cannot be a tax imposed upon the petitioner. In other words, the amounts so withheld to the Bureau of Internal Revenue ("BIR"). As a
petitioner who borrowed funds from several financial institutions by withholding agent, Picop is made personally liable for the thirty-five
issuing commercial papers merely withheld the 35% transaction tax percent (35%) transaction tax 10 and if it did not actually withhold thirty-
before paying to the financial institutions the interests earned by them five percent (35%) of the interest monies it had paid to its lenders, Picop
and later remitted the same to the respondent Commissioner of Internal had only itself to blame.
Revenue. The tax could have been collected by a different procedure
but the statute chose this method. Whatever collecting procedure is Picop claims that it had relied on a ruling, dated 6 October 1977, issued
adopted does not change the nature of the tax. by the CIR, which held that Picop was not liable for the thirty-five (35%)
transaction tax in respect of debenture bonds issued by Picop. Prior to
xxx xxx xxx 7 the issuance of the promissory notes involved in the instant case, Picop
had also issued debenture bonds P100,000,000.00 in aggregate face
(Emphasis supplied) value. The managing underwriter of this debenture bond issue, Bancom
Development Corporation, requested a formal ruling from the Bureau
of Internal Revenue on the liability of Picop for the thirty-five percent
(35%) transaction tax in respect of such bonds. The ruling rendered by
the then Acting Commissioner of Internal Revenue, Efren I. Plana, In the above quoted ruling, the CIR basically held that Picop's
stated in relevant part: debenture bonds did not constitute "commercial papers" within the
meaning of P.D. No. 1154, and that, as such, those bonds were not
It is represented that PICOP will be offering to the public primary bonds subject to the thirty-five percent (35%) transaction tax imposed by P.D.
in the aggregate principal sum of one hundred million pesos No. 1154.
(P100,000,000.00); that the bonds will be issued as debentures in
denominations of one thousand pesos (P1,000.00) or multiples, to The above ruling, however, is not applicable in respect of the
mature in ten (10) years at 14% interest per annum payable semi- promissory notes which are the subject matter of the instant case. It
annually; that the bonds are convertible into common stock of the must be noted that the debenture bonds which were the subject matter
issuer at the option of the bond holder at an agreed conversion price; of Commissioner Plana's ruling were long-term bonds maturing in ten
that the issue will be covered by a "Trust Indenture" with a duly (10) years and which could not be pre-terminated and could not be
authorized trust corporation as required by the Securities and redeemed by Picop until after eight (8) years from date of issue; the
Exchange Commission, which trustee will act for and in behalf of the bonds were moreover subordinated to present and future debts of
debenture bond holders as beneficiaries; that once issued, the bonds Picop and convertible into common stock of Picop at the option of the
cannot be preterminated by the holder and cannot be redeemed by the bondholder. In contrast, the promissory notes involved in the instant
issuer until after eight (8) years from date of issue; that the debenture case are short-term instruments bearing a one-year maturity period.
bonds will be subordinated to present and future debts of PICOP; and These promissory notes constitute the very archtype of money market
that said bonds are intended to be listed in the stock exchanges, which instruments. For money market instruments are precisely, by custom
will place them alongside listed equity issues. and usage of the financial markets, short-term instruments with a tenor
of one (1) year or less. 12 Assuming, therefore, (without passing upon)
In reply, I have the honor to inform you that although the bonds the correctness of the 6 October 1977 BIR ruling, Picop's short-term
hereinabove described are commercial papers which will be issued in promissory notes must be distinguished, and treated differently, from
the primary market, however, it is clear from the abovestated facts that Picop's long-term debenture bonds.
said bonds will not be issued as money market instruments. Such being
the case, and considering that the purposes of Presidential Decree No. We conclude that Picop was properly held liable for the thirty-five
1154, as can be gleaned from Letter of Instruction No. 340, dated percent (35%) transaction tax due in respect of interest payments on
November 21, 1975, are (a) to regulate money market transactions and its money market borrowings.
(b) to ensure the collection of the tax on interest derived from money
market transactions by imposing a withholding tax thereon, said bonds At the same time, we agree with the Court of Appeals that the
do not come within the purview of the "commercial papers" intended to transaction tax may be levied only in respect of the interest earnings of
be subjected to the 35% transaction tax prescribed in Presidential Picop's money market lenders accruing after P.D. No. 1154 went into
Decree No. 1154, as implemented by Revenue Regulations No. 7-77. effect, and not in respect of all the 1977 interest earnings of such
(See Section 2 of said Regulation) Accordingly, PICOP is not subject lenders. The Court of Appeals pointed out that:
to 35% transaction tax on its issues of the aforesaid bonds. However,
those investing in said bonds should be made aware of the fact that the PICOP, however contends that even if the tax has to be paid, it should
transaction tax is not being imposed on the issuer of said bonds by be imposed only for the interests earned after 20 September 1977 when
printing or stamping thereon, in bold letters, the following statement: PD 1154 creating the tax became effective. We find merit in this
"ISSUER NOT SUBJECT TO TRANSACTION TAX UNDER P.D. 1154. contention. It appears that the tax was levied on interest earnings from
BONDHOLDER SHOULD DECLARE INTEREST EARNING FOR January to October, 1977. However, as found by the lower court, PD
INCOME TAX." 11 (Emphases supplied) 1154 was published in the Official Gazette only on 5 September 1977,
and became effective only fifteen (15) days after the publication, or on
20 September 1977, no other effectivity date having been provided by
the PD. Based on the Worksheet prepared by the Commissioner's In addition to the above administrative penalties, the criminal and civil
office, the interests earned from 20 September to October 1977 was penalties as provided for under Section 337 of the Tax Code of 1977
P10,224,410.03. Thirty-five (35%) per cent of this is P3,578,543.51 shall be imposed for violation of any provision of Presidential Decree
which is all PICOP should pay as transaction tax. 13 (Emphasis No. 1154. 15 (Emphases supplied)
supplied)
The 1977 Tax Code itself, in Section 326 in relation to Section 4 of the
P.D. No. 1154 is not, in other words, to be given retroactive effect by same Code, invoked by the Secretary of Finance in issuing Revenue
imposing the thirty-five percent (35%) transaction tax in respect of Regulation 7-77, set out, in comprehensive terms, the rule-making
interest earnings which accrued before the effectivity date of P.D. No. authority of the Secretary of Finance:
1154, there being nothing in the statute to suggest that the legislative
authority intended to bring about such retroactive imposition of the tax. Sec. 326. Authority of Secretary of Finance to Promulgate Rules and
Regulations. — The Secretary of Finance, upon recommendation of the
(2) Whether Picop is liable Commissioner of Internal Revenue, shall promulgate all needful rules
for interest and surcharge and regulations for the effective enforcement of the provisions of this
on unpaid transaction tax. Code. (Emphasis supplied)

With respect to the transaction tax due, the CIR prays that Picop be Section 4 of the same Code contains a list of subjects or areas to be
held liable for a twenty-five percent (25%) surcharge and for interest at dealt with by the Secretary of Finance through the medium of an
the rate of fourteen percent (14%) per annum from the date prescribed exercise of his quasi-legislative or rule-making authority. This list,
for its payment. In so praying, the CIR relies upon Section 10 of however, while it purports to be open-ended, does not include the
Revenue Regulation 7-77 dated 3 June 1977, 14 issued by the imposition of administrative or civil penalties such as the payment of
Secretary of Finance. This Section reads: amounts additional to the tax due. Thus, in order that it may be held to
be legally effective in respect of Picop in the present case, Section 10
Sec. 10. Penalties. — Where the amount shown by the taxpayer to be of Revenue Regulation 7-77 must embody or rest upon some provision
due on its return or part of such payment is not paid on or before the in the Tax Code itself which imposes surcharge and penalty interest for
date prescribed for its payment, the amount of the tax shall be failure to make a transaction tax payment when due.
increased by twenty-five (25%) per centum, the increment to be a part
of the tax and the entire amount shall be subject to interest at the rate P.D. No. 1154 did not itself impose, nor did it expressly authorize the
of fourteen (14%) per centum per annum from the date prescribed for imposition of, a surcharge and penalty interest in case of failure to pay
its payment. the thirty-five percent (35%) transaction tax when due. Neither did
Section 210 (b) of the 1977 Tax Code which re-enacted Section 195-C
In the case of willful neglect to file the return within the period inserted into the Tax Code by P.D. No. 1154.
prescribed herein or in case a false or fraudulent return is willfully
made, there shall be added to the tax or to the deficiency tax in case The CIR, both in its petition before the Court of Appeals and its Petition
any payment has been made on the basis of such return before the in the instant case, points to Section 51 (e) of the 1977 Tax Code as its
discovery of the falsity or fraud, a surcharge of fifty (50%) per centum source of authority for assessing a surcharge and penalty interest in
of its amount. The amount so added to any tax shall be collected at the respect of the thirty-five percent (35%) transaction tax due from Picop.
same time and in the same manner and as part of the tax unless the This Section needs to be quoted in extenso:
tax has been paid before the discovery of the falsity or fraud, in which
case the amount so added shall be collected in the same manner as Sec. 51. Payment and Assessment of Income Tax. —
the tax.
(c) Definition of deficiency. — As used in this Chapter in respect of a
tax imposed by this Title, the term "deficiency" means:
(1) The amount by which the tax imposed by this Title exceeds the Section 72 of the 1977 Tax Code referred to in Section 51 (e) (2) above,
amount shown as the tax by the taxpayer upon his return; but the provides:
amount so shown on the return shall first be increased by the amounts
previously assessed (or collected without assessment) as a deficiency, Sec. 72. Surcharges for failure to render returns and for rendering false
and decreased by the amount previously abated, credited, returned, or and fraudulent returns. — In case of willful neglect to file the return or
otherwise in respect of such tax; . . . list required by this Title within the time prescribed by law, or in case a
false or fraudulent return or list is wilfully made, the Commissioner of
xxx xxx xxx Internal Revenue shall add to the tax or to the deficiency tax, in case
any payment has been made on the basis of such return before the
(e) Additions to the tax in case of non-payment. — discovery of the falsity or fraud, as surcharge of fifty per centum of the
amount of such tax or deficiency tax. In case of any failure to make and
(1) Tax shown on the return. — Where the amount determined by the file a return or list within the time prescribed by law or by the
taxpayer as the tax imposed by this Title or any installment thereof, or Commissioner or other Internal Revenue Officer, not due to willful
any part of such amount or installment is not paid on or before the date neglect, the Commissioner of Internal Revenue shall add to the
prescribed for its payment, there shall be collected as a part of the tax, tax twenty-five per centum of its amount, except that, when a return is
interest upon such unpaid amount at the rate of fourteen per centum voluntarily and without notice from the Commissioner or other officer
per annum from the date prescribed for its payment until it is filed after such time, and it is shown that the failure to file it was due to
paid: Provided, That the maximum amount that may be collected as a reasonable cause, no such addition shall be made to the tax. The
interest on deficiency shall in no case exceed the amount amount so added to any tax shall be collected at the same time, in the
corresponding to a period of three years, the present provisions same manner and as part of the tax unless the tax has been paid before
regarding prescription to the contrary notwithstanding. the discovery of the neglect, falsity, or fraud, in which case the amount
so added shall be collected in the same manner as the tax. (Emphases
(2) Deficiency. — Where a deficiency, or any interest assessed in supplied)
connection therewith under paragraph (d) of this section, or any
addition to the taxes provided for in Section seventy-two of this Code is It will be seen that Section 51 (c) (1) and (e) (1) and (3), of the 1977
not paid in full within thirty days from the date of notice and demand Tax Code, authorize the imposition of surcharge and interest only in
from the Commissioner of Internal Revenue, there shall be collected respect of a "tax imposed by this Title," that is to say, Title II on "Income
upon the unpaid amount as part of the tax, interest at the rate of Tax." It will also be seen that Section 72 of the 1977 Tax Code imposes
fourteen per centum per annum from the date of such notice and a surcharge only in case of failure to file a return or list "required by this
demand until it is paid: Provided, That the maximum amount that may Title," that is, Title II on "Income Tax." The thirty-five percent (35%)
be collected as interest on deficiency shall in no case exceed the transaction tax is, however, imposed in the 1977 Tax Code by Section
amount corresponding to a period of three years, the present provisions 210 (b) thereof which Section is embraced in Title V on "Taxes on
regarding prescription to the contrary notwithstanding. Business" of that Code. Thus, while the thirty-five percent (35%)
transaction tax is in truth a tax imposed on interest income earned by
(3) Surcharge. — If any amount of tax included in the notice and lenders or creditors purchasing commercial paper on the money
demand from the Commissioner of Internal Revenue is not paid in full market, the relevant provisions, i.e., Section 210 (b), were not inserted
within thirty days after such notice and demand, there shall be collected in Title II of the 1977 Tax Code. The end result is that the thirty-five
in addition to the interest prescribed herein and in paragraph (d) above percent (35%) transaction tax is not one of the taxes in respect of which
and as part of the tax a surcharge of five per centum of the amount of Section 51 (e) authorized the imposition of surcharge and interest and
tax unpaid. (Emphases supplied) Section 72 the imposition of a fraud surcharge.
It is not without reluctance that we reach the above conclusion on the xxx xxx xxx
basis of what may well have been an inadvertent error in legislative
draftsmanship, a type of error common enough during the period of (c) the penalties imposed hereunder shall form part of the tax and the
Martial Law in our country. Nevertheless, we are compelled to adopt entire amount shall be subject to the interest prescribed in Section 249.
this conclusion. We consider that the authority to impose what the
present Tax Code calls (in Section 248) civil penalties consisting of Sec. 249. Interest. — (a) In General. — There shall be assessed and
additions to the tax due, must be expressly given in the enabling collected on any unpaid amount of tax, interest at the rate of twenty
statute, in language too clear to be mistaken. The grant of that authority percent (20%) per annum or such higher rate as may be prescribed by
is not lightly to be assumed to have been made to administrative regulations, from the date prescribed for payment until the amount is
officials, even to one as highly placed as the Secretary of Finance. fully paid. . . . (Emphases supplied)

The state of the present law tends to reinforce our conclusion that In other words, Section 247 (a) of the current NIRC supplies what did
Section 51 (c) and (e) of the 1977 Tax Code did not authorize the not exist back in 1977 when Picop's liability for the thirty-five percent
imposition of a surcharge and penalty interest for failure to pay the (35%) transaction tax became fixed. We do not believe we can fill that
thirty-five percent (35%) transaction tax imposed under Section 210 (b) legislative lacuna by judicial fiat. There is nothing to suggest that
of the same Code. The corresponding provision in the current Tax Section 247 (a) of the present Tax Code, which was inserted in 1985,
Code very clearly embraces failure to pay all taxes imposed in the Tax was intended to be given retroactive application by the legislative
Code, without any regard to the Title of the Code where provisions authority. 16
imposing particular taxes are textually located. Section 247 (a) of the
NIRC, as amended, reads: (3) Whether Picop is Liable
for Documentary and
Title X Science Stamp Taxes.

Statutory Offenses and Penalties As noted earlier, Picop issued sometime in 1977 long-term
subordinated convertible debenture bonds with an aggregate face
Chapter I value of P100,000,000.00. Picop stated, and this was not disputed by
the CIR, that the proceeds of the debenture bonds were in fact utilized
Additions to the Tax to finance the BOI-registered operations of Picop. The CIR assessed
documentary and science stamp taxes, amounting to P300,000.00, on
Sec. 247. General Provisions. — (a) The additions to the tax or the issuance of Picop's debenture bonds. It is claimed by Picop that its
deficiency tax prescribed in this Chapter shall apply to all taxes, fees tax exemption — "exemption from all taxes under the National Internal
and charges imposed in this Code. The amount so added to the tax Revenue Code, except income tax" on a declining basis over a certain
shall be collected at the same time, in the same manner and as part of period of time — includes exemption from the documentary and
the tax. . . . science stamp taxes imposed under the NIRC.

Sec. 248. Civil Penalties. — (a) There shall be imposed, in addition to The CIR, upon the other hand, stresses that the tax exemption under
the tax required to be paid, penalty equivalent to twenty-five percent the Investment Incentives Act may be granted or recognized only to the
(25%) of the amount due, in the following cases: extent that the claimant Picop was engaged in registered
operations, i.e., operations forming part of its integrated pulp and paper
xxx xxx xxx project. 17 The borrowing of funds from the public, in the submission of
the CIR, was not an activity included in Picop's registered operations.
(3) failure to pay the tax within the time prescribed for its payment; or The CTA adopted the view of the CIR and held that "the issuance of
convertible debenture bonds [was] not synonymous [with] the It remains only to note that after commencement of the present litigation
manufactur[ing] operations of an integrated pulp and paper mill." 18 before the CTA, the BIR took the position that the tax exemption
granted by R.A. No. 5186, as amended, does include exemption from
The Court of Appeals took a less rigid view of the ambit of the tax documentary stamp taxes on transactions entered into by BOI-
exemption granted to registered pioneer enterprises. Said the Court of registered enterprises. BIR Ruling No. 088, dated 28 April 1989, for
Appeals: instance, held that a registered preferred pioneer enterprise engaged
in the manufacture of integrated circuits, magnetic heads, printed circuit
. . . PICOP's explanation that the debenture bonds were issued to boards, etc., is exempt from the payment of documentary stamp taxes.
finance its registered operation is logical and is unrebutted. We are The Commissioner said:
aware that tax exemptions must be applied strictly against the
beneficiary in order to deter their abuse. It would indeed be altogether You now request a ruling that as a preferred pioneer enterprise, you
a different matter if there is a showing that the issuance of the are exempt from the payment of Documentary Stamp Tax (DST).
debenture bonds had no bearing whatsoever on the registered
operations PICOP and that they were issued in connection with a totally In reply, please be informed that your request is hereby granted.
different business undertaking of PICOP other than its registered Pursuant to Section 46 (a) of Presidential Decree No. 1789, pioneer
operation. There is, however, a dearth of evidence in this regard. It enterprises registered with the BOI are exempt from all taxes under the
cannot be denied that PICOP needed funds for its operations. One of National Internal Revenue Code, except from all taxes under the
the means it used to raise said funds was to issue debenture National Internal Revenue Code, except income tax, from the date the
bonds. Since the money raised thereby was to be used in its registered area of investment is included in the Investment Priorities Plan to the
operation, PICOP should enjoy the incentives granted to it by R.A. following extent:
5186, one of which is the exemption from payment of all taxes under
the National Internal Revenue Code, except income taxes, otherwise xxx xxx xxx
the purpose of the incentives would be defeated. Documentary and
science stamp taxes on debenture bonds are certainly not income Accordingly, your company is exempt from the payment of
taxes. 19 (Emphasis supplied) documentary stamp tax to the extent of the percentage aforestated on
transactions connected with the registered business activity. (BIR
Tax exemptions are, to be sure, to be "strictly construed," that is, they Ruling No. 111-81) However, if said transactions conducted by you
are not to be extended beyond the ordinary and reasonable intendment require the execution of a taxable document with other parties, said
of the language actually used by the legislative authority in granting the parties who are not exempt shall be the one directly liable for the tax.
exemption. The issuance of debenture bonds is certainly conceptually (Sec. 173, Tax Code, as amended; BIR Ruling No. 236-87) In other
distinct from pulping and paper manufacturing operations. But no one words, said parties shall be liable to the same percentage
contends that issuance of bonds was a principal or regular business corresponding to your tax exemption. (Emphasis supplied)
activity of Picop; only banks or other financial institutions are in the
regular business of raising money by issuing bonds or other Similarly, in BIR Ruling No. 013, dated 6 February 1989, the
instruments to the general public. We consider that the actual Commissioner held that a registered pioneer enterprise producing
dedication of the proceeds of the bonds to the carrying out of Picop's polyester filament yarn was entitled to exemption "from the
registered operations constituted a sufficient nexus with such documentary stamp tax on [its] sale of real property in Makati up to
registered operations so as to exempt Picop from stamp taxes December 31, 1989." It appears clear to the Court that the CIR,
ordinarily imposed upon or in connection with issuance of such bonds. administratively at least, no longer insists on the position it originally
We agree, therefore, with the Court of Appeals on this matter that the took in the instant case before the CTA.
CTA and the CIR had erred in rejecting Picop's claim for exemption
from stamp taxes. II
(1) Whether Picop is entitled Thus, the general rule is that interest expenses are deductible against
to deduct against current gross income and this certainly includes interest paid under loans
income interest payments incurred in connection with the carrying on of the business of the
on loans for the purchase taxpayer. 20 In the instant case, the CIR does not dispute that the
of machinery and equipment. interest payments were made by Picop on loans incurred in connection
with the carrying on of the registered operations of Picop, i.e., the
In 1969, 1972 and 1977, Picop obtained loans from foreign creditors in financing of the purchase of machinery and equipment actually used in
order to finance the purchase of machinery and equipment needed for the registered operations of Picop. Neither does the CIR deny that such
its operations. In its 1977 Income Tax Return, Picop claimed interest interest payments were legally due and demandable under the terms
payments made in 1977, amounting to P42,840,131.00, on these loans of such loans, and in fact paid by Picop during the tax year 1977.
as a deduction from its 1977 gross income.
The CIR has been unable to point to any provision of the 1977 Tax
The CIR disallowed this deduction upon the ground that, because the Code or any other Statute that requires the disallowance of the interest
loans had been incurred for the purchase of machinery and equipment, payments made by Picop. The CIR invokes Section 79 of Revenue
the interest payments on those loans should have been capitalized Regulations No. 2 as amended which reads as follows:
instead and claimed as a depreciation deduction taking into account
the adjusted basis of the machinery and equipment (original acquisition Sec. 79. Interest on Capital. — Interest calculated for cost-keeping or
cost plus interest charges) over the useful life of such assets. other purposes on account of capital or surplus invested in the
business, which does not represent a charge arising under an interest-
Both the CTA and the Court of Appeals sustained the position of Picop bearing obligation, is not allowable deduction from gross income.
and held that the interest deduction claimed by Picop was proper and (Emphases supplied)
allowable. In the instant Petition, the CIR insists on its original position.
We read the above provision of Revenue Regulations No. 2 as referring
We begin by noting that interest payments on loans incurred by a to so called "theoretical interest," that is to say, interest "calculated" or
taxpayer (whether BOI-registered or not) are allowed by the NIRC as computed (and not incurred or paid) for the purpose of determining the
deductions against the taxpayer's gross income. Section 30 of the 1977 "opportunity cost" of investing funds in a given business. Such
Tax Code provided as follows: "theoretical" or imputed interest does not arise from a legally
demandable interest-bearing obligation incurred by the taxpayer who
Sec. 30. Deduction from Gross Income. — The following may be however wishes to find out, e.g., whether he would have been better off
deducted from gross income: by lending out his funds and earning interest rather than investing such
funds in his business. One thing that Section 79 quoted above makes
(a) Expenses: clear is that interest which does constitute a charge arising under an
interest-bearing obligation is an allowable deduction from gross
xxx xxx xxx income.
(b) Interest: It is claimed by the CIR that Section 79 of Revenue Regulations No. 2
was "patterned after" paragraph 1.266-1 (b), entitled "Taxes and
(1) In general. — The amount of interest paid within the taxable year Carrying Charges Chargeable to Capital Account and Treated as
on indebtedness, except on indebtedness incurred or continued to Capital Items" of the U.S. Income Tax Regulations, which paragraph
purchase or carry obligations the interest upon which is exempt from reads as follows:
taxation as income under this Title: . . . (Emphasis supplied)
(B) Taxes and Carrying Charges. — The items thus chargeable to payments against its gross income, the taxpayer cannot at the same
capital accounts are — time capitalize the interest payments. In other words, the taxpayer
is not entitled to both the deduction from gross income and the adjusted
(11) In the case of real property, whether improved or unimproved and (increased) basis for determining gain or loss and the allowable
whether productive or nonproductive. depreciation charge. The U.S. Internal Revenue Code does not prohibit
the deduction of interest on a loan obtained for purchasing machinery
(a) Interest on a loan (but not theoretical interest of a taxpayer using his and equipment against gross income, unless the taxpayer has also or
own funds). 21 previously capitalized the same interest payments and thereby
adjusted the cost basis of such assets.
The truncated excerpt of the U.S. Income Tax Regulations quoted by
the CIR needs to be related to the relevant provisions of the U.S. We have already noted that our 1977 NIRC does not prohibit the
Internal Revenue Code, which provisions deal with the general topic of deduction of interest on a loan incurred for acquiring machinery and
adjusted basis for determining allowable gain or loss on sales or equipment. Neither does our 1977 NIRC compel the capitalization of
exchanges of property and allowable depreciation and depletion of interest payments on such a loan. The 1977 Tax Code is simply silent
capital assets of the taxpayer: on a taxpayer's right to elect one or the other tax treatment of such
interest payments. Accordingly, the general rule that interest payments
Present Rule. The Internal Revenue Code, and the Regulations on a legally demandable loan are deductible from gross income must
promulgated thereunder provide that "No deduction shall be allowed for be applied.
amounts paid or accrued for such taxes and carrying charges as,
under regulations prescribed by the Secretary or his delegate, are The CIR argues finally that to allow Picop to deduct its interest
chargeable to capital account with respect to property, if the taxpayer payments against its gross income would be to encourage fraudulent
elects, in accordance with such regulations, to treat such taxes claims to double deductions from gross income:
or charges as so chargeable."
[t]o allow a deduction of incidental expense/cost incurred in the
At the same time, under the adjustment of basis provisions which have purchase of fixed asset in the year it was incurred would invite tax
just been discussed, it is provided that adjustment shall be made for all evasion through fraudulent application of double deductions from gross
"expenditures, receipts, losses, or other items" properly chargeable to income. 23 (Emphases supplied)
a capital account, thus including taxes and carrying charges;
however, an exception exists, in which event such adjustment to the The Court is not persuaded. So far as the records of the instant cases
capital account is not made, with respect to taxes and carrying charges show, Picop has not claimed to be entitled to double deduction of its
which the taxpayer has not elected to capitalize but for which a 1977 interest payments. The CIR has neither alleged nor proved that
deduction instead has been taken. 22 (Emphasis supplied) Picop had previously adjusted its cost basis for the machinery and
equipment purchased with the loan proceeds by capitalizing the interest
The "carrying charges" which may be capitalized under the above payments here involved. The Court will not assume that the CIR would
quoted provisions of the U.S. Internal Revenue Code include, as the be unable or unwilling to disallow "a double deduction" should Picop,
CIR has pointed out, interest on a loan "(but not theoretical interest of having deducted its interest cost from its gross income, also attempt
a taxpayer using his own funds)." What the CIR failed to point out is subsequently to adjust upward the cost basis of the machinery and
that such "carrying charges" may, at the election of the taxpayer, either equipment purchased and claim, e.g., increased deductions for
be (a) capitalized in which case the cost basis of the capital assets, depreciation.
e.g., machinery and equipment, will be adjusted by adding the amount
of such interest payments or alternatively, be (b) deducted from gross
income of the taxpayer. Should the taxpayer elect to deduct the interest
We conclude that the CTA and the Court of Appeals did not err in In claiming such deduction, Picop relies on section 7 (c) of R.A. No.
allowing the deductions of Picop's 1977 interest payments on its loans 5186 which provides as follows:
for capital equipment against its gross income for 1977.
Sec. 7. Incentives to Registered Enterprise. — A registered
(2) Whether Picop is entitled enterprise, to the extent engaged in a preferred area of investment,
to deduct against current shall be granted the following incentive benefits:
income net operating losses
incurred by Rustan Pulp xxx xxx xxx
and Paper Mills, Inc.
(c) Net Operating Loss Carry-over. — A net operating loss incurred in
On 18 January 1977, Picop entered into a merger agreement with the any of the first ten years of operations may be carried over as a
Rustan Pulp and Paper Mills, Inc. ("RPPM") and Rustan Manufacturing deduction from taxable income for the six years immediately following
Corporation ("RMC"). Under this agreement, the rights, properties, the year of such loss. The entire amount of the loss shall be carried
privileges, powers and franchises of RPPM and RMC were to be over to the first of the six taxable years following the loss, and any
transferred, assigned and conveyed to Picop as the surviving portion of such loss which exceeds the taxable income of such first year
corporation. The entire subscribed and outstanding capital stock of shall be deducted in like manner from the taxable income of the next
RPPM and RMC would be exchanged for 2,891,476 fully paid up Class remaining five years. The net operating loss shall be computed in
"A" common stock of Picop (with a par value of P10.00) and 149,848 accordance with the provisions of the National Internal Revenue Code,
shares of preferred stock of Picop (with a par value of P10.00), to be any provision of this Act to the contrary notwithstanding, except that
issued by Picop, the result being that Picop would wholly own both income not taxable either in whole or in part under this or other laws
RPPM and RMC while the stockholders of RPPM and RMC would join shall be included in gross income. (Emphasis supplied)
the ranks of Picop's shareholders. In addition, Picop paid off the
obligations of RPPM to the Development Bank of the Philippines Picop had secured a letter-opinion from the BOI dated 21 February
("DBP") in the amount of P68,240,340.00, by issuing 6,824,034 shares 1977 — that is, after the date of the agreement of merger but before
of preferred stock (with a par value of P10.00) to the DBP. The merger the merger became effective — relating to the deductibility of the
agreement was approved in 1977 by the creditors and stockholders of previous losses of RPPM under Section 7 (c) of R.A. No. 5186 as
Picop, RPPM and RMC and by the Securities and Exchange amended. The pertinent portions of this BOI opinion, signed by BOI
Commission. Thereupon, on 30 November 1977, apparently the Governor Cesar Lanuza, read as follows:
effective date of merger, RPPM and RMC were dissolved. The Board
of Investments approved the merger agreement on 12 January 1978. 2) PICOP will not be allowed to carry over the losses of Rustan prior to
the legal dissolution of the latter because at that time the two (2)
It appears that RPPM and RMC were, like Picop, BOI-registered companies still had separate legal personalities;
companies. Immediately before merger effective date, RPPM had over
preceding years accumulated losses in the total amount of 3) After BOI approval of the merger, PICOP can no longer apply for the
P81,159,904.00. In its 1977 Income Tax Return, Picop claimed registration of the registered capacity of Rustan because with the
P44,196,106.00 of RPPM's accumulated losses as a deduction against approved merger, such registered capacity of Rustan transferred to
Picop's 1977 gross income. 24 PICOP will have the same registration date as that of Rustan. In this
case, the previous losses of Rustan may be carried over by PICOP,
Upon the other hand, even before the effective date of merger, on 30 because with the merger, PICOP assumes all the rights and obligations
August 1977, Picop sold all the outstanding shares of RMC stock to of Rustan subject, however, to the period prescribed for carrying over
San Miguel Corporation for the sum of P38,900,000.00, and reported a of such
gain of P9,294,849.00 from this transaction. 25 losses. 26 (Emphasis supplied)
Curiously enough, Picop did not also seek a ruling on this matter, No. 5186 was only proper." (pp. 38-43, Rollo of SP No.
clearly a matter of tax law, from the Bureau of Internal Revenue. Picop 20070) 29 (Emphasis supplied)
chose to rely solely on the BOI letter-opinion.
In respect of the above underscored portion of the CTA decision, we
The CIR disallowed all the deductions claimed on the basis of RPPM's must note that the CTA in fact overlooked the statement made by
losses, apparently on two (2) grounds. Firstly, the previous losses were petitioner's counsel before the CTA that:
incurred by "another taxpayer," RPPM, and not by Picop in connection
with Picop's own registered operations. The CIR took the view that Among the attractions of the merger to Picop was the accumulated net
Picop, RPPM and RMC were merged into one (1) corporate personality operating loss carry-over of RMC that it might possibly use to relieve it
only on 12 January 1978, upon approval of the merger agreement by (Picop) from its income taxes, under Section 7 (c) of R.A. 5186. Said
the BOI. Thus, during the taxable year 1977, Picop on the one hand section provides:
and RPPM and RMC on the other, still had their separate juridical
personalities. Secondly, the CIR alleged that these losses had been xxx xxx xxx
incurred by RPPM "from the borrowing of funds" and not from carrying
out of RPPM's registered operations. We focus on the first ground. 27 With this benefit in mind, Picop addressed three (3) questions to the
BOI in a letter dated November 25, 1976. The BOI replied on February
The CTA upheld the deduction claimed by Picop; its reasoning, 21, 1977 directly answering the three (3) queries. 30 (Emphasis
however, is less than crystal clear, especially in respect of its view of supplied)
what the U.S. tax law was on this matter. In any event, the CTA
apparently fell back on the BOI opinion of 21 February 1977 referred to The size of RPPM's accumulated losses as of the date of the merger
above. The CTA said: — more than P81,000,000.00 — must have constituted a powerful
attraction indeed for Picop.
Respondent further averred that the incentives granted under Section
7 of R.A. No. 5186 shall be available only to the extent in which they The Court of Appeals followed the result reached by the CTA. The
are engaged in registered operations, citing Section 1 of Rule IX of the Court of Appeals, much like the CTA, concluded that since RPPM was
Basic Rules and Regulations to Implement the Intent and Provisions of dissolved on 30 November 1977, its accumulated losses were
the Investment Incentives Act, R.A. No. 5186. appropriately carried over by Picop in the latter's 1977 Income Tax
Return "because by that time RPPMI and Picop were no longer
We disagree with respondent. The purpose of the merger was to separate and different taxpayers." 31
rationalize the container board industry and not to take advantage of
the net losses incurred by RPPMI prior to the stock swap. Thus, when After prolonged consideration and analysis of this matter, the Court is
stock of a corporation is purchased in order to take advantage of the unable to agree with the CTA and Court of Appeals on the deductibility
corporation's net operating loss incurred in years prior to the purchase, of RPPM's accumulated losses against Picop's 1977 gross income.
the corporation thereafter entering into a trade or business different
from that in which it was previously engaged, the net operating loss It is important to note at the outset that in our jurisdiction, the ordinary
carry-over may be entirely lost. [IRC (1954), Sec. 382(a), Vol. 5, rule — that is, the rule applicable in respect of
Mertens, Law of Federal Income Taxation, Chap. 29.11a, p. corporations not registered with the BOI as a preferred pioneer
103]. 28 Furthermore, once the BOI approved the merger agreement, enterprise — is that net operating losses cannot be carried over. Under
the registered capacity of Rustan shall be transferred to PICOP, and our Tax Code, both in 1977 and at present, losses may be deducted
the previous losses of Rustan may be carried over by PICOP by from gross income only if such losses were actually sustained in the
operation of law. [BOI ruling dated February 21, 1977 (Exh. J-1)] It is same year that they are deducted or charged off. Section 30 of the 1977
clear therefrom, that the deduction availed of under Section 7(c) of R.A. Tax Code provides:
Sec. 30. Deductions from Gross Income. — In computing net income, . . . . If subsequent to its occurrence, however, a taxpayer first
there shall be allowed as deduction — ascertains the amount of a loss sustained during a prior taxable
year which has not been deducted from gross income, he may
xxx xxx xxx render an amended return for such preceding taxable year including
such amount of loss in the deduction from gross income and may in
(d) Losses: proper cases file a claim for refund of the excess paid by reason of the
failure to deduct such loss in the original return. A loss from theft or
(1) By Individuals. — In the case of an individual, losses actually embezzlement occurring in one year and discovered in another is
sustained during the taxable year and not compensated for by an ordinarily deductible for the year in which sustained. (Emphases
insurance or otherwise — supplied)

(A) If incurred in trade or business; It is thus clear that under our law, and outside the special realm of BOI-
registered enterprises, there is no such thing as a carry-over of net
xxx xxx xxx operating loss. To the contrary, losses must be deducted against
current income in the taxable year when such losses were incurred.
(2) By Corporations. — In a case of a corporation, all losses actually Moreover, such losses may be charged off only against income earned
sustained and charged off within the taxable year and not compensated in the same taxable year when the losses were incurred.
for by insurance or otherwise.
Thus it is that R.A. No. 5186 introduced the carry-over of net operating
(3) By Non-resident Aliens or Foreign Corporations. — In the case of a losses as a very special incentive to be granted only to registered
non-resident alien individual or a foreign corporation, the losses pioneer enterprises and only with respect to their registered operations.
deductible are those actually sustained during the year incurred in The statutory purpose here may be seen to be the encouragement of
business or trade conducted within the Philippines, . . . 32 (Emphasis the establishment and continued operation of pioneer industries by
supplied) allowing the registered enterprise to accumulate its operating losses
which may be expected during the early years of the enterprise and to
Section 76 of the Philippine Income Tax Regulations (Revenue permit the enterprise to offset such losses against income earned by it
Regulation No. 2, as amended) is even more explicit and detailed: in later years after successful establishment and regular operations. To
promote its economic development goals, the Republic foregoes or
Sec. 76. When charges are deductible. — Each year's return, so far as defers taxing the income of the pioneer enterprise until after that
practicable, both as to gross income and deductions therefrom should enterprise has recovered or offset its earlier losses. We consider that
be complete in itself, and taxpayers are expected to make every the statutory purpose can be served only if the accumulated operating
reasonable effort to ascertain the facts necessary to make a correct losses are carried over and charged off against income subsequently
return. The expenses, liabilities, or deficit of one year cannot be used earned and accumulated by the same enterprise engaged in the same
to reduce the income of a subsequent year. A taxpayer has the right to registered operations.
deduct all authorized allowances and it follows that if he does not within
any year deduct certain of his expenses, losses, interests, taxes, or In the instant case, to allow the deduction claimed by Picop would be
other charges, to permit one corporation or enterprise, Picop, to benefit from the
he can not deduct them from the income of the next or any succeeding operating losses accumulated by another corporation or enterprise,
year. . . . RPPM. RPPM far from benefiting from the tax incentive granted by the
BOI statute, in fact gave up the struggle and went out of existence and
xxx xxx xxx its former stockholders joined the much larger group of Picop's
stockholders. To grant Picop's claimed deduction would be to permit
Picop to shelter its otherwise taxable income (an objective which Picop loss carry-backs. Indeed, as already noted, our tax law expressly
had from the very beginning) which had not been earned by the rejects the very notion of loss carry-overs and carry-backs.
registered enterprise which had suffered the accumulated losses. In
effect, to grant Picop's claimed deduction would be to permit Picop to We conclude that the deduction claimed by Picop in the amount of
purchase a tax deduction and RPPM to peddle its accumulated P44,196,106.00 in its 1977 Income Tax Return must be disallowed.
operating losses. Under the CTA and Court of Appeals decisions, Picop
would benefit by immunizing P44,196,106.00 of its income from (3) Whether Picop is entitled
taxation thereof although Picop had not run the risks and incurred the to deduct against current
losses which had been encountered and suffered by RPPM. income certain claimed
Conversely, the income that would be shielded from taxation is not financial guarantee expenses.
income that was, after much effort, eventually generated by the same
registered operations which earlier had sustained losses. We consider In its Income Tax Return for 1977, Picop also claimed a deduction in
and so hold that there is nothing in Section 7 (c) of R.A. No. 5186 which the amount of P1,237,421.00 as financial guarantee expenses.
either requires or permits such a result. Indeed, that result makes non-
sense of the legislative purpose which may be seen clearly to be This deduction is said to relate to chattel and real estate mortgages
projected by Section 7 (c), R.A. No. 5186. required from Picop by the Philippine National Bank ("PNB") and DBP
as guarantors of loans incurred by Picop from foreign creditors.
The CTA and the Court of Appeals allowed the offsetting of RPPM's According to Picop, the claimed deduction represents registration fees
accumulated operating losses against Picop's 1977 gross income, and other expenses incidental to registration of mortgages in favor of
basically because towards the end of the taxable year 1977, upon the DBP and PNB.
arrival of the effective date of merger, only one (1) corporation, Picop,
remained. The losses suffered by RPPM's registered operations and In support of this claimed deduction, Picop allegedly showed its own
the gross income generated by Picop's own registered operations now vouchers to BIR Examiners to prove disbursements to the Register of
came under one and the same corporate roof. We consider that this Deeds of Tandag, Surigao del Sur, of particular amounts. In the
circumstance relates much more to form than to substance. We do not proceedings before the CTA, however, Picop did not submit in evidence
believe that that single purely technical factor is enough to authorize such vouchers and instead presented one of its employees to testify
and justify the deduction claimed by Picop. Picop's claim for deduction that the amount claimed had been disbursed for the registration of
is not only bereft of statutory basis; it does violence to the legislative chattel and real estate mortgages.
intent which animates the tax incentive granted by Section 7 (c) of R.A.
No. 5186. In granting the extraordinary privilege and incentive of a net The CIR disallowed this claimed deduction upon the ground of
operating loss carry-over to BOI-registered pioneer enterprises, the insufficiency of evidence. This disallowance was sustained by the CTA
legislature could not have intended to require the Republic to forego tax and the Court of Appeals. The CTA said:
revenues in order to benefit a corporation which had run no risks and
suffered no losses, but had merely purchased another's losses. No records are available to support the abovementioned expenses.
The vouchers merely showed that the amounts were paid to the
Both the CTA and the Court of Appeals appeared much impressed not Register of Deeds and simply cash account. Without the supporting
only with corporate technicalities but also with the U.S. tax law on this papers such as the invoices or official receipts of the Register of Deeds,
matter. It should suffice, however, simply to note that in U.S. tax law, these vouchers standing alone cannot prove that the payments made
the availability to companies generally of operating loss carry-overs and were for the accrued expenses in question. The best evidence of
of operating loss carry-backs is expressly provided and regulated in payment is the official receipts issued by the Register of Deeds. The
great detail by statute. 33 In our jurisdiction, save for Section 7 (c) of testimony of petitioner's witness that the official receipts and cash
R.A. No. 5186, no statute recognizes or permits loss carry-overs and vouchers were shown to the Bureau of Internal Revenue will not suffice
if no records could be presented in court for proper marking and In its assessment for deficiency income tax for 1977, the CIR claimed
identification. 34 Emphasis supplied) that Picop had understated its sales by P2,391,644.00 and, upon the
other hand, overstated its cost of sales by P604,018.00. Thereupon,
The Court of Appeals added: the CIR added back both sums to Picop's net income figure per its own
return.
The mere testimony of a witness for PICOP and the cash vouchers do
not suffice to establish its claim that registration fees were paid to the The 1977 Income Tax Return of Picop set forth the following figures:
Register of Deeds for the registration of real estate and chattel
mortgages in favor of Development Bank of the Philippines and the Sales (per Picop's Income Tax Return):
Philippine National Bank as guarantors of PICOP's loans. The witness
could very well have been merely repeating what he was instructed to Paper P 537,656,719.00
say regardless of the truth, while the cash vouchers, which we do not
find on file, are not said to provide the necessary details regarding the Timber P 263,158,132.00
nature and purpose of the expenses reflected therein. PICOP should
have presented, through the guarantors, its owner's copy of the ———————
registered titles with the lien inscribed thereon as well as an official
receipt from the Register of Deeds evidencing payment of the Total Sales P 800,814,851.00
registration fee. 35 (Emphasis supplied)
============
We must support the CTA and the Court of Appeals in their foregoing
rulings. A taxpayer has the burden of proving entitlement to a claimed Upon the other hand, Picop's Books of Accounts reflected higher sales
deduction. 36 In the instant case, even Picop's own vouchers were not figures:
submitted in evidence and the BIR Examiners denied that such
vouchers and other documents had been exhibited to them. Moreover, Sales (per Picop's Books of Accounts):
cash vouchers can only confirm the fact of disbursement but not
necessarily the purpose thereof. 37 The best evidence that Picop should Paper P 537,656,719.00
have presented to support its claimed deduction were the invoices and
official receipts issued by the Register of Deeds. Picop not only failed Timber P 265,549,776.00
to present such documents; it also failed to explain the loss thereof,
———————
assuming they had existed before. 38 Under the best evidence
rule, 39 therefore, the testimony of Picop's employee was inadmissible
Total Sales P 803,206,495.00
and was in any case entitled to very little, if any, credence.
============
We consider that entitlement to Picop's claimed deduction of
P1,237,421.00 was not adequately shown and that such deduction The above figures thus show a discrepancy between the sales figures
must be disallowed.
reflected in Picop's Books of Accounts and the sales figures reported
in its 1977 Income Tax Return, amounting to: P2,391,644.00.
III
The CIR also contended that Picop's cost of sales set out in its 1977
(1) Whether Picop had understated
Income Tax Return, when compared with the cost figures in its Books
its sales and overstated its
of Accounts, was overstated:
cost of sales for 1977.
Cost of Sales time the proceeds were actually received. It was this rate at time of
(per Income Tax Return) P607,246,084.00 receipt of the proceeds that determined the amount of pesos credited
Cost of Sales by the Central Bank (through the agent banks) in favor of PICOP. These
(per Books of Accounts) P606,642,066.00 accumulated differences were averaged by the external auditors and
this was what was used at the year-end for income tax and other
——————— government-report purposes. (T.s.n., Oct. 17/85, pp. 20-25) 40

Discrepancy P 604,018.00 The above explanation, unfortunately, at least to the mind of the Court,
============ raises more questions than it resolves. Firstly, the explanation assumes
that all of Picop's sales were export sales for which U.S. dollars (or
Picop did not deny the existence of the above noted discrepancies. In other foreign exchange) were received. It also assumes that the
the proceedings before the CTA, Picop presented one of its officials to expenses summed up as "cost of sales" were all dollar expenses and
explain the foregoing discrepancies. That explanation is perhaps best that no peso expenses had been incurred. Picop's explanation further
presented in Picop's own words as set forth in its Memorandum before assumes that a substantial part of Picop's dollar proceeds for its export
this Court: sales were not actually surrendered to the domestic banking system
and seasonably converted into pesos; had all such dollar proceeds
. . . that the adjustment discussed in the testimony of the witness, been converted into pesos, then the peso figures could have been
represent the best and most objective method of determining in pesos simply added up to reflect the actual peso value of Picop's export sales.
the amount of the correct and actual export sales during the year. It was Picop offered no evidence in respect of these assumptions, no
this correct and actual export sales and costs of sales that were explanation why and how a "pre-determined fixed exchange rate" was
reflected in the income tax return and in the audited financial chosen at the beginning of the year and maintained throughout.
statements. These corrections did not result in realization of income Perhaps more importantly, Picop was unable to explain why its Books
and should not give rise to any deficiency tax. of Accounts did not pick up the same adjustments that Picop's External
Auditors were alleged to have made for purposes of Picop's Income
xxx xxx xxx Tax Return. Picop attempted to explain away the failure of its Books of
Accounts to reflect the same adjustments (no correcting entries,
What are the facts of this case on this matter? Why were adjustments apparently) simply by quoting a passage from a case where this Court
necessary at the year-end? refused to ascribe much probative value to the Books of Accounts of a
corporate taxpayer in a tax case. 41 What appears to have eluded
Because of PICOP's procedure of recording its export sales (reckoned Picop, however, is that its Books of Accounts, which are kept by its own
in U.S. dollars) on the basis of a fixed rate, day to day and month to employees and are prepared under its control and supervision, reflect
month, regardless of the actual exchange rate and without waiting what may be deemed to be admissions against interest in the instant
when the actual proceeds are received. In other words, PICOP case. For Picop's Books of Accounts precisely show higher sales
recorded its export sales at a pre-determined fixed exchange rate. That figures and lower cost of sales figures than Picop's Income Tax Return.
pre-determined rate was decided upon at the beginning of the year and
continued to be used throughout the year. It is insisted by Picop that its Auditors' adjustments simply present the
"best and most objective" method of reflecting in pesos the "correct
At the end of the year, the external auditors made an examination. In and ACTUAL export sales" 42 and that the adjustments or "corrections"
that examination, the auditors determined with accuracy the actual "did not result in realization of [additional] income and should not give
dollar proceeds of the export sales received. What exchange rate was rise to any deficiency tax." The correctness of this contention is not self-
used by the auditors to convert these actual dollar proceeds into evident. So far as the record of this case shows, Picop did not submit
Philippine pesos? They used the average of the differences between in evidence the aggregate amount of its U.S. dollar proceeds of its
(a) the recorded fixed exchange rate and (b) the exchange rate at the
export sales; neither did it show the Philippine pesos it had actually over liabilities as reflected in the corporation's balance sheet provided
received or been credited for such U.S. dollar proceeds. It is clear to such balance sheet has been prepared in accordance with generally
this Court that the testimonial evidence submitted by Picop fell far short accepted accounting principles employed in keeping the books of the
of demonstrating the correctness of its explanation. corporation. 43

Upon the other hand, the CIR has made out at least a prima facie case The adjusted net income of Picop for 1977, as will be seen below, is
that Picop had understated its sales and overstated its cost of sales as P48,687,355.00. Its net worth figure or total stockholders' equity as
set out in its Income Tax Return. For the CIR has a right to assume that reflected in its Audited Financial Statements for 1977 is
Picop's Books of Accounts speak the truth in this case since, as already P464,749,528.00. Since its adjusted net income for 1977 thus
noted, they embody what must appear to be admissions against Picop's exceeded ten percent (10%) of its net worth, Picop must be held liable
own interest. for the five percent (5%) corporate development tax in the amount of
P2,434,367.75.
Accordingly, we must affirm the findings of the Court of Appeals and
the CTA. Recapitulating, we hold:

(2) Whether Picop is liable for (1) Picop is liable for the thirty-five percent (35%) transaction tax in the
the corporate development amount of P3,578,543.51.
tax of five percent (5%)
of its income for 1977. (2) Picop is not liable for interest and surcharge on unpaid transaction
tax.
The five percent (5%) corporate development tax is an additional
corporate income tax imposed in Section 24 (e) of the 1977 Tax Code (3) Picop is exempt from payment of documentary and science stamp
which reads in relevant part as follows: taxes in the amount of P300,000.00 and the compromise penalty of
P300.00.
(e) Corporate development tax. — In addition to the tax imposed in
subsection (a) of this section, an additional tax in an amount equivalent (4) Picop is entitled to its claimed deduction of P42,840,131.00 for
to 5 per cent of the same taxable net income shall be paid by a domestic interest payments on loans for, among other things, the purchase of
or a resident foreign corporation; Provided, That this additional tax shall machinery and equipment.
be imposed only if the net income exceeds 10 per cent of the net worth,
in case of a domestic corporation, or net assets in the Philippines in (5) Picop's claimed deduction in the amount of P44,196,106.00 for the
case of a resident foreign corporation: . . . . operating losses previously incurred by RPPM, is disallowed for lack of
merit.
The additional corporate income tax imposed in this subsection shall
be collected and paid at the same time and in the same manner as the (6) Picop's claimed deduction for certain financial guarantee expenses
tax imposed in subsection (a) of this section. in the amount P1,237,421.00 is disallowed for failure adequately to
prove such expenses.
Since this five percent (5%) corporate development tax is
an income tax, Picop is not exempted from it under the provisions of (7) Picop has understated its sales by P2,391,644.00 and overstated
Section 8 (a) of R.A. No. 5186. its cost of sales by P604,018.00, for 1977.

For purposes of determining whether the net income of a corporation (8) Picop is liable for the corporate development tax of five percent (5%)
exceeds ten percent (10%) of its net worth, the term "net worth" means of its adjusted net income for 1977 in the amount of P2,434,367.75.
the stockholders' equity represented by the excess of the total assets
Considering conclusions nos. 4, 5, 6, 7 and 8, the Court is compelled Deficiency Income Tax P 16,560,216.00
to hold Picop liable for deficiency income tax for the year 1977
computed as follows: Add:

Deficiency Income Tax Five percent (5%) Corporate


Development Tax P 2,434,367.00
Net Income Per Return P 258,166.00
Total Deficiency Income Tax P 18,994,583.00
Add:
===========
Unallowable Deductions
Add:
(1) Deduction of net
operating losses Five percent (5%) surcharge 45 P 949,729.15
incurred by RPPM P 44,196,106.00
——————
(2) Unexplained financial
guarantee expenses P 1,237,421.00 Total Deficiency Income Tax

(3) Understatement of with surcharge P 19,944,312.15


Sales P 2,391,644.00
Add:
(4) Overstatement of
Cost of Sales P 604,018.00 Fourteen percent (14%)

—————— interest from 15 April

Total P 48,429,189.00 1978 to 14 April 1981 46 P 8,376,610.80

—————— Fourteen percent (14%)

Net Income as Adjusted P 48,687,355.00 interest from 21 April

=========== 1983 to 20 April 1986 47 P 11,894,787.00

Income Tax Due Thereon 44 P 17,030,574.00 ——————

Less: Total Deficiency Income Tax

Tax Already Assessed per Due and Payable P 40,215,709.00


Return 80,358.00
===========
——————
WHEREFORE, for all the foregoing, the Decision of the Court of
Appeals is hereby MODIFIED and Picop is hereby ORDERED to pay
the CIR the aggregate amount of P43,794,252.51 itemized as follows:

(1) Thirty-five percent (35%)

transaction tax P 3,578,543.51

(2) Total Deficiency Income

Tax Due 40,215,709.00

———————

Aggregate Amount Due and Payable P 43,794,252.51

============

No pronouncement as to costs.

SO ORDERED.

Narvasa, C.J., Regalado, Davide, Jr., Romero, Bellosillo, Melo, Puno,


\
CIR vs. Itogon Suyoc Mines

Respondent Itogon-Suyoc Mines, a mining corporation duly organized


and existing under Philippine laws fild its income tax return.

Fiscal year (1956-1960) paid PhP 13, 155.20 as the first installment of
the income tax due. Then filed an amended income tax return
reporting a net loss of PhP 331,707.33

Fiscal year (1960-1961) setting forth its income tax liability of PhP
97.345 but deducting the amount of PhP 13,155.20 representing
alleged tax credit for over-payment of the preceding fiscal year 1959-
1960.

Petitioner assessed against the respondent the amount of PhP


1,512.83 as 1% monthly interest. The basis for such an assessment
was the absence of legal right to deduct said amount before the
refund or tax credit thereof was approved by petitioner CIR.

ISSUE: WON respondent corporation is liable to pay the sum of


PhP1,512.83 as 1% monthly interest for delinquency in the payment
of income tax.?

HELD:
NIRC provides that interest upon the amount determined as a
deficiency shall be assessed and shall be paid upon notice and
demand from the CIR at the specified. If in any preceding year the tax
payer was entitled to a refund of any amount due as tax, such
amount, if not yet refunded, maybe deducted from the tax to be paid
Gancayco vs.Collector

Gancyaco files his income tax return for the year 1949. Respondent
issued a warrant of distraint and levy against the properties of
Gancayco for the satisfaction of his deficiency income tax liability, and
accordingly, the municipal treasurer issued a notice of sale of said
property at public auction. Gancayco filed a petition to cancel the sale
and direct that the same be re-advertised at a future date

ISSUE: Whether the sum of PhP 16,860.31 is due from Gancayco as


deficiency income tax for 1949 hinges on the validity of his claim for
deduction:
a) farming expense PhP 27,459
b) representation expenses PhP 8,933.45

HELD:
a)Farming Expenses - no evidence has been presnted as to the
nature of the said farming expenses other than the care statement of
petitioner that they were spent for the development and cultivation of
his property.

No specification has been made as to the actual amount spent for


purchase of tools, equipment or materials or the amount spent for
improvement.

b) Representation expense
PhP 22, 820 is allowed
PhP 8,993.45 is disallowed because of the absence of recipt, invoices
or vouchers of the expenditures in question, petitioner could not
sspecify the items constituting the same when or on whom or on what
they were incurred.
Commissioner of Internal Revenue In the case, Hoskins fails to pass the test. CTA was correct in holding
that the payment of the company to Mr. Hoskins of the sum
Facts: P99,977.91 as 50% share of supervision fees received by the
company was inordinately large and could not be treated as an
Hoskins, a domestic corporation engaged in the real estate business ordinary and necessary expenses allowed for deduction.
as broker, managing agents and administrators, filed its income tax
return (ITR) showing a net income of P92,540.25 and a tax liability of
P18,508 which it paid.

CIR disallowed 4 items of deductions in the ITR. Court of Tax Appeals


upheld the disallowance of an item which was paid to Mr. C. Hoskins
representing 50% of supervision fees earned and set aside the
disallowance of the other 3 items.

Issue:

Whether or not the disallowance of the 4 items were proper.

Held:

NOT deductible. It did not pass the test of reasonableness which is:

General rule, bonuses to employees made in good faith and as


additional compensation for services actually rendered by the
employees are deductible, provided such payments, when added to
the salaries do not exceed the compensation for services rendered.

The conditions precedent to the deduction of bonuses to employees


are:

· Payment of bonuses is in fact compensation

· Must be for personal services actually rendered

· Bonuses when added to salaries are reasonable when


measured by the amount and quality of services performed with
relation to the business of the particular taxpayer.

There is no fixed test for determining the reasonableness of a given


bonus as compensation. This depends upon many factors.
CIR V GENERAL FOODS While the subject advertising expense was paid or incurred within the
corresponding taxable year and was incurred in carrying on a trade or
GR No. 143672| April 24, 2003 | J. Corona business, hence necessary, the parties’ views conflict as to whether or
not it was ordinary. To be deductible, an advertising expense should
Test of Reasonableness not only be necessary but also ordinary.

Facts: The Commissioner maintains that the subject advertising expense was
not ordinary on the ground that it failed the two conditions set by U.S.
Respondent corporation General Foods (Phils), which is engaged in the jurisprudence: first, “reasonableness” of the amount incurred and
manufacture of “Tang”, “Calumet” and “Kool-Aid”, filed its income tax second, the amount incurred must not be a capital outlay to create
return for the fiscal year ending February 1985 and claimed as “goodwill” for the product and/or private respondent’s business.
deduction, among other business expenses, P9,461,246 for media Otherwise, the expense must be considered a capital expenditure to be
advertising for “Tang”. spread out over a reasonable time.

The Commissioner disallowed 50% of the deduction claimed and There is yet to be a clear-cut criteria or fixed test for determining the
assessed deficiency income taxes of P2,635,141.42 against General reasonableness of an advertising expense. There being no hard and
Foods, prompting the latter to file an MR which was denied. fast rule on the matter, the right to a deduction depends on a number
of factors such as but not limited to: the type and size of business in
General Foods later on filed a petition for review at CA, which reversed which the taxpayer is engaged; the volume and amount of its net
and set aside an earlier decision by CTA dismissing the company’s earnings; the nature of the expenditure itself; the intention of the
appeal. taxpayer and the general economic conditions. It is the interplay of
these, among other factors and properly weighed, that will yield a
Issue: proper evaluation.
W/N the subject media advertising expense for “Tang” was ordinary The Court finds the subject expense for the advertisement of a single
and necessary expense fully deductible under the NIRC product to be inordinately large. Therefore, even if it is necessary, it
cannot be considered an ordinary expense deductible under then
Held: Section 29 (a) (1) (A) of the NIRC.
No. Tax exemptions must be construed in stricissimi juris against the Advertising is generally of two kinds: (1) advertising to stimulate
taxpayer and liberally in favor of the taxing authority, and he who claims the current sale of merchandise or use of services and (2) advertising
an exemption must be able to justify his claim by the clearest grant of designed to stimulate the future sale of merchandise or use of services.
organic or statute law. Deductions for income taxes partake of the The second type involves expenditures incurred, in whole or in part, to
nature of tax exemptions; hence, if tax exemptions are strictly create or maintain some form of goodwill for the taxpayer’s trade or
construed, then deductions must also be strictly construed. business or for the industry or profession of which the taxpayer is a
member. If the expenditures are for the advertising of the first kind,
To be deductible from gross income, the subject advertising expense then, except as to the question of the reasonableness of amount, there
must comply with the following requisites: (a) the expense must be is no doubt such expenditures are deductible as business expenses. If,
ordinary and necessary; (b) it must have been paid or incurred during however, the expenditures are for advertising of the second kind, then
the taxable year; (c) it must have been paid or incurred in carrying on normally they should be spread out over a reasonable period of time.
the trade or business of the taxpayer; and (d) it must be supported by
receipts, records or other pertinent papers.
The company’s media advertising expense for the promotion of a single
product is doubtlessly unreasonable considering it comprises almost
one-half of the company’s entire claim for marketing expenses for that
year under review. Petition granted, judgment reversed and set
aside.
CIR V. Isabela Cultural Corp. (2007) (c) it must have been paid or incurred in carrying on the trade or
business of the taxpayer; and
THIRD DIVISION
G.R. No. 172231 February 12, 2007 (d) it must be supported by receipts, records or other pertinent papers.
YNARES-SANTIAGO, J.
Revenue Audit Memorandum Order No. 1-2000, provides that under
Lessons Applicable: Accrual method, burden of proof in accrual the accrual method of accounting, expenses not being claimed as
method, deductibility of ordinary and necessary trade, business, or deductions by a taxpayer in the current year when they are incurred
professional expenses, all events test cannot be claimed as deduction from income for the succeeding year.
Thus, a taxpayer who is authorized to deduct certain expenses and
Laws Applicable: other allowable deductions for the current year but failed to do so
cannot deduct the same for the next year.
FACTS:
The accrual method relies upon the taxpayer’s right to receive amounts
BIR disallowed Isabela Cultural Corp. deductible expenses for or its obligation to pay them, in opposition to actual receipt or payment,
services which were rendered in 1984 and 1985 but only billed, paid which characterizes the cash method of accounting. Amounts of
and claimed as a deduction on 1986. income accrue where the right to receive them become fixed, where
there is created an enforceable liability. Similarly, liabilities are accrued
After CA sent its demand letters, Isabela protested. when fixed and determinable in amount, without regard to
indeterminacy merely of time of payment.
CTA found it proper to be claimed in 1986 and affirmed by CA
The accrual of income and expense is permitted when the all-events
ISSUE: W/N Isabela who uses accrual method can claim on 1986 only test has been met. This test requires: (1) fixing of a right to income or
liability to pay; and (2) the availability of the reasonable accurate
HELD: case is remanded to the BIR for the computation of Isabela determination of such income or liability.
Cultural Corporation’s liability under Assessment Notice No. FAS-1-86-
90-000680. The all-events test requires the right to income or liability be fixed, and
the amount of such income or liability be determined with reasonable
NO accuracy. However, the test does not demand that the amount of
income or liability be known absolutely, only that a taxpayer has at his
The requisites for the deductibility of ordinary and necessary trade, disposal the information necessary to compute the amount with
business, or professional expenses, like expenses paid for legal and reasonable accuracy. The all-events test is satisfied where computation
auditing services, are: remains uncertain, if its basis is unchangeable; the test is satisfied
where a computation may be unknown, but is not as much as
(a) the expense must be ordinary and necessary; unknowable, within the taxable year. The amount of liability does not
have to be determined exactly; it must be determined with "reasonable
(b) it must have been paid or incurred during the taxable year; - qualified accuracy." Accordingly, the term "reasonable accuracy" implies
by Section 45 of the National Internal Revenue Code (NIRC) which something less than an exact or completely accurate amount.
states that: "[t]he deduction provided for in this Title shall be taken for
the taxable year in which ‘paid or accrued’ or ‘paid or incurred’, The propriety of an accrual must be judged by the facts that a taxpayer
dependent upon the method of accounting upon the basis of which the knew, or could reasonably be expected to have known, at the closing
net income is computed of its books for the taxable year.
Accrual method of accounting presents largely a question of fact; such
that the taxpayer bears the burden of proof of establishing the accrual
of an item of income or deduction.

In the instant case, the expenses for professional fees consist of


expenses for legal and auditing services. The expenses for legal
services pertain to the 1984 and 1985 legal and retainer fees of the law
firm Bengzon Zarraga Narciso Cudala Pecson Azcuna & Bengson, and
for reimbursement of the expenses of said firm in connection with ICC’s
tax problems for the year 1984. As testified by the Treasurer of ICC,
the firm has been its counsel since the 1960’s. - failed to prove the
burden

Labels: 2007, Accrual method, all events test, burden of proof in


accrual method, Case Digest, CIR v. Isabela Cultural
Corp., expenses, February 12, G.R. No. 172231, tax case digest
PAPER INDUSTRIES CORPORATION OF THE PHILIPPINES 1. YES. PICOP reiterates that it is exempt from the payment of the
(PICOP) v. CA, CIR and CTA, G.R. Nos. 106949-50 (1995) transaction tax by virtue of its tax exemption under R.A. No. 5186, as
amended, known as the Investment Incentives Act, which in the form it
Facts: existed in 1977-1978, read in relevant part as follows: "SECTION 8.
Incentives to a Pioneer Enterprise. — In addition to the incentives
Paper Industries Corporation of the Philippines (PICOP) is a Philippine provided in the preceding section, pioneer enterprises shall be granted
corporation registered with the Board of Investments (BOI) as a the following incentive benefits: (a) Tax Exemption. Exemption from all
preferred pioneer enterprise with respect to its integrated pulp and taxes under the National Internal Revenue Code, except income tax,
paper mill, and as a preferred non-pioneer enterprise with respect to its from the date of investment is included in the Investment Priorities Plan
integrated plywood and veneer mills. Petitioner received from the x x x”. The Supreme Court holds that that PICOP's tax exemption under
Commissioner of Internal Revenue (CIR) two (2) letters of assessment R.A. No. 5186, as amended, does not include exemption from the thirty-
and demand (a) one for deficiency transaction tax and for documentary five percent (35%) transaction tax. In the first place, the thirty-five
and science stamp tax; and (b) the other for deficiency income tax for percent (35%) transaction tax is an income tax, a tax on the interest
1977, for an aggregate amount of PhP88,763,255.00. income of the lenders or creditors as held by the Supreme Court in the
case of Western Minolco Corporation v. Commissioner of Internal
PICOP protested the assessment of deficiency transaction tax , the Revenue. The 35% transaction tax is an income tax on interest
documentary and science stamp taxes, and the deficiency income tax earnings to the lenders or placers. The latter are actually the taxpayers.
assessment. CIR did not formally act upon these protests, but issued a Therefore, the tax cannot be a tax imposed upon the petitioner. In other
warrant of distraint on personal property and a warrant of levy on real words, the petitioner who borrowed funds from several financial
property against PICOP, to enforce collection of the contested institutions by issuing commercial papers merely withheld the 35%
assessments, thereby denying PICOP's protests. Thereupon, PICOP transaction tax before paying to the financial institutions the interest
went before (CTA) appealing the assessments. earned by them and later remitted the same to the respondent CIR. The
tax could have been collected by a different procedure but the statute
On 15 August 1989, CTA rendered a decision, modifying the CIR’s chose this method. Whatever collecting procedure is adopted does not
findings and holding PICOP liable for the reduced aggregate amount of change the nature of the tax. It is thus clear that the transaction tax is
P20,133,762.33. Both parties went to the Supreme Court, which an income tax and as such, in any event, falls outside the scope of the
referred the case to the Court of Appeals (CA). tax exemption granted to registered pioneer enterprises by Section 8 of
R.A. No. 5186, as amended. PICOP was the withholding agent, obliged
CA denied the appeal of the CIR and modified the judgment against to withhold thirty-five percent (35%) of the interest payable to its lenders
PICOP holding it liable for transaction tax and absolved it from payment and to remit the amounts so withheld to the Bureau of Internal Revenue
of documentary and science stamp tax and compromise penalty. It ("BIR"). As a withholding, agent, PICOP is made personally liable for
also held PICOP liable for deficiency of income tax. the thirty-five percent (35%) transaction tax 10 and if it did not actually
withhold thirty-five percent (35%) of the interest monies it had paid to
Issues: its lenders, PICOP had only itself to blame.
1. Whether PICOP is liable for transaction tax

2. Whether PICOP is liable for documentary and science stamp tax 2. NO. The CIR assessed documentary and science stamp taxes,
amounting to PhP300,000.00, on the issuance of PICOP's debenture
3. Whether PICOP is liable for deficiency income tax bonds. Tax exemptions are, to be sure, to be "strictly construed," that
is, they are not to be extended beyond the ordinary and reasonable
Held: intendment of the language actually used by the legislative authority in
granting the exemption. The issuance of debenture bonds is certainly
conceptually distinct from pulping and paper manufacturing operations.
But no one contends that issuance of bonds was a principal or regular Aggregate Amount Due and Payable P 43,794,252.51
business activity of PICOP; only banks or other financial institutions are
in the regular business of raising money by issuing bonds or other
instruments to the general public. The actual dedication of the proceeds
of the bonds to the carrying out of PICOP's registered operations
constituted a sufficient nexus with such registered operations so as to
exempt PICOP from taxes ordinarily imposed upon or in connection
with issuance of such bonds. The Supreme Court agrees with the Court
of Appeals on this matter that the CTA and the CIR had erred in
rejecting PICOP's claim for exemption from stamp taxes.

3. YES. PICOP did not deny the existence of discrepancy in their


Income Tax Return and Books of Account owing to their procedure of
recording its export sales (reckoned in U.S. dollars) on the basis of a
fixed rate, day to day and month to month, regardless of the actual
exchange rate and without waiting when the actual proceeds are
received. In other words, PICOP recorded its export sales at a pre-
determined fixed exchange rate. That pre-determined rate was decided
upon at the beginning of the year and continued to be used throughout
the year. Because of this, the CIR has made out at least a prima facie
case that PICOP had understated its sales and overstated its cost of
sales as set out in its Income Tax Return. For the CIR has a right to
assume that PICOP's Books of Accounts speak the truth in this case
since, as already noted, they embody what must appear to be
admissions against PICOP's own interest.

Dispositive:

WHEREFORE, for all the foregoing, the Decision of the Court of


Appeals is hereby MODIFIED and Picop is hereby ORDERED to pay
the CIR the aggregate amount of P43,794,252.51 itemized as follows:

(1) Thirty-five percent (35%) transaction


tax P 3,578,543.51

(2) Total Deficiency Income Tax


Due P 40,215,709.00
Commissioner of Internal Revenue vs. Central Luzon Drug
Corporation A tax credit generally refers to an amount that is “subtracted directly
from one’s total tax liability.” It is an “allowance against the tax itself”
G.R. No. 159647 April 15, 2005 or “a deduction from what is owed” by a taxpayer to the government.
A tax credit should be understood in relation to other tax concepts.
One of these is tax deduction – which is subtraction “from income for
Facts: tax purposes,” or an amount that is “allowed by law to reduce income
Respondents operated six drugstores under the business name prior to the application of the tax rate to compute the amount of tax
Mercury Drug. From January to December 1996 respondent granted which is due.” In other words, whereas a tax credit reduces the tax
20% sales discount to qualified senior citizens on their purchases of due, tax deduction reduces the income subject to tax in order to arrive
medicines pursuant to RA 7432 for a total of ₱ 904,769. at the taxable income.

On April 15, 1997, respondent filed its annual Income Tax Return for A tax credit is used to reduce directly the tax that is due, there ought
taxable year 1996 declaring therein net losses. On Jan. 16, 1998 to be a tax liability before the tax credit can be applied. Without that
respondent filed with petitioner a claim for tax refund/credit of ₱ liability, any tax credit application will be useless. There will be no
904,769.00 allegedly arising from the 20% sales discount. Unable to reason for deducting the latter when there is, to begin with, no existing
obtain affirmative response from petitioner, respondent elevated its obligation to the government. However, as will be presented shortly,
claim to the Court of Tax Appeals. The court dismissed the same but the existence of a tax credit or its grant by law is not the same as the
upon reconsideration, the latter reversed its earlier ruling and ordered availment or use of such credit. While the grant is mandatory, the
petitioner to issue a Tax Credit Certificate in favor of respondent citing availment or use is not. If a net loss is reported by, and no other taxes
CA GR SP No. 60057 (May 31, 2001, Central Luzon Drug Corp. vs. are currently due from, a business establishment, there will obviously
CIR) citing that Sec. 229 of RA 7432 deals exclusively with illegally be no tax liability against which any tax credit can be applied. For the
collected or erroneously paid taxes but that there are other situations establishment to choose the immediate availment of a tax credit will
which may warrant a tax credit/refund. be premature and impracticable.

CA affirmed Court of Tax Appeal's decision reasoning that RA 7432


required neither a tax liability nor a payment of taxes by private
establishments prior to the availment of a tax credit. Moreover, such
credit is not tantamount to an unintended benefit from the law, but
rather a just compensation for the taking of private property for public
use.

Issue:
Whether or not respondent, despite incurring a net loss, may still
claim the 20% sales discount as a tax credit.

Ruling:
Yes, it is clear that Sec. 4a of RA 7432 grants to senior citizens the
privilege of obtaining a 20% discount on their purchase of medicine
from any private establishment in the country. The latter may then
claim the cost of the discount as a tax credit. Such credit can be
claimed even if the establishment operates at a loss.

Вам также может понравиться